3250 Note

You might also like

Download as pdf or txt
Download as pdf or txt
You are on page 1of 186

C = erTE*(CT) p* = (e

(r )h

d)/(u d)

dS(t) = ( )S(t)dt + S(t)dZ(t) c(F, K, T) = P(0, T)[FN(d1) KN(d2)]

Derivatives for Actuaries


c p = x(0)exp(rfT) Kexp(rT) df = ftdt + fxdx + 0.5fxx(dx) Ft,T(S) = S(t)e(r )(T t) ui = ln(Si / Si1)
2

PREFACE
In the past decades there have been major changes in insurance industry. To compete with banks and other financial institutions, insurance companies sell products that are not pure insurance policies. Most products have savings components; some contain investment guarantees; some have their face value reset periodically; the returns of a retirement plan in the accumulation phase can be linked to a market equity index; some are even hedge-fund like and provide little insurance protection. Also, derivative securities have become indispensable tools in risk management and asset-liability management. Actuaries, while good at handling diversifiable mortality risk, have little experience in handling the non-diversifiable risk inherited in these new financial products and not enough knowledge to take advantage of the modern risk management tools available in the financial market. To cope with the change in the industry and the competition with CFA and other designations, the Society of Actuaries education system has undergone many changes. While finance elements were virtually absent in exam ten years before, the current exam MFE and two FSA tracks include substantial finance elements. Apart from these, candidates also have to take a number of introductory finance courses in universities in order to obtain exemption for exams. FINA3250 and FINA4280 are designed for students to learn the foundation of mathematical finance. While the materials are fundamental in the understanding of option pricing theory, they are extremely challenging and you will find that they are the most mathematical courses in the IFA curriculum. The materials selected cover the MFE syllabus thoroughly; a few additional topics which enhance the understanding of the theoretical part of the MFE syllabus are also included. Exam MFE aims at depth rather than breadth and is commonly regarded as the hardest preliminary exam. To pass MFE, a thorough reflection on the material is needed and your hard work is absolutely necessary.

DISCLAIMER
In FINA3250 and 4280 you will learn some stochastic models that have entered the hall of fame of rocket science. However, they are just models and there are model risks. Even if the model is correct, it still involves parameter risk: parameters cannot be estimated with 100% accuracy. These two courses focus on a rational framework of derivatives pricing but not the skills in market speculation. I strongly object students to trading futures and derivatives: they are highly leveraged and the risk entailed is far beyond students risk tolerance. If you try to apply the course materials to speculate and lose money, dont blame me! The following are some comments from Warren Buffett, one of the most successful and influential investors in history, on financial valuation and derivatives pricing: [1] Price is what you pay. Value is what you get. [2] The Black-Scholes formula has approached the status of holy writ in finance, and we use it when valuing our equity put options for financial statement purposes. Key inputs to the calculation include a contracts maturity and strike price, as well as the analysts expectations for volatility, interest rates and dividends. If the formula is applied to extended time periods, however, it can produce absurd results. In fairness, Black and Scholes almost certainly understood this point well. But their devoted followers may be ignoring whatever caveats the two men attached when they first unveiled the formula. (From: To the Shareholders of Berkshire Hathaway Inc., 28th Feb, 2009) [3] Upon leaving (getting out of the General Re derivatives business), our feeling about the business mirrored a line in a country song: I liked you better before I got to know you so well. (From: Berkshire Hathaway 2008 Chairmans Letter)

CONTENTS OF BOOK 1
1 Forwards, Futures and Swaps 1.1 Underlying Asset 1.2 Prepaid Forward and Forward 1.3 Futures 1.4 Swaps Appendix 1 How to Equate Forward and Futures Profit? Exercise 1 An Introduction to Options 2.1 Options Markets 2.2 Spreads and Combinations 2.3 Option Bounds and the Effect of Time of Expiration 2.4 Early Exercising Appendix 2 Put-Call Parity Bounds for American Options Exercise 2 Binomial Option Pricing 3.1 Binomial Tree 3.2 Log-normality 3.3 Options on Other Assets 3.4 State Price Vector 3.5 Pricing under P measure Appendix 3 Proof of the Convergence of Binomial Tree Exercise 3 Stock Price Process 4.1 Brownian Motion 4.2 Variations on Brownian Motion 4.3 Variation and Stochastic Integral 4.4 Stochastic Differential Equation 4.5 Its Lemma 4.6 The Black-Scholes Framework Appendix 4A Mean and Variance of It Integral Appendix 4B Exercise 4 Calculating E[X I(X > K)] when X is Lognormal 1 2 5 15 19 28 29 35 36 43 49 51 55 56 62 62 68 71 77 80 83 84 93 93 99 105 109 114 119 123 124 124

The Black-Scholes Model 5.1 Change of Probability Measure 5.2 Market Price of Risk 5.3 The Black-Scholes Equation 5.4 Valuing a Contingent Claim 5.5 Estimation of Volatility and Expected Return 5.6 Violation of Black-Scholes Assumption Appendix 5 The Newton-Raphson Method Exercise 5 Extra Materials for FM 2B.1 More on Buying a Stock Option 2B.2 Paylater Strategy and Equity-linked CD 2B.3 Uses of Derivatives

131 131 136 140 147 153 157 160 161 171 171 173 176

2B

CONTENTS OF BOOK 2
6 Greek Letters and Their Applications 6.1 Greek Letters 6.2 Delta-Hedging 6.3 Gamma Neutrality 6.4 Greek Letters in the Binomial Model Exercise 6 The Multivariate Black-Scholes Model 7.1 Commodity Asset and Lease Rate 7.2 Derivatives on Several Underlying Assets 7.3 Change of Numraire 7.4 Exchange Options 7.5 Further Topics in Currency Options Exercise 7 Exotic Options 8.1 Gap Options 8.2 Asian Options 8.3 Barrier Options 8.4 Multistage Options Exercise 8 Interest Rate Processes 9.1 Put-call Parity 9.2 Binomial Interest Rate Tree 9.3 The Black-Derman-Toy Model 9.4 Blacks Model 9.5 One-Factor Equilibrium Models: Theory 9.6 One-Factor Equilibrium Models: Applications Appendix 9A Bond Pricing under the Vasicek Model Appendix 9B Limit of YTM of Zeros under the CIR Model Exercise 9 1 1 10 17 19 21 32 32 38 44 49 53 63 71 75 69 77 89 94 104 105 110 117 121 125 133 140 141 142

10

Simulation 10.1 Simulating a Random Variate 10.2 Simulation of European Derivatives 10.3 Statistical Applications of Simulation Exercise 10 Mathematical Prerequisite 0.1 Ordinary Differential Equation 0.2 Taylors Theorem 0.3 Solutions of the Kolmogorov Backward Equation Exercise 0

154 154 162 175 183 191 191 196 199 204

VBA User-Defined Functions and Subroutines Summary Notes for Exam MFE Classification of Exam MFE Sample Exam Questions

206 219 238

The following flowchart indicates what material depends on which previous sections. Unlike the textbook which introduces topics according to the level of difficulty, I arrange the materials so that ideas flow logically.

Section 1.3 1.4

Section 1.1 1.2

FIN3250

Section 2.3 2.4

Section 2.1 2.2

Chapter 2B

Chapter 3

FINA2220

Section 5.5 5.6

Section 5.1 5.4

Chapter 4

FIN4280 Chapter 6

Section 9.1 9.3

Section 7.1

Section 9.4 9.6

Chapter 8

Section 7.2 7.3

Section 7.4 7.5

Section 10.2

Section 10.1

In the above, solid boxes are chapters and sections in the MFE syllabus, blue boxes are theoretical chapters and sections that are essential in understanding the materials.

C.Y. Ng

Chapter 1. Forwards, Futures and Swaps

CHAPTER 1
Related Readings

Forwards, Futures and Swaps

Derivatives Markets (2nd edition)


Sections 1.4, 2.1, 5.1 5.4, 5.6 5.7, Appendix 5B, 7.2, 8.1 8.2

Options, Futures & Other Derivatives (7th edition)


Sections 1.1 1.4, 2.2, 2.4, 4.7, 5.1 5.6, 5.9 5.10, 5.14, 7.1, 7.7

Some Remarks on Derivatives Markets (by E. S. W. Shiu)

L earning Objectives Financial assets, short selling, the principle of no arbitrage, forwards and prepaid forwards, forward rate agreements, futures, commodity and interest rate swaps

This course is about derivatives (). A derivative is a financial instrument or contract that has a value determined by the price of something else. An example of a derivative is reinsurance (): Traditionally, insurance industry has hedged its exposure to catastrophic risks using reinsurance. Suppose that an insurance company has a risk exposure of $100 million to hurricane in Hong Kong and wants to limit losses. It can enter into annual reinsurance contracts that cover on a pro rata basis 70% of its losses, subject to a deductible of $10 million. If in a year the total hurricane claims total $60 million, then the reinsurance company would pay the insurance company 50 70% = $35 million and the losses of the insurance company would only be 60 35 = $25 million. More generally, the payoff of the reinsurance contract is Y = 0.7(X 10)+ = 0.7max(X 10, 0), where X is the actual claim size. (This contract is actually a European call.) We are interested in pricing and risk management of derivative contracts. But before going into the theory, we first talk about the various types of the underlying asset ().

FINA 3250 | Derivatives for Actuaries I

C.Y. Ng

Chapter 1. Forwards, Futures and Swaps Section 1. Underlying Assets

Underlying Assets

For a derivative, the payoff is a function of the price of the underlying asset at a future time. The underlying asset is usually a commodity or a financial asset: (a) Commodity includes metals (e.g. gold, copper), agricultural products (e.g. hog, corn) and energy (e.g. crude oil, electricity, natural gas). Even temperature can be an underlying asset. (b) Financial asset includes stocks, stock indexes, foreign currencies and bonds. In this course, we focus on (b). We shall discuss commodity and imaginary assets in FIN4280. Stock (): Some stocks provide cash income known as dividends (). After paying dividends, (theoretically speaking, if tax rate = 0) the share price drops by the amount of dividend per share.

Stock Index (): A stock index is an average of a collection of stock prices. Some stock indexes track the movement of the stock market as a whole (e.g. S&P 500); some are based on the performance of a particular sector (e.g. NASDAQ). For stock indexes containing many stocks, it is common to model the dividends as being paid continuously at a rate that is proportional to the level of the index. The dividend yield of an index is defined as the annualized dividend payment divided by the stock index. Let St be the (ex-dividend) stock index at time t, and be the dividend yield (which is assumed to be constant), then at time (t, t + dt), the stockholder receives dividend of amount per share. St (dt)

Suppose that we have 1 share of the stock index at time 0 and we reinvest all dividends in the index by buying extra shares. Let nt be the number of shares we own at time t. Because the additional number of shares purchased between (t, t + dt) is dnt, we have Stdnt = nt(Stdt) or
dn t = nt. dt

Since n0 = 1, the solution of the differential equation above is nt = exp(t). So if we have 1 share are the beginning, we end up having exp(T) shares at time T. Tailing position If we want to have 1 share of the index at time T, we only need to buy exp(T ) share of the index at time 0 and then reinvest all dividends in the index.
FINA 3250 | Derivatives for Actuaries I 2

C.Y. Ng

Chapter 1. Forwards, Futures and Swaps Section 1. Underlying Assets

Foreign Currencies (): Exchange rate is the value of one unit of the foreign currency measured in the domestic currency. Foreign currency earns interest at the risk-free interest rate rf prevailing in the foreign country. As a result, investing in a foreign currency can be regarded as owing a stock index paying dividends continuously at rate = rf.

Bonds (): Some bonds pay coupons at regular intervals. The coupons are like cash dividends of a stock. Every time a coupon is paid, the dirty price of the bond drops. Some bonds (called pure discount bonds or zeros) do not pay any coupon. They can be used to infer spot rates and forward rates. Convertible bonds (CB,) allows the holder to convert into shares of common stock in the issuing company or cash of equal value, at an agreed-upon price. It is a hybrid security with debt- and equity-like features. For mandatorily CB, conversion to shares is compulsory.

Buying / Longing an Asset If you buy a can of soup, you go to a supermarket. But what if you want to buy an asset? Some assets are listed () in an exchange (). For other assets you go to an over-the-counter (OTC, ) market. In both cases you contact a broker (), who then contacts a market maker to create a trade. Market makers (/) are traders who will buy assets from customers who wish to sell, and sell financial assets from customers who wish to buy. Just like a supermarket which buys commodities from producers at a low wholesale price and then sells at a high retail price, market makers buy financial assets from customers at a low bid price and sell financial assets to customers (who ask the market maker for the financial asset) at a high ask price (also called offer price). The difference between the two prices is called the bidask spread. Moreover, for every trade you have to pay brokerage fee to the broker. Usually brokers keep the financial asset for the investors. One may ask for a physical delivery of the asset (e.g. the certificate of ownership for a stock) but usually investors would not do that (or else he has to delivery it to the broker if he wishes to sell it later). Usually the brokerage contract gives brokers the right to lend shares to other investors.

Example

1. 1

Stock XYZ is bid at $49.75 and offered at $50, and the brokerage fee is 0.3% of the bid or ask price. Suppose you buy 100 shares, and then immediately sell the 100 shares. What is the roundtrip transaction cost?
FINA 3250 | Derivatives for Actuaries I 3

C.Y. Ng

Chapter 1. Forwards, Futures and Swaps Section 1. Underlying Assets

Shorting an Asset If you think an asset S is going to rise (you are bullish), you can buy S to make a profit: (1) you buy the asset at price S0, (2) you own the asset, and hence entitled to dividends, interest or coupons (if any) of the asset, (3) later you sell the asset at a higher price ST to capture the price appreciation (ST S0). If you think the price of the asset is going to decline (you are bearish), how can you make a profit? You can borrow some asset from a broker and do a short-selling (): Step 1: Short selling at time 0 Lender Share borrowing Lending fee, dividends, etc Step 2: Covering the short at time T Lender

Share return Short seller

Short seller Share selling Spot price at time 0

Share purchasing Market

Spot price at time T

Market (1) you borrow one share from a lender,

(2) you sell the asset at price S0 to a market maker and thus receive a cash of S0; the amount S0 is usually called the proceeds from short selling, (3) later you buy back the asset from the market at a lower price ST and return it to the lender, so you capture a profit of (S0 ST), (4) if the asset pays any dividends, interest or coupons before you cover the short position, you have to pay dividends to the lender. If the asset is a commodity, such payments are called lease rate. Lease rate is the payment that one has to make when he borrow an asset. In the above the short seller must always be prepared to cover the short position (since the lender has the right to sell his assets any time), so he has a liability of St at time t before closing the position. However, in practice a short seller typically borrows through a broker (e.g. investment bank), who usually holds a large amount of assets for other investors who go long (e.g. pension funds, mutual funds). If the lender of the asset, who in most cases does not even know that his assets are borrowed, would like to sell the asset, the broker can transfer the asset from another investor to the lender.

FINA 3250 | Derivatives for Actuaries I

C.Y. Ng

Chapter 1. Forwards, Futures and Swaps Section 2. Prepaid Forward and Forward

To avoid the short seller from going away after receiving S0 without covering the short (such is credit risk), the broker may seize S0 at the beginning as collateral (). When the short seller covers the position, the broker returns S0, plus interest. The rate paid on S0 is called the repo rate in bond markets and short rebate in stock markets. (When the short rebate is generally less than the risk-free rate, we shall assume they are equal unless otherwise stated.)

Example

1. 2

Suppose you desire to short-sell 400 shares of JKI, which has a bid price of $25.12 and an ask price of $25.31. You cover the short position 180 days later when the bid price is $22.87 and the ask price is $23.06. There is a 0.3% brokerage commission in the short-sale. (a) What profit do you earn in the short sales? (b) Suppose that the 6-month interest rate is 3% and that you are paid 2% on the short-sale proceeds. How much interest do you lose?

Prepaid Forward and Forward

A spot contract / outright purchase is an agreement to buy an asset S now. The price S0 is set at time 0 The buyer pays the seller S0 at time 0 The seller gives the asset to the buyer at time 0

Suppose a buyer wants to get the asset at time T (called the expiration date). But in order to avoid fluctuation of future asset price, he first contacts a seller at time 0 to lock in a price to be paid. Such a contract is called a prepaid forward if the buyer pays the seller at time 0; a forward () if the buyer pays the seller at time T. No cash changes hand at t = 0. Contract Spot contract Prepaid forward Forward Pay at time 0 0 T Receive S at time 0 T T Payment S0 at 0
F ( S ) at 0
P 0 ,T

F0,T ( S ) at T

For commodity assets (e.g. corns, crude oil) the buyer requires physical delivery of the asset. But for most financial assets, the buyer does not really want the underlying asset. They enter into the contract because they want to speculate or hedge. This leads to Cash Settlement Instead of delivering the asset, the seller pays the value of the asset ST to the buyer at time T.

FINA 3250 | Derivatives for Actuaries I

C.Y. Ng

Chapter 1. Forwards, Futures and Swaps Section 2. Prepaid Forward and Forward

How to Determine Prepaid Forward Price? To determine the prepaid forward price F0PT ( S ), we use , The Law of One Price In an efficient market all identical goods must have only one price. Non-dividend-paying stock Firstly, we assume that S is a non-dividend-paying stock. Consider the following two portfolios: (1) Buy one share of stock at time 0 by paying S0 at time 0. (2) Enter into a prepaid forward for a delivery of one share of stock at time T. Since the two portfolios have identical payoffs of ST at time T, they must have identical value (price) today. This means that F0PT ( S ) = S0 . , Stock paying continuous dividends We amend portfolio (1) slightly so that we have exactly one share at time T: (1) Buy eT shares, with dividends on the shares being reinvested in additional shares. Again, the two portfolios have identical payoffs at time T. They must, therefore, have identical values today. This means that
F0PT ( S ) = eTS0 . ,

Stock paying known discrete dividends (1) Buy one share at time 0, with all dividends invested in a bank account to earn interest. (2) Do (2) and also put PV0,T(Div) into a bank account at time 0 to earn interest. Both portfolios are worth ST + FV0,T(Div) at time T. This means that
F0PT ( S ) + PV0,T(Div) = S0 ,

and hence
F0PT ( S ) = S0 PV0,T (Div) . ,

How to Determine Forward Price? The forward price F0,T(S) is also known as delivery price. For notational convenience, we write F0,T(S) as K. A financial forward contract with cash settlement consists of the following: (1) At time 0, both the buyer and the seller agree on a delivery price K. (2) At time T, the buyer pays the seller the delivery price K and then the seller gives the buyer ST. When netting out the payments, the buyer gets ST K.

FINA 3250 | Derivatives for Actuaries I

C.Y. Ng

Chapter 1. Forwards, Futures and Swaps Section 2. Prepaid Forward and Forward

The payoff of a contract (the cash settlement above) is the value of the position at expiration. A long position is the position of the buyer, while a short position is the position of the seller. For a long position in a forward contract: Payoff For a short position in a forward contract: Payoff

ST

ST

payoff : ST K

payoff : K ST

Payoff of a contract is the cash flow received at expiration. It does not reflect the initial cost / profit of setting up the positions. Since the initial cost / profit are paid / received at time 0, to incorporate the time value of money, profit is defined as Profit = Payoff at time T FVT (initial cash flow at time 0). For forwards, profit = ST K because initial cash flow at time 0 is zero. For prepaid forwards, profit = ST F0PT ( S ) / P(0, T). , To determine K, we notice that to obtain the payoff ST K, we (1) Enter into a prepaid forward contract for a delivery of one share of stock at time T. (2) Borrow KP(0, T) at time 0 from a bank and repay the loan with interest at time T. The cost of the portfolio is F0PT ( S ) KP(0, T). But if K is selected properly, the cost should be 0. , Relation between Prepaid Forward Price and Forward Price F0,T ( S ) = F0PT ( S ) , P(0, T )

The following summarizes the formulas for prepaid forward and forward prices for constant r: Type of Dividend None Discrete Continuous Prepaid Forward Price S0 S0 PV0,T (Div) S0 eT Forward Price S0 erT S0 erT FV0,T (Div) S0 e(r) T

FINA 3250 | Derivatives for Actuaries I

C.Y. Ng

Chapter 1. Forwards, Futures and Swaps Section 2. Prepaid Forward and Forward

Furthermore, define Forward premium: the ratio of forward price to spot price: F0,T (S) / S0 1 F0, ( S ) = r (in the case of continuous dividend) Annualized forward premium: ln T T S0 Cost of carry: consider buying 1 share of stock index by borrowing S at risk-free rate. Interest is paid continuously so that the loan does not grow. Suppose the stock pays continuous dividend and its price stays constant. The interest payment of the loan is S(rdt) and the stock pays dividend S( dt). So the investor has to pay a net amount of S(r )dt and (r ) is the net cost of carrying the position. Constructing an Arbitrage Portfolio An arbitrage opportunity is a trading strategy that costs you nothing (or even better) to enter but produces riskless profit. If there are mis-priced securities, an arbitrage opportunity can be constructed by creating an arbitrage portfolio that buy low, sell high.

Example

2. 1

You are the owner of a large stationery store in Causeway Bay. You observe that a brand of very popular pencil case that you sell in your store at a price of $2.9 is sold in a small supermarket in Shum Shui Po at a price of $2.7. How can you make an arbitrage opportunity?

Example

2. 2

The current annual effective risk-free interest is 10%. You observe that a zero-coupon risk-free bond of face value 1,000 with two years of maturity has a price of 824. Construct an arbitrage portfolio. Assume that you can lend and borrow at the risk-free rate.

FINA 3250 | Derivatives for Actuaries I

C.Y. Ng

Chapter 1. Forwards, Futures and Swaps Section 2. Prepaid Forward and Forward

Example

2. 3

The S&R index spot price is $1100, the continuously compounded risk-free interest rate is 5%, and the dividend yield is 0. (a) Calculate the theoretical six-month forward price. (b) Suppose you observe a 6-month forward price of $1130. What arbitrage opportunity would you undertake?

Example

2. 4

A stock index currently stands at 350. The continuously compounded risk-free interest rate is 8% and the dividend yield on the index is 4%. (a) What should the forward price of a 4-month forward contract on the index be? (b) If the market forward price of the contract is 353, how can you make an arbitrage profit? (c) Suppose that 2 months from now, the stock index is 348. What is the value of the forward contract at that time if the forward price was chosen to be the value in (a)?

FINA 3250 | Derivatives for Actuaries I

C.Y. Ng

Chapter 1. Forwards, Futures and Swaps Section 2. Prepaid Forward and Forward

Observations
(1) Assume that the underlying asset is a stock paying continuous dividends at yield . As in the derivation of forward price, we can replicate a forward contract by purchasing eT shares of the underlying stock and reinvesting all dividends into extra shares, shorting a zero-coupon bond of face value F0,T(S) = S0 e(r)T. Symbolically, Forward = Stock Bond. Thus we call the position long stock, short bond (buying eT shares of the underlying stock and borrowing S0 eT amount of cash at time 0) a synthetic long forward position. (2) Flipping terms around, Forward + (Stock Bond) = 0. That is, if you have just sold short a forward contract of one share of stock to be delivered at time T, you can completely offset your position by entering into a synthetic long forward position. (3) Consider the following transactions: Transaction Long e shares, reinvest all dividends in extra shares Short forward Net position
T

Time 0 S0eT 0 S0eT

Time T ST F0,T(S) ST F0,T(S): non-random

This shows that Bond = Stock Forward. That is, we can create a long bond position earning r by buying stock and then offsetting the risk by shorting a forward contract. Such a synthetic bond position is called a cash-andcarry position. It is so called because one buys the stock using cash and then carries a futures / forward position to hedge the risk. Cash-and-carry trading is an example of basis trading and it is done when the investor feels that either the market stock price is too low (hence you buy low) or the market forward price is too high (hence you sell high). The rate of return on this synthetic bond is called the implied repo rate (). Generally speaking, implied repot rate is the rate of return of borrowing money to buy an asset and delivering it in the forward / futures market. If the forward contract is fairly valued, the implied repo rate would be 1 F0,T ( S ) 1 S 0 e ( r )T ln ln =r. T S 0 e T T S 0 e T Example 2.3 gives you an example of cash-and-carry arbitrage with an implied repo rate of 2 ln(1130/1100) = 5.381% > 5% risk-free rate.

FINA 3250 | Derivatives for Actuaries I

10

C.Y. Ng

Chapter 1. Forwards, Futures and Swaps Section 2. Prepaid Forward and Forward

(4) Note, however that for an investor with different lending and borrowing rates, things become complicated. For an investor with borrowing rate greater than 5.381%, he cannot conduct an explicit arbitrage to earn profit. Only investors who are lending money at rate less than 5.381% can conduct an implicit arbitrage. The cash-and-carry arbitrage becomes a quasi arbitrage opportunity. (5) The reverse of (3) is the synthetic short bond position called reverse cash-and-carry, which is a long forward contract and a short position in the stock. It is done when investors feel that either the market stock price is too high or the market forward price is too low. Example 2.4(b) gives you an example of reverse cash-and-carry. No-Arbitrage Bound with Transaction Costs The discussion so far (except Observation (4) above) was based on the following assumptions: no transaction cost (in reality, cost per transaction = k > 0) and no tax borrowing rate = lending rate = risk-free rate (in reality, rb > rl ) bid price = ask price (in reality, bid price < ask price) buying or selling in large quantities does not cause the price of the underlying asset to change (this is very hard to model and will not be incorporated into the analysis below) If the above are taken into consideration, rather than a single no-arbitrage price, there exists a no-arbitrage bound. In the following, the underlying asset is a non-dividend-paying stock. No-arbitrage bounds of forward price: (S0b 2k)exp(rlT ) F0,T(S) (S0a + 2k)exp(rbT ) Proof: If one believes that the observed forward price is too high, he can cash-and-carry

On the other hand, if one believes that the observed forward price is too low,

FINA 3250 | Derivatives for Actuaries I

11

C.Y. Ng

Chapter 1. Forwards, Futures and Swaps Section 2. Prepaid Forward and Forward

Forward Price and the Expected Future Price Since F0,T(S) is the price to be paid at time T, in order to obtain a random payoff ST, a fundamental question in the pricing of derivative is whether we have E(ST) = F0,T(S). In order to relate the two terms, suppose that the stock earns an expected rate of return of , where can be estimated from CAPM or APT. By investing S0 today in 1 share, at time T the stockholder would have eT shares, which is worth eT ST. As a result, S0 eT = E(eT ST). which means E(ST) = S0e( )T and hence F0,T (S) = E(ST) exp[( r)T ]. Because the stock is a risky investment, usually > r and hence E(ST) > F0,T (S): The forward price predicts too low the future expected stock price. The difference r is called the risk premium. Currency Prepaid Forwards and Currency Forwards (out of FM but in MFE syllabus) Let xt ($/) be the exchange rate between HK Dollars and British Pounds at time t. If you want to get 1 at time T by paying F0PT ( x) HKD at time 0, what is the amount of HKD you should pay? , Consider two portfolios: (1) Exchange x0exp(rfT) HKD into exp(rfT) Pounds, and put the proceedings in a UK bank. (2) Enter into a prepaid forward contract for a delivery of 1 at time T. Since both portfolios yield 1 at time T, they must have identical values today. This means that
F0PT ( x) = x0exp(rfT) ,

Similarly,
F0,T ( x) = e rT F0PT ( x) = x0 exp[(r r f )T ] . ,

Example

2. 5

Suppose the current exchange rate between Euro and Japanese Yen is 0.02 /. The eurodenominated annual continuously compounded risk-free rate is 4% and the yen-denominated annual continuously compounded risk-free rate is 1%. What are the 6-month Euro/Yen and Yen/Euro forward price?

FINA 3250 | Derivatives for Actuaries I

12

C.Y. Ng

Chapter 1. Forwards, Futures and Swaps Section 2. Prepaid Forward and Forward

Forward Rate Agreement () A forward rate agreement is an over-the-counter contract that guarantees a borrowing or lending rate on a given notional principal amount. It can be settled at maturity (called FRA in arrears) or at the time of borrowing or lending (called FRA with Eurodollar-style settlement)

Consider an FRA in arrears where it is specified that the lending interest rate on a notional principal L for the period of time between T1 and T2 is RK. Define R : the actual interest rate observed at time T1 for a maturity T2 RF : the forward interest rate for the period between times T1 and T2 P(t, T) : time-t price of a zero-coupon bond maturing at T

We depart from the usual assumption of continuous compounding and assume that rates RK, RF and R are measured with a compounding frequency corresponding to T2 T1. If T2 T1 = 0.5, then the rates are all in i(2). loan L T1 FRA guarantee: LRK(T2 T1) actual payment: LR(T2 T1) From FINA2210, P(0, T1) [1 + RF(T2 T1)] 1 = P(0, T2). payoff: L(RK R)(T2 T1) T2

What is the cost of the FRA at time 0? Consider the following two portfolios: (1) Short a zero-coupon bond of face value L for a maturity T1, and defer the repayment of principal (with interest) until T2. (2) Long a zero-coupon bond of face value L[1 + RK(T2 T1)] for a maturity T2. Position Short zero of maturity T1 Long zero of maturity T2 Sum Cost at 0 Payoff at T1 Payoff at T2

L P(0, T1)
L[1 + RK(T2 T1)] P(0, T2) L(RK RF)(T2 T1) P(0, T2)

L (defer) 0
0 0

L[1 + R(T2 T1)]


L[1 + RK(T2 T1)] L(RK R)(T2 T1)

The value of the FRA in arrears that specifies an interest rate RK to be earned for L in (T1, T2) V = L(RK RF)(T2 T1) P(0, T2).

FINA 3250 | Derivatives for Actuaries I

13

C.Y. Ng

Chapter 1. Forwards, Futures and Swaps Section 2. Prepaid Forward and Forward

Observations
(1) (2) (3) An FRA in arrears can be priced by discounting cash flow at risk-free rate as if the variable rate is replaced by the forward rate. In order for the FRA to have a zero value at the beginning, RK should be equal to the forward rate. In practice, this is usually done when the FRA is initialized. An FRA that settles at the time of borrowing or lending as a time-T1 payoff of
L( RK R)(T2 T1 ) . 1 + R(T2 T1 )

Example

2. 6
Maturity (in months) 3 6 9 12 15 18 Rate (% per annum) 8.0 8.2 8.4 8.5 8.6 8.7

Assume that the zero-coupon (continuously compounded) yield curve is given by

What is the value of an FRA in arrears that enables the holder to borrow at 9.5% for a threemonth period starting in one year on a principal of $1,000,000? The interest rate 9.5% is expressed with quarterly compounding.

FINA 3250 | Derivatives for Actuaries I

14

C.Y. Ng

Chapter 1. Forwards, Futures and Swaps Section 3. Futures

Futures

Like a forward contract, a futures contract () is an agreement between two parties to buy or sell an asset at a certain time in the future for a certain price. One of the two significant differences between forwards and futures is that forwards are overthe-counter, but futures are traded on exchanges. In Hong Kong, the exchange is HKFE. The exchange specifies certain standardized features of the contract. For example, for Hang Seng Futures Index (), the asset is Hang Seng Index. The delivery date can only be spot, next calendar month and next two calendar quarter months. Further information on HSFI is available on http://www.hkex.com.hk/prod/hsifo/hsifo_c.htm . The counterparty of the futures contract is always the exchange. The exchange matches long and short futures contract internally so that the net position of the exchange is zero. Buyer and seller do not know each other.

Some terminology: Trading date 2007/06/04 2007/06/05 2007/06/06 Futures price of HSI Next calendar Spot month month 20,756 20,830 20,924 20,993 20,789 20,870 Contract volume () 43,162 57,096 39,320 Open interest () 127,483 126,500 125,643

Futures price: It is the delivery price currently applicable to a futures contract. On 2007/06/05, the July futures price of HSI is 20,993. This is the price, exclusive of commissions, at which traders can agree to buy or sell 1 index point for July delivery. Futures price is determined on the floor of exchange by supply and demand. If more traders want to go short than to go long, the futures price goes down. New buyers then enter the market so that a balance between buyers and sellers is maintained. As the delivery month is approached, the futures price converges to the spot price of the asset. Contract size: It specifies the amount of the asset that has to be delivered under one contract. If the asset is an index, it specifies the value of 1 index point. For HSI it is 50 per contract, while for S&P 500 it is 250. Open interest: It is the total number of the long positions, or equivalently, all short positions. Daily price limits: There are typically daily price limits in futures markets. A price limit is a move in the futures price that triggers a temporary halt () in trading. The purpose of daily price limits is to prevent large movements from occurring due to speculative excesses. Closing out (): Most futures traders use futures to hedge risk and hence the vast majority of future contracts do not lead to delivery of the underlying asset. To close a futures position, one enters into the opposite type of trade from the original one.

FINA 3250 | Derivatives for Actuaries I

15

C.Y. Ng

Chapter 1. Forwards, Futures and Swaps Section 3. Futures

The second significant difference between forward and futures contract is the credit enhancement inherited in futures contract. Futures contract minimizes credit risk by (1) having the exchange stands as the counterpart, (2) marking-to-market () daily, (3) imposing margin () requirement. Marking-to-Market and Margin Requirement To illustrate how margin works, consider a trader who contacts a broker on June 3 to buy 2 gold futures that expires on Jun 14. The current futures price is $400 per ounce. The contract size is 100 ounces per contract. The initial margin is $2000 per contract and margin account earns 6% (continuously compounded). The margin account is adjusted daily to reflect the traders gain and loss. The maintenance margin is 75%. If the balance in the margin account falls below the maintenance margin, the trader receives a margin call and is requested to top up the margin account to the initial margin (). If the trader does not top up, the broker closes out the position by selling the contract ().

Fill in the table below: Day Jun 3 Jun 4 Jun 5 Jun 6 Jun 7 Jun 10 Jun 11 Jun 12 Jun 13 Jun 14 Futures Price 400 397 398.2 394.1 392.4 393.8 395.4 391.1 388.3 389.9 390.6 Daily Gain 600 240 820 340 Cumulative Gain 600 360.099 1180.16 1520.35 Margin Account 4000 3400.65759 3641.21665 4000 3660.65759 Margin Call 0 0 1178.185 0

140

1882.38

4461.36789

The profit on the position is 1882.38. Two other methods to obtain the profit: (1) 4461.367 4000exp(12/365 6%) 1178.185exp(9/365 6%) 1155.536exp(2/365 6%) (2) 140 + 320e6%/365 560e6% 2/365 + + 240e6% 10/365 600e11%11/365

FINA 3250 | Derivatives for Actuaries I

16

C.Y. Ng

Chapter 1. Forwards, Futures and Swaps Section 3. Futures

Relation between Futures Price and Forwards Price (Advanced Topic) If the position had been a forward contract, the profit would be (390.6 400) 200 = 140 + 320 560 + 240 600 = 1880. Why do the futures and forward profits differ? With forward contract, the profit or loss is realized at expiration. With futures, the profits or losses are realized daily and are then compounded with interest. The effect of gain and loss is magnified by the time value of money.

If the number of futures contract is adjusted to offset the effect of compounding as in the appendix, then the profits of forward and futures would be the same. This adjustment can only be made if interest rate is a deterministic function of time. If interest rate varies unpredictably, futures and forward profit would differ. Since futures price is determined from supply and demand, while forward price is not, in general they would not be equal. But the difference between forward and futures price for contracts that last only a few months is in most circumstances sufficiently small to be ignored. Moreover, the appendix proves that Under the (unrealistic) assumption that interest rate is not random, forward and futures price for the same delivery date are equal. When interest rate varies unpredictably, we can get a sense of the nature of their relationship by considering if the price of the underlying asset S is positively or negatively correlated with interest rates. Suppose the correlation is positive. When S increases, an investor holding a long futures position makes an immediate gain due to daily settlement and the gain will tend to be invested at a higher-than-average interest rate. When S decreases, the investor will take an immediate loss and the loss will tend to be financed (by margin) at a lower-than-average interest rate. An investor holding a forward is not affected by interest rate. Thus on average, ceteris paribus, a long futures position would outperform a long forward contract. As a result, the futures contract is more attractive and the futures price would be higher than forward price.

Example

3. 1

Give one justification why S is usually positively correlated with interest rates when S is a stock.

FINA 3250 | Derivatives for Actuaries I

17

C.Y. Ng

Chapter 1. Forwards, Futures and Swaps Section 3. Futures

Eurodollar Futures (Advanced Topic) The most popular futures contract on interest rates is the Eurodollar futures. The most commonly traded type is the 3-month Eurodollar futures.

daily mark-to-market

realized rate = RT

T + 0.25 T Futures price at T = 100 RT

To understand Eurodollar futures, we look at the following: A Eurodollar is a deposit denominated in US dollars at foreign banks outside the United States. For example, if you have a foreign currency account in HK and you have some US dollars there, then you have Eurodollar. (Similar concept applies to Euroyen and Euroeuro. Euroeuro means a Euro deposited in a non-EU country.) The 3-month Eurodollar interest rate is the rate of interest earned on Eurodollars deposited for three months by one bank with another bank, that is, 3-month LIBOR (a floating rate). A Eurodollar futures is a futures on a 3-month Eurodollar deposit of 1 million USD, traded at the Chicago Mercantile Exchange (CME). Similar to FRAs, it allows banks and companies to lock in an interest rate today for money it intends to borrow or lend in the future. At the delivery date (T) of the futures, the long position has to lend the short position 1 million at the realized rate RT for 3 months. The interest compounding frequency is 90/360. The futures price is 100 Annualized 3-month rate. E.g. if at expiration, the LIBOR over the next three months is 1.5%, the futures price is FT = 100 100RT = 100 1.5 4 = 94. Prior to T, if the market expects RT to increases, the futures price drops, and vice versa. Let Ft be the futures price at t ( T ). Then the daily settlement for a long position is 1,000,000(0.01Ft 0.01Ft1) 0.25 = 2500(Ft Ft1). As a result, for a daily decrease of 1% in F (which roughly translated into time-t expected value of RT increases by 0.01% = 1 basis point), a long position entails a lose of $25. If we ignore daily settlement, the total gain in one long futures contract over (0, T) is
T

2500 ( Ft Ft 1 ) = 2500( FT F0 ) = 2500(100 100 RT F0 ) .


t =1

The payoff is 250,000(1 0.01F0 RT), which is the payoff of a shorting an FRA with RK = 1 0.01F0 and L = 1,000,000 = 1 million, the only difference being that the payment is made at T but not T + 0.25. Eurodollar futures offer greater liquidity and lower transaction cost than FRAs, and it is free of credit risk through daily mark-to-market and a margining process.

FINA 3250 | Derivatives for Actuaries I

18

C.Y. Ng

Chapter 1. Forwards, Futures and Swaps Section 4. Swaps

Example

3. 2

Suppose the September Eurodollar futures contract has a price of 96.4. You plan to borrow $50 million for 3 months in September at LIBOR, and you intend to use the Eurodollar futures contract to hedge your borrowing rate. (a) What position should you enter into? (b) Assume the realized 3-month LIBOR is 1% in September. What is the settlement in dollars at expiration of the futures contract? Ignore daily marking-to-market on the futures contract.

If we ignore daily mark-to-market interest effect, the payoff of Eurodollar futures is similar to FRA in arrears, except that it is settled based on current 3-month LIBOR, which is the rate quoted for the next 3 months (91 days). Given the price F of the Eurodollar futures with maturity t, we can approximate the implied (non-annualized) 3-month forward rate by

r (t , t + 91 days) =

100 F 91 . 100 360

The above rate is always higher than the true forward rate, and the difference between the two numbers is called convexity bias. For small values of T, the bias is extremely small and can be ignored.
4

Swaps

A forward contract can be thought of as an exchange of the physical asset and the forward price at a future time point: Forward Short position pays 0 Long position pays ST T K

The time-0 value of the forward at time 0 is F0PT ( S K ) = F0PT ( S ) KP (0, T ) = 0 . This gives an , , equation for determining K. A commodity swap () can be thought of as an extension of a forward contract: the commodity asset is delivered and the same swap price K is paid at many time points.

FINA 3250 | Derivatives for Actuaries I

19

C.Y. Ng

Chapter 1. Forwards, Futures and Swaps Section 4. Swaps

Swap
Floating leg pays 0 Fixed leg pays S1 t1 K Si1 ti1 K Si ti K Sn1 Sn tn1 tn K K

In the above figure, n: number of swap payments ti (i = 1, 2, , n): the ith swap payment date K: fixed payment (swap price) Si: the spot price of the commodity at ti Such a contract is useful for hedging. For example, a bakery would be interested in locking in the future prices of flour on a weekly basis. It would be more convenient to enter into the fixed leg of a swap than a series of forward contracts, each of which having a different forward price. We are interested in determining the swap price such that the value of the swap is 0 at t = 0:
0= Thus Swap price that zeroizes the value of a commodity swap at initialization K=

F0P,ti (S K ) = [ F0P,ti (S ) KP(0, t i )] = P(0, t i )[ F0,ti (S ) K ]


i =1 i =1 i =1

(1.1)

P(0, t ) F
i =1 i n i =1

0 ,t i

(S ) . (1.2)

P(0, t )
i n

Equation (1.1) can be written as

P(0, t i ) F0,ti (S ) = K P(0, t i ) .


i =1 i =1

(1.3)

The RHS of (1.3) is the present value of the payments made by the fixed leg. The LHS is the sum of the values of a series of prepaid forward contracts to obtain all the floating payments S1, S2, , Sn. Equation (1.3) leads to the idea of a prepaid swap. For a prepaid swap, the fixed leg pays the floating leg a single payment P at time 0. Later at each ti, the floating leg pays the fixed leg the floating payment Si: Floating leg pays 0 P
FINA 3250 | Derivatives for Actuaries I 20

S1 t1

Si1 ti1

Si ti

Sn1 Sn tn1 tn

C.Y. Ng

Chapter 1. Forwards, Futures and Swaps Section 4. Swaps

The prepaid swap is a generalization of the prepaid forward contract. P is called the prepaid swap price. By (1.3), P = P(0, t i ) F0,ti ( S ) = K P(0, t i ) .
i =1 i =1 n n

Example

4. 1

Suppose that the prepaid swap price of a 3-year swap on corn is $6.50, and that 1-year, 2-year, 3year zero-coupon bond yields are 6.2%, 6.5% and 6.8%. Calculate the 3-year swap price.

Example

4. 2

Consider an industrial producer, IP Inc., that is going to buy 100,000 barrels of oil 1 year from today and 2 years from today. The forward price for delivery in 1 year is $20/barrel and in 2 years is $21/barrel. The 1-year and 2-year zero-coupon bond yields are 6% and 6.5%. (a) Find the price of the 2-year prepaid swap for 100,000 barrels of oil for the next 2 years. (b) Why is the arrangement in (a) risky? (c) Find the 2-year swap price. (d) The counterparty in (c) is a dealer but not an oil producer. Describe how the dealer can hedge its position by using forward contracts. Hence, calculate the internal rate of return of the dealer. Show that the internal rate of return is equal to the 1-year forward rate at year 1. (e) Suppose that immediately after IP Inc. enters into the swap, the forward price curves for oil rise by $2 in years 1 and 2. Assume that interest rates are unchanged. (i) Calculate the new swap price and the market value of the original swap to IP. (ii) Immediately after the first swap payment, the counterparty wants to calculate the value of the swap. Suppose that at that time the 1-year forward price is $19/barrel and the 1-year zero-coupon bond yield becomes 5%. (The value of the swap is the implicit loan balance of the swap to the counterpart.)

FINA 3250 | Derivatives for Actuaries I

21

C.Y. Ng

Chapter 1. Forwards, Futures and Swaps Section 4. Swaps

Example

4. 3

Assume year 1, 2, and 3 forward gold prices are $205, $215, and $230, respectively and do not change with interest rates. An investor enters into a 3-year swap on gold, paying the swap price at the end of every year. Interest rates immediately rise on 1, 2, and 3-year zero coupon bonds from 5.1%, 5.4%, and 5.7% to 5.2%, 5.7%, and 6.2%, respectively. Suppose that the investor unwind the swap by entering a 3-year swap, paying the floating. What is net payment per year for the investor?

Interest Rate Swaps

Interest rate swaps are mainly used to transform floating-rate loan to fix-rate loan, or vice versa. A floating-rate loan is a loan that credits interest according to some floating interest rate. Some commonly used floating rates are Treasury rates: the rate of interest applicable to borrowing by a government in its own currency. Since it is usually assumed that a government will not default, treasury rates are regarded as risk-free rates. LIBOR (London Interbank Offer Rate, ): the rate at which large international banks are willing to lend to other large international banks. LIBOR rates are determined in trading between banks and the rates change as economic conditions change. It is generally higher than Treasury zero rates. Consider, for example, a floating-rate loan with a rate of interest specified as six-month LIBOR plus 0.5% per annum. The life of the loan is divided into six-month periods. For each period, the rate of interest is set at 0.5% per annum above the six-month LIBOR at the beginning of the period. Interest is paid at the end of the period like FRAs.
Illustration of a Plain-Vanilla Interest Rate Swap

Company A issues semiannual coupon bonds of face $100 million and coupon rate 5.5%. Consider a three-year swap initiated on March 1, 2005, in which company B agrees to pay to company A an interest rate of 5% per annum (compounded semiannually) on a notional principal of $100 million and in return company A agrees to pay company B the six-month LIBOR on the same notional principal. The agreement specifies that payments are to be exchanged every six months. 5.5% Company A 5% fix LIBOR
FINA 3250 | Derivatives for Actuaries I 22

Company B

C.Y. Ng

Chapter 1. Forwards, Futures and Swaps Section 4. Swaps

The 5% fix rate is the swap rate of the interest rate swap. Fill in the following table. All cash flows are in millions.
Day Six-month LIBOR 4.2% 4.8% 5.3% 5.5% 5.6% 5.9% 6.4% Cash Flow (A B) 2.1 2.4 2.65 2.75 2.8 2.95 Cash Flow (B A) 2.5 2.5 2.5 2.5 2.5 2.5 Net Cash Flow (A B) 0.4 0.1 +0.15 +0.25 +0.3 +0.45

March 1, 05 Sept 1, 05 March 1, 06 Sept 1, 06 March 1, 07 Sept 1, 07 March 1, 08

By using the swap, company A transforms a fix-rate loan to a floating-rate loan.


Value of an Interest Rate Swap as a Sum of FRAs

Define n: number of swap payments ti (i = 1, 2, , n): the ith swap payment date t0: the time for the first floating interest rate to be applied to the swap (Here we assume that ti ti1 does not depend on i) r(ti1, ti): forward rate in (ti1, ti), (not annualized, and is not continuously compounded) ri: the actual floating interest rate in (ti1, ti) rK: swap rate (not annualized, not continuously compounded) L: notional principal, it is not exchanged but only used to calculate the size of cashflows. The following figure shows the cash flows of the swap: receiver 0 payer t0 r1 Lr1 t1 LrK

Lri1 Lri ti1 ri ti LrK

Lrn1 Lrn rn tn1 tn LrK LrK

LrK

The net payment at time ti to B is L(ri rK). We can treat the contingent cash flow above at time ti as an FRA. The time-0 value of the FRA is L[r(ti1, ti) rK]P(0, ti). The value of the swap to the payer (floating rate recipient) is V(payer) = L P(0, t i )[r (t i 1 , t i ) rK ] .
i =1 n

(1.4)

FINA 3250 | Derivatives for Actuaries I

23

C.Y. Ng

Chapter 1. Forwards, Futures and Swaps Section 4. Swaps

Computing the Swap Rate

Usually when the swap is initialized, the value of rK is selected so that V(payer) is zero. Thus

P(0, t )[r (t
i =1 i

i 1

, t i ) rK ] = 0

or Swap rate that zeroizes the value of an interest rate swap at initialization rK =

P(0, t )r (t
i =1 i n i =1

i 1

, ti ) .
(1.5)

P(0, t )
i

In (1.5), the denominator is the PV of an annuity certain. The numerator is the PV of interest payments implied by the strip of forward rates. Equation (1.5) can be written as n P(0, t i ) rK = n r (t i 1 , t i ) . i =1 P(0, t j )
j =1

This form emphasizes that the swap rate is a weighted average of the forward rates, where zerocoupon bond prices are used to determine the weights. Since P(0, ti1) [1 + r(ti1, ti)] 1 = P(0, ti), equation (1.5) can also be written as
n n P(0, t i 1 ) 1 = P(0, t 0 ) P(0, t n ) . rK P(0, t i ) = P(0, t i ) i =1 i =1 P(0, t i )

Normally t0 = 0 (that is, the swap starts immediately after initialization) so that P(0, t0) = 1 and Swap rate as par yield rK P(0, t i ) + P(0, t n ) = 1
i =1 n

which shows that rK is the par yield of a coupon bond. (Recall that par yield is the coupon rate that causes the bond price to equal its face value when all payments are priced using the yield curve.)

Example

4. 4

Consider a one-year interest rate swap with semi-annual payments. Determine the swap rate and expresses it in annualized terms, using the term structure of LIBOR spot rates given as follows: Days Annualized LIBOR rate Assume a 30/360 day count convention. 180 7.2% 360 8.0%

FINA 3250 | Derivatives for Actuaries I

24

C.Y. Ng

Chapter 1. Forwards, Futures and Swaps Section 4. Swaps

Ninety days later, the term structure is as follows: Days Annualized LIBOR rate 90 7.1% 270 7.4%

Determine the market value of the swap from the perspective of the party paying the floating rate. Assume a notional principal of 15 million.

Variation of Plain-Vanilla Interest Rate Swaps

(1) Deferred Interest Rate Swaps It is just a swap with t0 > 0. It can be priced using the same methodology. (2) Amortizing and Accreting Swaps It is a swap with a non-constant notational principal. Suppose that the notational principal to be applied at the ith payment is Li, and let Qi be the relative notional amount at ti, (e.g., if Li = 100000i, then we may let Qi = i), then rK =

Q P(0, t )r (t
i =1 i i n i =1 i

i 1

, ti ) .

Q P(0, t )
i

If {Qi} is increasing, the swap is accreting. If {Qi} is decreasing, the swap is amortizing.
Implicit Loan Balance of Interest Rate Swap

As already mentioned, the swap rate of an interest rate swap is chosen so that the swap is initially worth zero. This means that the sum of the values of the FRAs underlying the swap is zero, but it does not mean that the value of each individual FRA is zero. In general, some FRAs will have positive values whereas others have negative values (A means floating and B means fix):

FINA 3250 | Derivatives for Actuaries I

25

C.Y. Ng

Chapter 1. Forwards, Futures and Swaps Section 4. Swaps

Value of FRAs (to B)

Value of FRAs (to B)

downward slopping yield curve

upward slopping yield curve

After initialization, if forward rate (or yield curve) changes, the value of all FRAs will change and hence the swap would have non-zero value. Even if interest rate changes according to the yield curve, when some FRA matures, the value of the swap will change. Assume that the yield curve is upward-slopping and the interest rate follows the shape of the yield curve: realized interest rate realized payment (A B)

rK

t As can be seen from the figure for the realized payment, B lends money to A for the first and second period and then A repays B in subsequent payments. Thus the swap can be thought of as a loan where B is the lender. The following figure shows the implicit loan balance of A. Implicit loan balance (of A)

FINA 3250 | Derivatives for Actuaries I

26

C.Y. Ng

Chapter 1. Forwards, Futures and Swaps Section 4. Swaps

Eurodollar Futures and Swap Curve (Advanced Topic)

A swap curve is a plot of swap rates for swaps of various maturities. Given the values of P(0, ti) for various is, we can find r(ti1, ti) and vice versa. By (1.5), to calculate swap rates, we need either values of P(0, ti)s or values of r(ti1, ti)s. Swap writers use Eurodollar futures to hedge their position in interest rate swaps. Thus the swap rates should be consistent with the 3-month LIBOR forward rate curves implied by Eurodollar futures. As a result, we first calculate forward rates from Eurodollar futures prices (ignoring convexity bias), and then use (1.5) to calculate swap rates. Here is an example in McDonald (p.259):
ti Eurodollar futures 98.555 98.01 97.495 97.025 96.6 96.235 95.91 95.65 Forward rates 0.003653 0.00503 0.006332 0.00752 0.008594 0.009517 0.010339 0.010996 P(0, ti) Swap rate quarterly annualized 0.003653 0.014611 0.00434 0.017359 0.005 0.02 0.005624 0.022495 0.006209 0.024836 0.006749 0.026997 0.007249 0.028995 0.007702 0.030808

Jun-04 Sep-04 Dec-04 Mar-05 Jun-05 Sep-05 Dec-05 Mar-06 Jun-06

0.996361 0.991374 0.985136 0.977783 0.969451 0.960311 0.950485 0.940147

The difference between the quarterly forward rates and the quarterly swap rates is known as the swap spread.

Example

4. 5

Eurodollar futures prices with maturities of 3, 6, and 9 months are 89.04, 88.75, and 88.55, respectively. What is the annualized swap rate on 9-month securities? What is the swap spread?

FINA 3250 | Derivatives for Actuaries I

27

C.Y. Ng

Chapter 1. Forwards, Futures and Swaps Section 4. Swaps

Appendix 1: How to Equate Forward and Futures Profit?

From the example in Section 3, we see that the futures profit can be calculated by accumulating the daily gains and losses. Consider entering into a long position at the end of day 0 in 1 futures contract maturing after n days. Let Fi be the futures price at the end of the day.
Day Futures Price Gain/Loss

1 F1 F1 F0

2 F2 F2 F1

i Fi Fi Fi1

n Fn Fn Fn1

For simplicity we assume that interest rate is constant. The case for a deterministic (but perhaps varying) force of interest can be dealt with similarly. Let r be the daily force of interest. The accumulated gain/loss at the end of day n is thus P = (F1 F0)er(n 1) + (F2 F1)er(n 2) + + (Fi Fi1)er(ni) + + (Fn Fn1). Now we consider the following strategy: 1. Take a long position of er(n 1) futures contract at the end of day 0. 2. Increase the long position to er(n 2) futures contract at the end of day 1. 3. Increase the long position to er(n i 1) at the end of day i (i = 2, 3, , n 1) 4. At the end of day n, the long position is 1 futures contract.
Day Futures Price Position Gain/Loss

er(n 1) (F1 F0)er(n 1)

1 F1

er(n i) (Fi Fi1)er(n i)

i Fi

n Fn 1 Fn Fn1

The accumulated gain/loss is P = [(F1 F0)er(n 1)]er(n 1) + + [(Fi Fi1)er(ni)]er(ni) + + (Fn Fn1) = Fn F0. The profit is thus the same as that of a forward contract. If an investor combines the above strategy with an investment of F0ern at the end of day 0 in a bank account, then at the end of day n he would obtain Fn. Since the series of futures contract requires no initial investments, effectively speaking the only cash flows are F0ern (outflow) at the end of day 0 and Fn (inflow) at the end of day n. Notice that the terminal cash flow is the spot price of the asset at the end of day n, and thus this strategy is equivalent to a prepaid forward contract. By law of one price, F0ern = F0Pn ( S ) .
and hence F0 = F0,n(S). Thus when interest rate is deterministic, futures price equals forward price.

FINA 3250 | Derivatives for Actuaries I

28

C.Y. Ng

Chapter 1. Forwards, Futures and Swaps Exercise 1

Exercise 1

The questions marked by (*) are out of the SoA Exam FM syllabus. Section 1.1 Underlying Asset 1. (DM 1.3 and 1.4) ABC stock has a bid price of $40.95 and an ask price of $41.05. Assume there is a $20 brokerage commission. (a) What amount will you pay to buy 100 shares? (b) What amount will you receive for selling 100 shares? (c) Suppose you buy 100 shares, then immediately sell 100 shares with the bid and ask prices being the same in both cases. What is your round trip transaction cost? (d) Repeat by assuming that the brokerage fee is quoted as 0.3% of the bid or ask price. 2. (DM 1.6) Suppose you short-sell 300 shares of XYZ stock at $30.19 with a commission charge of 0.5%. Supposing you pay commission charges for purchasing the security to cover the short-sale, how much profit have you made if you close the short-sale at a price of $29.875? Explain the rationale behind the followings. (a) Sometimes the broker might require the short seller to deposit extra cash (called haircut) apart from the proceeds from short sales as collateral. (b) Sometimes a trustworthy third party (e.g. a bank), instead of the broker, would hold the proceedings from short sales. Section 1.2 Prepaid Forward and Forward 4. (DM 2.7) Suppose XYZ stock pays no dividends and has a current price of $50. The forward price for delivery in 1 year is $55. Suppose the 1-year effective annual interest rate is 10%. (a) Graph the payoff and profit diagrams for a forward contract on XYZ stock with a forward price of $55. (b) Is there any advantage to investing in the stock or the forward contract? Why? (c) Suppose XYZ paid a dividend of $2 per year and everything else stayed the same. Now is there any advantage to investing in the stock or the forward contract? Why? 5. (DM 2.9) An off-market forward contract is a forward where either you have to pay a premium or you receive a premium for entering into the contract. (With a standard forward contract, the premium is zero.) Suppose the effective annual interest rate is 10% and the S&R index is 1000. Consider 1-year forward contracts.

3.

FINA 3250 | Derivatives for Actuaries I

29

C.Y. Ng

Chapter 1. Forwards, Futures and Swaps Exercise 1

(a) Verify that if the forward price is $1100, the profit diagrams for the index and the 1year forward are the same. (b) Suppose you are offered a long forward contract at a forward price of $1200. How much would you need to be paid to enter into this contract? (c) Suppose you are offered a long forward contract at $1000. What would you be willing to pay to enter into this forward contract? 6. (DM 5.2) A $50 stock pays a $1 dividend every 3 months, with the first dividend coming 3 months from today. The continuously compounded risk-free rate is 6%. (a) What is the price of a prepaid forward contract that expires 1 year from today, immediately after the fourth-quarter dividend? (b) What is the price of a forward contract that expires at the same time? 7. (DM 5.4) Suppose the stock price is $35 and the continuously compounded interest rate is 5%. (a) What is the 6-month forward price, assuming dividends are zero? (b) If the 6-month forward price is $35.50, what is annualized forward premium? (c) If the forward price is $35.50, what is the annualized continuous dividend yield? 8. (DM 5.6) Suppose you are a market-maker in S&R index forward contracts. The S&R index spot price is 1100, the risk-free rate is 5%, and the dividend yield on the index is 1.5%. (a) What is the no-arbitrage forward price for delivery in 9 months? (b) Suppose a customer wishes to enter a short index futures position. If you take the opposite position, demonstrate how you would hedge your resulting long position using the index and borrowing or lending. (c) Suppose a customer wishes to enter a long index futures position. If you take the opposite position, demonstrate how you would hedge your resulting short position using the index and borrowing or lending. 9. (DM 5.8) The S&R index spot price is 1100, the risk-free rate is 5%, and the continuous dividend yield on the index is 2%. (a) Suppose you observe a 6-month forward price of 1120. What arbitrage would you undertake? What is the implied repo rate? (b) Suppose you observe a 6-month forward price of 1110. What arbitrage would you undertake? 10. (DM 5.10) The S&R spot price is 1100 and the continuously compounded risk-free rate is 5%. You observe a 9-month forward price of 1129.257. (a) What dividend yield is implied by this forward price? (b) Suppose you believe the dividend yield over the next 9 months will be only 0.5%. What arbitrage would you undertake?
30

FINA 3250 | Derivatives for Actuaries I

C.Y. Ng

Chapter 1. Forwards, Futures and Swaps Exercise 1

(c) Suppose you believe the dividend yield will be only 3% over the next 9 months. What arbitrage would you undertake? 11. (DM 5.15) Suppose the S&R index is 800, and that the dividend yield is 0. You are an arbitrageur with a continuously compounded borrowing rate of 5.5% and a continuously compounded lending rate of 5%. (a) Supposing that there are no transaction fees, show that a cash-and-carry arbitrage is not profitable if the forward price is less than 845.23, and that a reverse cash-and-carry arbitrage is not profitable if the forward price is greater than 841.02. (b) Now suppose that there is a $1 transaction fee, paid at time 0, for going either long or short the forward contract. Show that the upper and lower no-arbitrage bounds now becomes 846.29 and 839.97. (c) Now suppose that in addition to the fee for the forward contract, there is also a $2.40 fee for buying or selling the index. Suppose the contract is settled by delivery of the index, so that this fee is paid only at time 0. What are the new upper and lower noarbitrage bounds? (d) Make the same assumptions as in the previous part, except assume that the contract is cash-settled. This means that it is necessary to pay the stock index transaction fee at both times 0 and 1. What are the new no-arbitrage bounds? (e) Now suppose that transactions in the index have a fee of 0.3% of the value of the index (this is for both purchases and sales). Transactions in the forward contract still have a fixed fee of $1 per unit of the index at time 0. Suppose the contract is cash-settled so that when you do a cash-and-carry or reverse cash-and-carry you pay the index transaction fee both at time 1 and time 0. What are the new upper and lower noarbitrage bounds? (Hint: To handle the time 1 transaction fee, you may want to consider tailing the stock position.) 12. (DM 5.19) Suppose the spot $/ exchange rate is 0.008, the 1-year continuously compounded dollar-denominated rate is 5% and the 1-year continuously compounded yendenominated rate is 1%. Suppose the 1-year forward exchange rate is 0.0084. Explain precisely the transactions you could use (being careful about currency of denomination) to make money with zero initial investment and no risk. How much do you make per yen? Repeat for a forward exchange rate of 0.0083. 13. For a fully leverage contract, one gets the asset at time 0 and pays T. Calculate the price that one needs to pay at time T if he receives (a) 1 share of a stock that pays dividends continuously at rate ; (b) 1 share of a stock that pays discrete dividends. Section 1.3 Futures 14. By using the procedures introduced in Appendix 1, show how you can adjust the futures position on page 16 to equate the profits from futures and forwards.

FINA 3250 | Derivatives for Actuaries I

31

C.Y. Ng

Chapter 1. Forwards, Futures and Swaps Exercise 1

15. (DM 5.12) Suppose the S&P index is currently 950 and the initial margin is 10%. You wish to enter into 10 S&P 500 futures contracts (a) What is the notional value of your position? What is the margin? (b) Suppose you earn a continuously compounded rate of 6% on your margin balance. Your position is marked to market weekly, and the maintenance margin is 80% of the initial margin. What is the greatest S&P 500 index futures price 1 week from today at which you receive a margin call? 16. (Hull 2.11) An investor enters into two long July futures contracts on orange juice. Each contract is for the delivery of 15,000 pounds. The current futures price is 160 cents per pound, the initial margin is $6000 per contract, and the maintenance margin is $4500 per contract. What price change would lead to a margin call? 17. (Hull 2.3) You enter into a short futures contract to sell July silver for $5.2 per ounce on the New York Commodity Exchange. The size of the contract is 5,000 ounces. The initial margin is $4,000 and the maintenance margin is $3,000. What change in the futures price will lead to a margin call? What happens if you do not meet the margin call? 18.* (DM 7.17, 7.18) A lender plans to invest $100m for 150 days, 60 days from today. The rate on a 150-day FRA in arrears is 2.5% (non-annualized). The actual interest rate over that period could be either 2.2% or 2.8%. (a) The lender hedges interest rate risk by entering into a position in FRA. What is the payoff from the FRA on day 210 if the interest rate on day 60 turns out to be 2.8%? (b) The lender considers another way of hedging using 3-month Eurodollar futures. Suppose that the Eurodollar futures contract that matures 60 days from today has a price on day 0 of 94. What position in Eurodollar futures would the lender use to lock in a lending rate? (c) Are there any difference between (a) and (b)? Which position is better? Section 1.4 Swaps 19. (DM 8.2) Suppose that oil forward prices for 1 year, 2 years, and 3 years are $20, $21, and $22. The 1-year effective annual interest rate is 6.0%, the 2-year interest rate is 6.5%, and the 3-year interest rate is 7.0%. (a) What is the 3-year swap price? (b) What is the price of a 2-year swap beginning in one year? (That is, the first swap settlement will be in 2 years and the second in 3 years.) 20. (DM 8.3, 8.4, 8.5) Consider the same 3-year oil swap as in (a) of the previous question. Suppose a dealer is paying the fixed price and receiving floating. (a) What position in oil forward contracts will hedge oil price risk in this position? Verify that the present value of the locked-in net cash flows is zero.

FINA 3250 | Derivatives for Actuaries I

32

C.Y. Ng

Chapter 1. Forwards, Futures and Swaps Exercise 1

(b) How much have the dealer overpaid relative to the forward price after the first swap settlement? What is the cumulative overpayment after the second swap settlement? Verify that the cumulative overpayment is zero after the third payment. (c) Suppose that all interest rates rise 50 basis points (oil forward prices unchanged) immediately after the dealer enters into the swap. What happens to the value of the dealers swap position? 21. (DM 8.8, 8.9, 8.11, 8.13) This question involves some tedious calculations which can be handled effectively using Excel. Consider the following table:
Quarter 1 2 3 4 5 6 7 8 Oil forward price 21.0 21.1 20.8 20.5 20.2 20.0 19.9 19.8 Gas swap price 2.2500 2.4236 2.3503 2.2404 2.2326 2.2753 2.2583 2.2044 P(0, t) 0.9852 0.9701 0.9546 0.9388 0.9231 0.9075 0.8919 0.8763

(a) What is the swap price of a 4-quarter oil swap with the first settlement occurring in the third quarter? (b) Show that the 8-quarter oil swap price is $20.4302, and construct the implicit loan balance for each quarter over the life of the swap assuming all forwards are realized. (c) What are the gas forward prices for each of the 8 quarters? (d) What is the fixed rate in a 5-quarter interest rate swap with the first settlement in quarter 2? 22. Assume the net swap payment is $.50 on a reverse transaction involving a 3-year corn swap. What is the market value of the swap given interest rates on zero coupon treasury bonds are 5.2%, 5.6%, and 6.0% for 1, 2, and 3 years, respectively? 23. An investor enters into a two year swap agreement to purchase crude oil at $43.26 per barrel. Soon after the swap is created forward prices rise and the new swap price on a similar swap is $44.12. If interest rates are 5.0% per year, what is the gain to be made from unwrapping the original swap agreement? 24. Consider a 1-year interest rate swap with quarterly payments with a notional principal of 15 million. Given the following LIBOR term structure, calculate the annualized swap rate. Days Annualized LIBOR rate 90 6.56% 180 6.40% 270 6.21% 360 5.99%

Assume a 30/360 day count convention. 30 days later, the term structure is as follows:

FINA 3250 | Derivatives for Actuaries I

33

C.Y. Ng

Chapter 1. Forwards, Futures and Swaps Exercise 1

Days Annualized LIBOR rate

60 3.84%

150 3.79%

240 3.82%

330 4.06%

Determine the market value of the swap from the perspective of the fixed leg payer. 25. (Hull 7.3) A $100 million interest rate swap has a remaining life of 10 months. Under the terms of the swap, six-month LIBOR is exchanged for 12% per annum (compounded semiannually). The average of the bid-ask rate being exchanged for six-month LIBOR in swaps of all maturities is currently 10% per annum with continuous compounding. The sixmonth LIBOR rate was 9.6% per annum two months ago. What is the current value of the swap to the party paying floating? 26. (Hull 7.10) A financial institution has entered into an interest rate swap with company X. Under the terms of the swap, it receives 10% per annum and pays 6-month LIBOR on a principal of $10 million for 5 years. Payments are made every six months. Suppose that company X defaults on the sixth payment date (end of year 3) when the interest rate (with semi-annual compounding) is 8% for all maturities. What is the loss to the financial institution? Assume that 6-month LIBOR was 9% halfway through year 3. Numerical Answers 1. 2. 5. 6. 7. 8. 9. 10. 11. 13. 15. 16. 17. 18. 19. 20. 21. 22. 23. 24. 25. 26. (a) 4125 (b) 4075 (c) 50 (d) 34.6 4.4025 (b) (c) both 90.91 (a) 46.147 (b) 49 (a) 35.886 (b) 0.0284 (c) 2.16% (a) 1129.26 (a) Short forward, purchase e0.01 shares (reinvest all dividends by buying extra shares), borrow 1120 for 6 months. Implied repo rate = 5.6037% compounded continuously. (a) 1.5% (c) 848.82, 837.44 (d) 851.22, 835.04 (e) 851.36, 834.92 rT rT (a) S0e (b) S0e (a) margin = 237500 (b) 930.88 150 cents per contract $5.4 per ounce (a) 0.3 million (b) 100 long futures contract (a) 20.9519 (b) 21.481 (b) 0.9520, 0.97066 (c) 0.0081 (a) 20.3807 (b) 0.596579, 1.248024, 1.637867, 1.735012, 1.5341, 1.13005, 0.619394, 0 (c) 2.25, 2.60, 2.20, 1.90, 2.20, 2.50, 2.15, 1.80 (d) 1.66% 1.34 1.6 0.0588, 177,000 1.964 million 413000

FINA 3250 | Derivatives for Actuaries I

34

C.Y. Ng

Chapter 2. An Introduction to Options

CHAPTER 2
Related Readings

An Introduction to Options

Derivatives Markets (2nd edition)


Sections 2.2 2.5, 3.1 3.4, 9.1, 9.3, Appendix 9A, Section 11.1

Options, Futures & Other Derivatives (7th edition)


Chapters 9 and 10

Theory of Rational Option Pricing (by R. C. Merton, 1976)


Bell Journal of Economics and Management Science, 4(1)

L earning Objectives Options markets, simple option strategies, put-call parity, early exercising, comparing options with respect to maturity and strike

In Chapter 1 we discussed forwards, futures and swaps. These derivatives are called linear derivatives because their payoffs are linear functions of the underlying asset S. A stochastic model of S is not needed to analyze them. Starting from this chapter, we study option pricing theory. Option pricing theory is mathematically demanding and requires the use of advanced probability theory. Robert Merton and Myron Scholes received the Nobel price in economics in 1997 for establishing a general framework for the pricing of derivatives. We shall follow their approach to derive the BlackScholes equation one of the cornerstones of modern mathematical finance. This chapter gives an overview of options. We shall start with the specification of the contract and then study some of their elementary properties. In the remaining of the course, we shall study how we can price options using a stochastic model of S.

FINA 3250 | Derivatives for Actuaries I

35

C.Y. Ng

Chapter 2. An Introduction to Options Section 1. Options Markets

Options Markets

There are two basic types of options (): A call option () gives the holder the right to buy the underlying asset by a certain date for a certain price. A put option () gives the holder the right to sell the underlying asset by a certain date for a certain price. Here are some details of options: An option gives the holder the right but not the obligation to do something. The option holder may choose not to exercise the option and walk away. The price to buy or sell the underlying asset in the contract is known as the exercise price () or strike price. European options can only be exercised on the expiration date, while American options can be exercised at any time up to the expiration date. One type of non-standard American option is known as a Bermudan option. In this, early exercise is restricted to certain dates during the life of the option.

The date specified in the contract is known as the expiration date or maturity.

There are two kinds of position a trader can take. Long position: the trader has bought the contract. Short position: the trader has sold or written the contract (the writer of an option receives cash up front but has potential liabilities later.)

For European options, let T be the expiration date, ST be the price of the underlying asset at T, and K be the strike. The characterizations of the payoffs to the trader at maturity are as follows: For a long position in a call option: Payoff For a long position in a put option: Payoff

K payoff : (ST K)+

ST

K payoff : (K ST )+

ST

FINA 3250 | Derivatives for Actuaries I

36

C.Y. Ng

Chapter 2. An Introduction to Options Section 1. Options Markets

For a short position in a call option: Payoff

For a short position in a put option: Payoff

ST

ST

payoff : (ST K)+ Recall that profit is defined as

payoff : (K ST )+

profit = payoff at time T FVT (initial cash flow at time 0) . For long positions, the initial cash flow is the cost (< 0) of the option; for short positions, the initial cash flow (> 0) is the profit from writing or selling the option. Moneyness describes whether the payoff of the option would be positive if it were exercised immediately. An in-the-money (ITM, ) option is one that would lead to a positive cash flow to the holder if it were exercised immediately. An at-the-money (ATM, ) option is one that would lead to zero cash flow if it were exercised immediately. An out-of-the-money (OTM, ) option is one that would lead to a negative cash flow if it were exercised immediately.

Underlying Assets of Options The underlying assets of options commonly encountered include stocks, stock indexes, foreign currencies, bonds, and many futures contracts. Stock Option The early over-the-counter options were dividend-protected. If a company declared a cash dividend, strike price for options on the companys stock was reduced on the ex-dividend day by the amount of the dividend. Now both exchange-traded and over-the-counter options are generally not adjusted for cash dividends. Thus the dividends have the effect of reducing the stock price on the ex-dividend date. This is bad news for the value of calls and good news for the value of put because the owner of an option has no right to receive dividends over the life of the option.
37

FINA 3250 | Derivatives for Actuaries I

C.Y. Ng

Chapter 2. An Introduction to Options Section 1. Options Markets

In US exchanges, one option contract gives the holder the right to buy or sell 100 shares at the specified strike price. This contract size is convenient because the shares themselves are normally traded in lots of 100.

Stock Index Option The most popular exchange-traded contracts in US are those on the S&P 500 index (SPX) and the Dow Jones Industrial Index (DJX). Most of the contracts are European (except S&P 100). One contract is usually to buy or sell 100 times the index at the specified strike price. Settlement is always in cash, rather than by delivering the portfolio underlying the index. The cash payment is based on the index value at the end of the day on which exercise instructions are issued. Currencies Option Both European and American foreign currencies options are very common. The size of one contract depends on the currency. For example, in the case of British pounds, a dollar-denominated European call option gives the holder the right to sell 31,250 at maturity at a specified strike price K. The payoff is thus $(31,250xT K)+ where xT is the value of 1 in terms of US dollars at T. Futures Option When an exchange trades a particular futures contract it often also trades futures options on that contract. Note that there are two maturity days: the underlying futures contract maturity day (T + s) and the futures option maturity day (T). Normally a futures option matures shortly before the delivery period in the futures contract. When a call futures option is exercised, the holder acquires a cash settlement equal to the excess of the futures price over the strike, plus a long position in the underlying futures. Get (FT K)+ Futures mature and long futures 0 T T+s

When a put futures option is exercised, the holder acquires a cash settlement equal to the excess of the strike price over the futures price, plus a short position in the underlying futures. At any time t T, Ft might not be equal to St. But if s = 0, then FT = ST and in this case the payoff of a European futures option is the same as that of the European spot option (the regular option to buy or sell the underlying asset).

FINA 3250 | Derivatives for Actuaries I

38

C.Y. Ng

Chapter 2. An Introduction to Options Section 1. Options Markets

Options as Insurance Contracts Options are very useful risk management tools in insurance and finance. (1) Floor Suppose you are a hedge fund manager and you own stocks of a bank. Sub prime loans are harming the banking industry and the stock price would fall, but for some reason you cannot sell the stock. Then you can purchase put option as insurance to hedge against a fall in stock price. This is called a floor or a protective put position.

Example

1. 1

Suppose you buy the S&R index for $1000 and buy a $950-strike six-month European put at a price of $51.777. Sketch the payoff and profit diagram for the index and the combined position after 6 months. Assume that the effective six-month risk-free interest rate is 2%.

(2) Cap Suppose you have just sold short a stock. You will make a profit if stock price declines, but you will suffer from unlimited upside risk. You can hedge the loss by purchasing a call:

FINA 3250 | Derivatives for Actuaries I

39

C.Y. Ng

Chapter 2. An Introduction to Options Section 1. Options Markets

(3) Covered Call and Covered Put A short position of a call (put) always has a negative payoff. A call (put) option writer has an upside (downside) risk of the asset. The risk can be hedged away by simultaneously having a long (short) position in the asset. This is called a covered position, in contrast to a naked position, in which the writer does not own the asset for which the option is underwritten. Another Synthetic Forward Position From Section 1.2, we know that a forward position can be synthesized from stock bond. Now lets see how we can synthesize a forward position from a call and a put. We shall use the notation c(S(t), t; K, T) (resp. p(S(t), t; K, T)) to denote the time-t price of a T-year K-strike European call (resp. put) when the time-t stock price is S(t). When t = 0, we also use the simplified notation c(S(0), K, T).

Example

1. 2

You own a European call and simultaneously write a put of the same strike K and maturity T for the same stock S. (a) Sketch the payoff diagram for the position and illustrate that it is the payoff of a forward contract with delivery price K. (b) Hence, show the put-call parity c(S0, K, T) p(S0, K, T) = F0PT ( S ) Ke rT , , (c) What do you observe if you set K equal to the forward price of the stock?

FINA 3250 | Derivatives for Actuaries I

40

C.Y. Ng

Chapter 2. An Introduction to Options Section 1. Options Markets

In Example 1.2 we have obtained the important put-call parity, which states that the time-0 price of a European call and put on the same stock S must satisfy c(S0, K, T) p(S0, K, T) = F0PT ( S ) Ke rT , , where F0PT ( S ) is the prepaid forward price at time 0 for the stock to be delivered at T. If we , consider the prices of the call and put at time t, then the formula becomes c(St, t; K, T) p(St, t; K, T) = Ft ,P ( S ) Ke r (T t ) , T where Ft ,P ( S ) is the prepaid forward price at time t for the stock to be delivered at time T. Now T we generalize the put-call parity to different kinds of assets: Asset S Non-dividend-paying stock Stock/Bond paying discrete dividends/coupons Stock providing continuous yield Futures Foreign currency Cash account (K) providing continuous yield r Prepaid forward price St St PVt,T (Div) or Bt PVt,T (Coupons) St exp[ (T t)] Ft exp[r (T t)] xt exp[rf (T t)] K exp[r(T t)]

You will be asked to prove the case of futures and foreign currencies in Exercise 2.7 and 2.9.

?
(i) (ii)

Example

1. 3 [07 May MFE #1]

On April 30, 2007, a common stock is priced at $52.00. You are given the following: Dividends of equal amounts will be paid on June 30, 2007 and September 30, 2007. A European call option on the stock with strike price of $50.00 expiring in six months sells for $4.50.

(iii) A European put option on the stock with strike price of $50.00 expiring in six months sells for $2.45. (iv) The continuously compounded risk-free interest rate is 6%. Calculate the amount of each dividend. Assume each month is equivalent to 1/12 years. (A) $0.51 (B) $0.73 (C) $1.01 (D) $1.23 (E) $1.45 (Ans: B)

FINA 3250 | Derivatives for Actuaries I

41

C.Y. Ng

Chapter 2. An Introduction to Options Section 1. Options Markets

Example

1. 4

A 4-month European 60-strike call on a stock is currently selling for 8. The current stock price is 64 and a dividend of 0.8 is expected in 1 month. The continuously compounded risk-free interest rate is 12%. (a) Calculate the no-arbitrage price of a 4-month European 60-strike put of the stock. (b) Construct an arbitrage portfolio if the market put price is 3, assuming that an investor can lend or borrow at the risk-free interest rate.

FINA 3250 | Derivatives for Actuaries I

42

C.Y. Ng

Chapter 2. An Introduction to Options Section 2. Spreads and Combinations

Spreads and Combinations

A spread trading strategy involves taking a position in two or more options of the same type (i.e., two or more calls or two or more puts). Bull and Bear Spreads A bull spread can be created by buying a call on a stock with a certain strike and selling a call on the same stock with a higher strike. Payoff

Payoff K1 payoff : (ST K1)+ Payoff ST

K2

ST

K1

K2

payoff : (ST K2)+

Example

2. 1

(a) For K1 < K2, show that 0 c(S0, K1, T) c(S0, K2, T) (K2 K1)erT, Prove a similar inequality for puts. (b) Bull spreads can also constructed using puts. Consider buying a low-strike put and selling a high-strike put. Sketch the payoff diagram and hence explain the meaning of the name bull spread. What is the difference between bull spreads constructed from calls and puts?

FINA 3250 | Derivatives for Actuaries I

43

C.Y. Ng

Chapter 2. An Introduction to Options Section 2. Spreads and Combinations

(c) A bear spread can be constructed by (i) buying a call on a stock with a certain strike and selling a call on the same stock with a lower strike, or (ii) buying a put on a stock with a certain strike and selling a put on the same stock with a lower strike. Draw the payoff diagrams of (i) and (ii). (d) A ratio spread is constructed by buying m calls at one strike and selling n calls at a different strike, with all options having the same maturity and underlying assets. Construct the payoff diagram for a ratio spread constructed by buying two 950-strike calls and selling three 1500strike calls.

FINA 3250 | Derivatives for Actuaries I

44

C.Y. Ng

Chapter 2. An Introduction to Options Section 2. Spreads and Combinations

Butterfly Spreads A butterfly spread (or sandwich spread) involves positions in options of the same type with three different strikes (K1 < K2 < K3). We define K K2 = 3 K 3 K1 so that K2 = K1 + (1 )K3. If = 0.5, then K2 is midway between K1 and K3 and the butterfly spread is symmetric. A long position in a butterfly spread can be created by buying calls with strike K1 and (1 ) calls with strike K3, and selling one call with a strike K2. What is the payoff diagram of the butterfly spread above? Payoff long calls with strike K1 short 1 call with strike K2 long (1 ) calls with strike K3 Butterfly spread Payoff ST < K1 0 0 0 0 K1 ST K2 (ST K1) 0 0 K2 < ST K3 (ST K1) (ST K2) 0 ST > K3 (ST K1) (ST K2) (1 )(ST K3)

ST K1 K2 K3

Example

2. 2

Show that puts?

c( K1 ) c( K 2 ) c( K 2 ) c( K 3 ) . What is the corresponding formula for European K 2 K1 K3 K2

FINA 3250 | Derivatives for Actuaries I

45

C.Y. Ng

Chapter 2. An Introduction to Options Section 2. Spreads and Combinations

To summarize, for K1 < K2 < K3, European 0 c(K1) c(K2) (K2 K1)erT Price constraint 0 p(K2) p(K1) (K2 K1)erT
c( K1 ) c( K 2 ) c( K 2 ) c( K 3 ) K 2 K1 K3 K2 p( K 2 ) p( K1 ) p( K 3 ) p( K 2 ) K 2 K1 K3 K 2

American 0 C(K1) C(K2) K2 K1 0 P(K2) P(K1) K2 K1


C ( K1 ) C ( K 2 ) C ( K 2 ) C ( K 3 ) K 2 K1 K3 K2 P( K 2 ) P( K1 ) P( K 3 ) P( K 2 ) K 2 K1 K3 K2

Convexity constraint

Example

2. 3
Strike Call price Put price 80 22 4 100 9 21 105 5 24.8

Suppose European call and put prices are given by

Demonstrate that there exists an arbitrage opportunity.

FINA 3250 | Derivatives for Actuaries I

46

C.Y. Ng

Chapter 2. An Introduction to Options Section 2. Spreads and Combinations

A combination is an option trading strategy that involves a position in both calls and puts on the same stock. Collars A collar (or range forward contract) can be created by a long K1-strike put and a short K2-strike call (K1 < K2) with the same underlying asset and expiration date. K2 K1 is called the collar width.

Example

2. 4

Suppose we sell a 45-strike call with a price of 0.97 and buy a 40-strike put with a price of 1.99. Sketch the payoff diagram of the collar formed.

The diagrams above shows that a collar looks like a short forward: it benefits from stock price decreases and suffers losses from stock price increases. When K1 and K2 are selected so that the prices of the constituent call and put are equal, we obtain a zero-cost collar or a cylinder option. The zero-cost collar guarantees that the underlying stock can be purchased for a price between K1 and K2 at the maturity of the options, no matter how high or low the stock price becomes. The combined position of a long collar and a long stock is called a collared stock position, and payoff diagram looks like a bull spread. Straddles and Strangles Like the butterfly spread, these are positions commonly used for volatility speculations. A bottom straddle involves buying a call and a put with the same strike K and expiration. By considering the payoff and profit diagram, a straddle gives significant profits when there is a sufficient deviation of terminal stock price from the strike. A bottom straddle is usually not cheap because two options have to be bought.

FINA 3250 | Derivatives for Actuaries I

47

C.Y. Ng

Chapter 2. An Introduction to Options Section 2. Spreads and Combinations

Payoff

Profit

K K payoff = |ST K| ST

ST

The reverse position of a bottom straddle is the top straddle (or written straddle). A top straddle is a bet that the volatility of the stock is low. This is a highly risky position. The loss arising from a large move in stock price is unlimited. To lower the cost of a bottom straddle, we consider buying a strike-K1 put and a strike-K2 call where K1 < K < K2. Both the call and the put become cheaper, but the stock price has to move farther in a strangle than in a straddle for the trader to make a profit: Payoff

ST K1 K2

Different spreads and combinations can also be combined together. See the following:

Example

2. 5

A box spread is a combination of a bull call spread with strike prices K1 and K2 and a bear put spread with the same strike prices. The expiration date is the same for all options. What are the characteristics of a box spread?

FINA 3250 | Derivatives for Actuaries I

48

C.Y. Ng

Chapter 2. An Introduction to Options Section 3. Option Bounds and the Effect of Time

Option Bounds and the Effect of Time of Expiration

Let c and C be the prices of European and American calls and p and P the prices of European and American puts on the same underlying asset, strike and maturity. First we look at two facts: Maximum and Minimum Option Prices (1) For calls, [ Ft ,P ( S ) Ker(T t) ]+ c(St, t; K, T) Ft ,P ( S ) , T T (2) For puts, c(St, t; K, T) C(St, t; K, T) St . [Ker(T t) Ft ,P ( S ) ]+ p(St, t; K, T) Ker(T t), T p(St, t; K, T) P(St, t; K, T) K . Proof:

FINA 3250 | Derivatives for Actuaries I

49

C.Y. Ng

Chapter 2. An Introduction to Options Section 3. Option Bounds and the Effect of Time

Time to Expiration The value of American options increases with time to expiration. That is, for T2 > T1 > t, C(St, t; K, T2) C(St, t; K, T1) and P(St, t; K, T2) P(St, t; K, T1). To see this, consider two American call options that differ only with T. The owner of the longerlife option has all the exercise opportunities open to the owner of the short-life option and more. The longer-life option must, therefore, always be worth at least as much as the short-life option. European options do not necessarily become valuable as the time to expiration increases. The owner of a long-life European option has different exercise opportunities when compared with the owner of a short-life European option. Example 1: Imagine the board of directors of a stock has just announced that the stock will pay a liquidating dividend 2 weeks from today. Compare the values of two European calls options, one expiring 1 week from today and one expiring 3 weeks from today. Example 2: Imagine that you hold European put options of a stock, one expiring 1 week from today and the other expiring 3 weeks from today. The company issuing the stock has just gone bankrupt. Which put option gives you a better discounted payoff? If the underlying asset is a stock that does not pay dividends, then because the European call and American call are the same, it follows that the value of the European call increases with time to expiration. This rules out the situation in Example 1 above. The following is a related result, which applies only to non-dividend-paying stocks. (See Exercise 6.18 for the case when the stock pays dividends.) European Options when the Strike Grows over Time Let c(T) = c(S0, KerT, T) and p(t) = p(S0, KerT, T). If the stock does not pay dividend, then c(T) and p(T) are both increasing in T. Proof:

FINA 3250 | Derivatives for Actuaries I

50

C.Y. Ng

Chapter 2. An Introduction to Options Section 4. Early Exercising

Early Exercising

American Call If we have an American option on a stock, should we use the right to early exercise? To see if the right of exercising is useful, we look at the case when we call for a physical settlement for the underlying stock: Exercise now (at t) Action: pay K now, get 1 share (i) Difference in interest versus Wait until T Action: put K in a bank to earn interest

(ii) Difference in dividend

(iii) Payoff at T: implicit put protection

From the above, we see that the only advantage of early exercising is get you get the stock early and hence get more dividends. This also suggests (1) If the stock pays no dividends, then one should not exercise the call until T; (2) It is only optimal to exercise an American call at maturity or just before a dividend payment. By exercising earlier than that, we pay K prematurely and lose interest as well as the implicit put protection.

FINA 3250 | Derivatives for Actuaries I

51

C.Y. Ng

Chapter 2. An Introduction to Options Section 4. Early Exercising

One may however argue that it may still be optimal to exercise early if one immediately sells the stock (i.e. settle in cash) and hence get a dollar amount of St K. For example, it may happen that St > 2K and you expect ST < K with high uncertainty (e.g. you have insider information). Then wouldnt it be optimal to early exercise the call and get St K?

Example

4. 1

Formulate a strategy using an American call so that no matter how large St is, you can get higher profit than settling in cash if K[1 er(T t)] > PVt,T(Div).

Sufficient Condition for NOT Exercising an American Call If K[1 er(T t)] > PVt,T(Div), (2.1)

then it is not rational to exercise at t. In particular, it is NEVER optimal to exercise an American call on a non-dividend-paying stock before expiration. The value of such an American call is equal to its European counterpart. Proof: It would make sense not to exercise only if the call price C(St, K, T t) is greater than the payoff St K: a better strategy is to sell the option. By put-call parity, C(St, t, K, T) c(St, t; K, T) = St K + p(St, t; K, T) + K[1 er(T t)] PVt,T(Div). If p(St, t; K, T) + K[1 er(T t)] > PVt,T(Div), (2.2)

then C(St, t; K, T) > St K and we would not early exercise. A sufficient condition for (2.2) is (2.1).

FINA 3250 | Derivatives for Actuaries I

52

C.Y. Ng

Chapter 2. An Introduction to Options Section 4. Early Exercising

Observations
(1) For an American call on a non-dividend-paying stock, if you early-exercise, you throw away p(St, t; K, T), that is, the implicit put protection should the stock later move below the strike price (in that case you will regret exercising it), pay K now and hence lose interest on K if you exercise it later.

(2) Equation (2.1) is not a necessary condition for not early exercise the option. Equation (2.2) clearly suggests that if p(St, t; K, T t) is large enough, then even if (2.1) does not hold, we may still have C(St, t, K, T) St K. This is the case if S is volatile, or if T t is large (i.e., t is small). (3) In general, two cases can occur: (i) (2.1) holds: not optimal to exercise no matter how large St is. C(St, t, K, T) volatility (ii) (2.1) does not hold: exercise is optimal when St is sufficiently large. C(St, t, K, T) volatility

St K C(St, t, K, T) > St K for any St K S* C(St, t, K, T) = St K when St > S*

St

In case (ii), the value of S* is the exercise boundary of the American call. It is the minimum value of St that would induce early exercise. See the figure below for the variation of exercise boundary with volatility and t. S* Larger volatility Smaller volatility

K t T

FINA 3250 | Derivatives for Actuaries I

53

C.Y. Ng

Chapter 2. An Introduction to Options Section 4. Early Exercising

Example

4. 2

Prove that if it is optimal to early-exercise an American call, it is also optimal to early-exercise an otherwise identical call that is more in-the-money.

American Puts It may be optimal to exercise American puts early, even if the stock does not pay dividends. Example: Consider a company is just bankrupt at t. Since the Su = 0 for t u T, the payoff of exercising at any time t is K. It is thus optimal to exercise the put right away to receive K, or else you lose interest on K. Dividends do not really matter because the company has bankrupt.

Sufficient Condition for NOT Exercising an American Put If PVt,T(Div) > K[1 er(T t)], (2.3)

then it is not rational to exercise at t. Also, it is never optimal to exercise an American put just before a dividend payment. Proof: We can use the previous 3 arguments to establish this result. I will let you do the first two heuristic arguments on your own. For a mathematical proof, we use the put-call parity: P(St, t; K, T) p(St, t; K, T) = K St + c(St, t; K, T) K[1 er(T t)] + PVt,T(Div). If (2.3) holds, then P(St, t; K, T) > K St and it is not optimal to exercise at t. If (2.3) does not hold, then there is a maximum value of St such that P(St, t, K, T) = K St when St < S*. This S* is again the exercise boundary. See the figure on the right for the variation of exercise boundary with volatility and t. S* K

Smaller volatility Larger volatility t T


FINA 3250 | Derivatives for Actuaries I 54

C.Y. Ng

Chapter 2. An Introduction to Options Section 4. Early Exercising

Finally, we present a table which summarizes many results in this chapter. European Call Put + + ?? ?? + + + + American Call Put + + + + + + + +

Variable Stock price St Strike price K Time to expiration T Dividends Risk-free rate r Volatility

Appendix 2: Put-Call Parity Bounds for American Options (a) For American options on a stock which pays known dollar dividends, S0 PV0,T (Div) K C P S0 KerT (b) For American options on a stock index providing a continuous dividend yield , eTS0 K C P S0 KerT Proof: (i) For the first half of the inequality in (a), consider (1) one European call plus K + PV0,T (Div) invested at risk-free rate (2) one American put plus 1 share of stock For (1), the cost at time 0 is c + K + PV0,T (Div). At time T, it is worth

( S T K ) + + Ke

rT

Ke rT + FV0,T (Div) if S T < K + FV0,T (Div) = rT S T + K (e 1) + FV0,T (Div) if S T K

(2) is subject to early exercise. Notice that if the put is exercised, then the investor delivers the underlying asset and thus cannot earn the dividends of the stock after exercising the put.
Payoff at time T not early exercised early exercise at time t ST < K (K ST) + [ST + FV0,T (Div)] [K + FV0, t(Div)]er(T t) ST K ST + FV0,T (Div) [K + FV0, t(Div)]er(T t)

The portfolio is worth P + S0. Since the payoff for (2) at time T is always less than that of (1), P + S0 < c + K + PV0,T (Div) < C + K + PV0,T (Div). The proof for the case of continuous dividends is simpler. (ii) For the second half of the inequalities, consider the case of continuous dividends (3) one American call plus KerT invested at risk-free rate (4) one European put plus 1 share of stock, with dividends reinvested in the stock. Complete the proof yourself using a method similar to (i).
Increases if the underlying stock pays no dividends.

FINA 3250 | Derivatives for Actuaries I

55

C.Y. Ng

Chapter 2. An Introduction to Options Exercise 2

Exercise 2

Section 2.1 Options Markets 1. (DM 3.10) You are given the following prices for S&R European options with 6 months to expiration: Strike Call Put 950 120.405 51.777 1000 93.809 74.201 1050 71.802 101.214 (a) Construct payoff and profit diagrams for the purchase of a 1050-strike S&R call and sale of a 950-strike S&R call. (b) Verify that you obtain exactly the same profit diagram for the purchase of a 1050-strike S&R put and sale of a 950-strike S&R put. (c) Find the difference in the initial cost of the two positions in (a) and (b). 2. (DM 3.14) Consider the information in question 1. Suppose you buy a 950-strike S&R call, sell a 1000-strike S&R call, sell a 950-strike S&R put, and buy a 1000-strike S&R put. (a) Verify that there is no S&R price risk in this transaction. (b) What are the initial cost and the value of the position after 6 months? (c) What is the effective 6-month interest rate? 3. (DM 9.1) A stock currently sells for 32. A 6-month European call with a strike of 35 has a price of 2.27. Assuming a 4% continuously compounded risk-free rate and a 6% continuous dividend yield, what is the price of a 6-month European put with a strike of 35? (DM 9.2) A stock currently sells for $32. A 6-month European 30-strike call has a price of 4.29, and a 6-month European 30-strike put has a price of 2.64. Assume a 4% risk-free rate. What is the present value of dividends payable over the next 6 months? (Hull 9.11) A 4-month European 60-strike call on a dividend-paying stock is currently selling for 5. The stock price is 64 and a dividend of 0.8 is expected in 1 month. The riskfree interest rate is 12% per annum. What opportunities are there for an arbitrageur? (Hull 9.14) The price of a European 30-strike call that expires in 6 months is 2. The underlying stock price is 29, and a dividend of 0.5 is expected in 2 months and again in 5 months. The term structure is flat, with all risk-free interest rates being 10%. (a) What is the price of a European put that expires in 6 months and has a strike price of 30? (b) Explain carefully the arbitrage opportunities if the European put price is 3.

4.

5.

6.

FINA 3250 | Derivatives for Actuaries I

56

C.Y. Ng

Chapter 2. An Introduction to Options Exercise 2

7.

Consider European call and put futures options, both with strike price K and time to expiration T. The underlying futures (with a time to expiration T) has a price of F0 currently. We form two portfolios: (1) A European call futures option plus KerT dollars invested in a bank at risk-free rate r. (2) A European put futures option plus a long futures contract plus F0erT dollars invested in a bank at risk-free rate r. (a) Assuming that there is no difference between the payoffs from futures and forward contracts, prove that both portfolios are worth max(FT, K) at T. (b) Hence, prove the put-call parity for futures options. (c) Comment on the statement A futures price is like a stock paying a continuous dividend yield. What is the continuous dividend yield? (d) (Out of the MFE syllabus) Show that if the options are American, then F0erT K C P F0 KerT using a similar approach as illustrated in Appendix 2. (See also Exercise 36.) Compare this with the put-call parity bound for stock index. Does it reconfirm (c)?

8.

(Hull 16.16) Suppose that a futures price is currently 35. A European call option and a European put option on the futures with a strike price of 34 are both priced at 2 in the market. The risk-free interest rate (compounded continuously) is 10% per annum. Identify an arbitrage opportunity. Assume that both options have one year to maturity. Prove the put-call parity for European foreign currency options.

9.

10. (DM 9.4) Suppose the current exchange rate is 0.95$/, the euro-denominated continuously compounded interest rate is 4%, the dollar-denominated continuously compounded interest is 6%, and the price of a 1-year 0.93-strike European call on the euro is $0.0571. What is the price of a 0.93-strike European put? Section 2.2 Spreads and Combinations 11. (Hull 10.8) Use put-call parity to relate the initial investment for a bull spread created using calls to the initial investment for a bull spread created using puts.

12. (Hull 10.11) Use put-call parity to show that the cost of a butterfly spread created from European puts is identical to the cost of a butterfly spread created from European calls. 13. (DM 3.13) Using the information in Exercise 2.1, draw profit diagrams for (a) 1050-strike S&R straddle; (b) Written 950-strike S&R straddle; (c) Simultaneous purchase of a 1050-strike straddle and sale of a 950-strike S&R straddle. 14. Show that the prices in Exercise 1 do not violate price constraints and convexity constraints.

FINA 3250 | Derivatives for Actuaries I

57

C.Y. Ng

Chapter 2. An Introduction to Options Exercise 2

15. (Hull 10.7) A 50-strike call costs 2. A put with a strike price of 45 costs 3. Explain how a strangle can be created. What is the pattern of profits from the strangle? 16. (DM 3.11) Consider the option prices in Exercise 2.1. Suppose you invest in the S&R index for 1000, buy a 950-strike put, and sell a 1050-strike call. (a) Draw a profit diagram for this position. (b) If you want to construct a zero-cost collar keeping the put strike equal to 950, in what direction would you have to change the call strike? 17. Let p(K, T) be the price of a European T-year K-strike put on a stock. If the continuously compounded risk-free rate of interest is 0.15, find the maximum and minimum of p(25, 1.5) p(30, 1.5). 18. (DM 3.18) Show that the following three portfolios all give the same butterfly spread: (a) Buy one 35-strike call and 45-strike call, and sell two 40-strike calls. (b) Buy one 35-strike put and 45-strike put, and sell two 40-strike puts. (c) Buy one share of the underlying asset (which does not pay dividends), borrow 35erT, buy one 35-strike put and one 45-strike call, and sell two 40-strike calls. Use the put-call parity to show that the three portfolios have the same initial cost. 19. (DM 9.9) Suppose European call and put prices are given by
Strike 50 55 Call 16 10 Put 7 14

What no-arbitrage property is violated? Construct an arbitrage opportunity. 20. (DM 9.10) Suppose European call and put prices are given by
Strike Call price Put price

50 18 7

55 14 10.75

60 9.5 14.45

What no-arbitrage property is violated? Construct an arbitrage opportunity. 21. You are given the following information on prices of European calls with the same underlying stock and maturity: Strike 35 38 Call prices 4 2

What are the maximum and minimum price of a 37-strike call on the same underlying and maturity?

FINA 3250 | Derivatives for Actuaries I

58

C.Y. Ng

Chapter 2. An Introduction to Options Exercise 2

22. Given the same information as in the previous exercise, find the minimum and maximum price of a 33-strike call on the same underlying and maturity. 23. A 4-month European 60-strike call on a dividend-paying stock is currently selling for 8. The current stock price is 64 and a dividend of 0.8 is expected in 1 month. The continuously compounded risk-free interest rate is 12%. (a) Calculate the no-arbitrage price of a 4-month European 60-strike put of the stock. (b) Construct an arbitrage portfolio if the market put price is 3, assuming that an investor can lend or borrow at the risk-free interest rate. Section 2.3 Option Bounds and the Effect of Time to Expiration 24. (Hull 9.3) What is a lower bound for the price of a 1-month European put option on a nondividend-paying stock when the stock price is 12, the strike price is 15, and the continuously compounded risk-free interest rate is 6%? 25. Consider a non-dividend-paying S. Draw the graph for the price of a European 1-year 5strike call option as a function of the current stock price S0. Assume the continuously compounded risk-free interest rate is 10%. 26. You are given: (i) (ii) Stock S is going to pay a 2-dollar dividend 3 months from now. This is the only dividend to be paid in the coming 6 months. The continuously compounded risk-free interest rate is 10%.

Draw the graph for the price of a European 6-month 15-strike call option as a function of the current stock price S0. 27. (09 FIN3250 Midterm) The price of a non-dividend-paying stock is currently 60 and the effective annual interest rate is 5%. A 1-year 54-strike European put is worth 3.2, while a 2-year 56.7-strike European call is worth 11.5. Demonstrate an arbitrage opportunity. Section 2.4 Early Exercising 28. For a stock whose current price is 42, you are given that (i) (ii) The stock is going to pay a dividend of 1 after 3 months. This is the only dividend to be paid in the coming 9 months. A 6-month 42-strike European call option on the stock has a premium of 0.36.

The continuously compounded risk-free interest rate is 5%. Consider a 6-month 42-strike American put option on the stock. Is it optimal to exercise the American put option now?

FINA 3250 | Derivatives for Actuaries I

59

C.Y. Ng

Chapter 2. An Introduction to Options Exercise 2

29. (Hull 9.5) The early exercise of an American put is a trade-off between the time value of money and the insurance value of a put. Explain this statement. 30. For a stock, you are given that (i) (ii) The current stock price is 100. The stock is going to pay a 1.5 dollar dividend after 6 months. This is the only dividend to be paid in the coming 9 months.

The continuously compounded risk-free interest rate is 5%. Consider a 9-month K-strike American put option on the stock. Given only the information above, what is the range of values of K so that it is certain that it is not optimal to exercise the K-strike American put? 31. (DM 9.13) Suppose that r = 0 and the stock of XYZ pays known positive dividend. Is there any circumstance in which you would early-exercise an American XYZ call? Is there any circumstance in which you would early-exercise an American XYZ put? Explain. 32. (Hull 9.8) Explain why the arguments leading to put-call parity for European options cannot be used to give a similar result for American options. 33. (Hull 9.13) Give an intuitive explanation of why the early exercise of an American put becomes more attractive as the risk-free rate increases and volatility decreases. 34. An American cash-or-nothing call (also called a digital or binary call) can be exercised at any time t [0, T]. If exercised at time t its payoff is 1 dollar when St > K and is 0 otherwise. When should such an option be exercised? 35. Suppose you have an American call option that allows you to receive one share of Yammy Corporation by giving up one share of Czar Corporation. The share of Czar Corporation is expected to pay a dividend, but the share of Yammy Corporation will not pay a dividend. Would it be optimal to exercise this option early? Why or why not? 36.* Consider the case of continuous dividends for the unfinished proof of Appendix 2. (a) Show that at time T, (3) is either max(ST, K), if the option holder does not early exercise, or ST e(T t) Ker(T t) + K, if the option holder early exercise at t ( T). (b) Show that at time T, (4) (3) and hence show that p + S0 C + KerT. (c) Prove that p + S0 C + KerT also holds for stocks paying discrete dividends. [Hint: assuming that dividends are invested at risk-free rate.] Numerical Answers 1. 2. 3. (c) 98.04 (b) 49.027, 50 5.523 (c) 2%

FINA 3250 | Derivatives for Actuaries I

60

C.Y. Ng

Chapter 2. An Introduction to Options Exercise 2

4. 6. 7. 10. 11. 16. 17. 21. 22. 23. 24. 25.

0.94404 2.51 (c) r 0.0202 (K2 K1)erT (b) increase 0, 3.9926 2, 8/3 5.33, 6 2.4394 2.93 c(S0) S0 S0 4.524

S0 4.524 26. c(S0) S0 1.9506 S0 16.2191

1.9506 28. 30. 34. 35.

16.2191

S0

No 39.7485 The first time when St hits K No.

FINA 3250 | Derivatives for Actuaries I

61

C.Y. Ng

Chapter 3. Binomial Option Pricing Section 1. Binomial Tree

CHAPTER 3
Related Readings

Binomial Option Pricing

Derivatives Markets (2nd edition)


Chapter 10, Section 11.2, 11. 3 and 11.5, Appendix 11A and 11B

Options, Futures & Other Derivatives (7th edition)


Chapter 11, Sections 19.1 19.3

L earning Objectives Concept of replication and risk-neutral pricing, single and multi-period tree on stock, tree as an approximation of GBM, trees for stock indices, futures and foreign exchange, handling discrete dividends, state price vector

In this chapter we study option pricing theory in discrete time. The main ideas in this chapter are the construction of replicating portfolio, the concept of risk-neutral pricing, log-normality of stock price at maturity as a limit of binomial tree. These three ideas will be used again in the geometric Brownian motion (GBM) model for pricing options in continuous time. The binomial tree is an approximation of the GBM. It is a very powerful tool because it can be used to price American options and many exotic options for which analytical pricing formula in the GBM framework does not exist.

Binomial Tree

Consider a binomial model for a stock price which pays dividends at rate : at time 0, the price is S0 at time h, its price Sh can be equal to S0u or S0d (u > 1; d < 1) the continuously compounded risk-free interest rate is r at time h, the payoff of a derivative is Cu or Cd, depending on the state of the stock

FINA 3250 | Derivatives for Actuaries I

62

C.Y. Ng

Chapter 3. Binomial Option Pricing Section 1. Binomial Tree

S0u Cu S0 C0 S0d Cd
How can we find the price of the derivative C0? To price the derivative, we try to replicate the cash flow of the derivative by forming a replicating portfolio using stock and bond. (a) Invest number of shares in stock and B dollars in risk-free bond at time 0. (b) Reinvest all dividends of the stock by buying additional shares. This gives eh shares at h. (c) The replicating portfolio at time h is worth ehSh + Berh.

h C u C d = e S (u d ) e h S 0 u + Be rh = C u 0 h rh e S 0 d + Be = C d B = e rh uC d dC u ud The replicating portfolio at time 0 is worth S0 + B. If there are no arbitrage opportunities,


e ( r ) h d u e ( r ) h C 0 = S 0 + B = e rh Cu + Cd . ud ud

(3.1)

Example

1. 1

Prove that for the binomial tree to make sense, we must have d < e(r )h < u.

FINA 3250 | Derivatives for Actuaries I

63

C.Y. Ng

Chapter 3. Binomial Option Pricing Section 1. Binomial Tree

Observations
(1) e ( r ) h d u e ( r ) h and are between 0 and 1 and they sum to 1, we let Since both ud ud e ( r ) h d p* = ud and write C0 = erh [p*Cu + (1 p*)Cd] or even C0 = erh E*[CT] (3.3)

(3.2)

where CT = Cu and Cd with probabilities p* and (1 p*). (2) Observe that the risk premium of the stock is nowhere to be found in the pricing formula. Moreover, E*[Sh] = p*S0u + (1 p*) S0d = Setting the probability of the up movement equal to p* is, therefore, equivalent to assuming that the return on the stock equals the risk-free rate. p* is called the risk-neutral probability and the probability measure constructed from p* is called the risk neutral world. Formula (3.3) is the risk neutral pricing formula. (3) (4) can be interpreted as the sensitivity of the option to a change in the stock price. If the derivative is a European call and S0d < K < S0u, then > 0 and B < 0. If the derivative is a European put, then < 0 and B > 0.

Example

1. 2

A stock price is currently 40. At the end of one month it will be either 42 or 38. The continuously compounded risk-free interest rate is 8%. What is the value of a one-month 39-strike European call? What would you do if the market price of the option is 1.5?

FINA 3250 | Derivatives for Actuaries I

64

C.Y. Ng

Chapter 3. Binomial Option Pricing Section 1. Binomial Tree

We can extend the analysis to a two-period binomial tree: S0u2 Cuu S0u Cu S0 C0 S0d Cd S0ud Cud S0d2 Cdd

The initial stock price is S0. During each time step, the stock price either moves up to u times its initial value or moves down to d times its initial value. The risk-free interest rate is r and the length of the time step is h years. e ( r ) h d . ud

The risk-neutral probability of going up is p* = Repeated applications of equation (3.1) gives

Cu = erh [p*Cuu + (1 p*)Cud] , Cd = erh [p*Cud + (1 p*)Cdd] , C0 = erh [p*Cu + (1 p*)Cd] . Substituting the first and the second equations into the third equation, we get C0 = e2rh [(p*)2Cuu + 2p*(1 p*)Cud + (1 p*)2Cdd] or C0 = e2rh E*[CT] where CT = Cuu, Cud, Cdd with probabilities (p*) , 2p*(1 p*) and (1 p*) , respectively. Notice that the three probabilities are the probabilities that the up, middle and lower final nodes will be reached in the risk-neutral world. The option price is again equal to its expected payoff in the risk-neutral world, discounted at the risk-free interest rate.
2 2

(3.4)

(3.5)

FINA 3250 | Derivatives for Actuaries I

65

C.Y. Ng

Chapter 3. Binomial Option Pricing Section 1. Binomial Tree

Example

1. 3

Consider a 2-year 52-strike European put option on a stock whose current price is $50. There are one time steps in one year and in each time step the stock price either moves up by a proportional amount of 20% or moves down by a proportional amount of 20%. The continuously compounded risk-free interest rate is 5%. Calculate the price of the put.

American options can be priced similarly. The value of an American option at the end nodes is the same as for its European counterpart. At earlier nodes the value of the option is the greater of 1. The value given by (3.1). 2. The payoff from early exercise.

Example

1. 4

Repeat Example 1.3, assuming the put is American.

FINA 3250 | Derivatives for Actuaries I

66

C.Y. Ng

Chapter 3. Binomial Option Pricing Section 1. Binomial Tree

Path-dependent option can also be priced under a binomial tree. Note that in this case we have to calculate the payoffs path by path and weigh them by the corresponding risk-neutral probabilities.

Example

1. 5

Consider a three-period stock price model with S0 = 4, u = 2, d = 0.5, = 0, h = 1 and an annual effective rate of interest of 25%: 32 16 8 4 2 1 0.5 A European lookback call pays ST min{Su: 0 u T} at maturity. The option gets this name because the option holder can lookback and choose the strike at his own advantage. Calculate the price of the three-year lookback call. Solution: There are 8 possible states, with associated payoff and risk-neutral probabilities w P*(w) Payoff UUU UUD UDU UDD DUU DUD DDU DDD 4 2 8

FINA 3250 | Derivatives for Actuaries I

67

C.Y. Ng

Chapter 3. Binomial Option Pricing Section 2. Log-normality

Log-normality

Previously, we assumed that S jumps up or down at times h, 2h, 3h, and that u and d are given. In reality, h should be extremely small; a volatility of the stock, instead of u and d, is given. The precise definition of is the following: The volatility of a stock is defined so that the variance of the continuously compounded return (not annualized) on the stock in a short period of time t is 2t. In the above, we have implicitly assumed that the volatility is constant through time. That is, if the variance of one-year return is 2, then the variance of two-year return is 2 2, etc. Now, for a tree with time step h, we determine u and d so that the volatility of the return of the tree matches the given. Consider Stu St Std t t+h 1 p* = u e ( r ) h ud p* = e ( r ) h d ud

Since the h-year simple return can only take values u 1 and d 1, when h is small (so that the difference between simple and continuously compounded return is negligible), (u d)2 p*q* 2h. There are many choices of u and d satisfying the above.
The Forward Tree

u = exp[(r )h + h ] d = exp[(r )h h ]

(3.6)

The idea underpinning the forward tree stems from the forward price of S. Example 1.1 in Section 3.1 states that d < e(r )h < u. Multiplying the whole inequality by St, St d < St e(r )h < St u or St d < Ft, t + h (S) < St u.

FINA 3250 | Derivatives for Actuaries I

68

C.Y. Ng

Chapter 3. Binomial Option Pricing Section 2. Log-normality

This means that u and d should be chosen so that u > apart from satisfying (u d) p*q* h.
2 2

Ft ,t + h ( S ) St

= e ( r ) h and d <

Ft ,t + h ( S ) St

= e ( r ) h ,

Let us see how we can construct an n-period tree over [0, T] with volatility . T (1) The time step of the tree is h = . n 1 (2) Calculate u and d using (3.6) and hence get p* = using (3.2). 1 + e h

(3)

Build the tree. There are n + 1 end nodes. The kth node (k = 0, 1, , n counting from the bottom) has value S0ukd n k , corresponding to a continuously compounded rate of return of

[(r )h + h ]k + [(r )h h ](n k ) = (r )hn + (2k n) h = (r )T + (2k n) h (4) (5) n The risk-neutral probability that the kth end node will be reached is ( p*) k (1 p*) n k . k Thus the continuously compounded rate of return can be thought of as a random variable R = R(0, T) = ln(S T / S 0 ) = (r )T + (2U n) h where U ~ B(n, p*) is the number of up moves. Note that p* depends on n. From the discussion above, we see that the binomial tree so constructed has R(0, T) distributed as a scaled binomial distribution. What happens to the distribution of R(0, T) when n (or equivalent, h 0)? To identify the limiting distribution of R(0, T), we calculate its MGF evaluated at x:
E[exp( xR)] = E[exp( x(r )T ) + x(2U n) h )] = exp[ x(r )T xn h ]E[exp(2 x hU )] = exp[ x(r )T xn h ][1 p * + p * exp(2 x h )]n

Putting the expression for p* into the above and taking limit, we obtain (see appendix for details) E[exp( xR)] exp[(r 0.5 2 )Tx + 0.5 2Tx 2 ] which is the MGF of N[(r 0.5 2 )T , 2 T ]. Recall from FIN2220 that if X follows a normal distribution, then Y = eX is said to follow a lognormal distribution. Thus
Limit of Binomial Tree

In the limit, R(0, T) is normally distributed, and since ST = S0eR(0, T), ST follows a lognormal distribution in the risk-neutral world.

FINA 3250 | Derivatives for Actuaries I

69

C.Y. Ng

Chapter 3. Binomial Option Pricing Section 2. Log-normality

Alternative Binomial Tree Models

Equation (3.6) is not the only way to select u and d. Here are two other methods: u = exp( h ) The Cox-Ross-Rubinstein binomial tree: d = exp( h ) u = exp[(r 0.5 2 )h + h ] The lognormal / Jarrow-Rudd binomial tree: d = exp[(r 0.5 2 )h h ] All three methods give the same ratio of u over d. The Cox-Ross-Rubinstein approach is most commonly used in practice due to its simplicity ud = 1 (but see Exercise 3.12). The lognormal tree is also called the equal-probability tree because the resulting p* is very close to 0.5. While the three different methods yield different option prices for finite n, they all converge to the same limit as n . The limiting value is the Black-Scholes price that we shall derive in Chapter 5. (See also Exercise 3.15.)

Example

2. 1

Consider a 1-year 40-strike American put option on a stock whose current price is 41. The volatility of the stock is 30% and the continuously compounded risk-free interest rate is 8%. Calculate the price of the put by constructing a three-period binomial tree using the forward tree approach.

FINA 3250 | Derivatives for Actuaries I

70

C.Y. Ng

Chapter 3. Binomial Option Pricing Section 3. Options on Other Assets

Options on Other Assets

Options on Foreign Currencies

A currency option is an option whose payoff depends on exchange rate (the value of one unit of the foreign currency measure in the domestic currency). Let xt be the spot exchange rate of HKD and Euro at time t (1 at time t = $xt). At time 0, the exchange rate is x0 At time h, the exchange rate xh can be equal to x0u or x0d (u > 1; d < 1) For a HK dollar-denominated derivative (that pays HKD at maturity), consider a replicating portfolio consisting of units of Euro, and B dollars invested in HK at risk-free rate at time 0, rf h Cu C d = e e x0 u + Be = C u x0 (u d ) rf h e x0 d + Be rh = C d B = e rh uC d dC u ud
rf h rh

So the exchange rate is like a stock paying a continuous dividend which equals the foreign riskfree interest rate rf. By setting equal to the constant annualized volatility of the exchange rate, We can price options on currencies much the same as stock options: just replace by rf.

?
(i) (ii)

Example

3. 1 [09 FIN3250 Midterm]

Suppose that the spot price of the Canadian dollar is $0.75 US; the Canadian-dollar US-dollar exchange rate has a volatility of 15%;

(iii) the continuously compounded risk-free interest rates of Canada and the United States are 9% and 8%, respectively. (a) Calculate the no-arbitrage price (in Canadian dollars) of a call option to buy 75 USD with 100 Canadian dollars six months from now. Use a one-period forward tree to model the movement of the exchange rate.

FINA 3250 | Derivatives for Actuaries I

71

C.Y. Ng

Chapter 3. Binomial Option Pricing Section 3. Options on Other Assets

(b) The market price of the call in (a) is 5 Canadian dollars. Suppose that the one-period forward tree model is correct. Demonstrate an arbitrage opportunity.

Options on Futures Contracts

Options on futures contracts deserve a special derivation because it requires no initial investment to enter into a futures contracts (if not taking margin requirement into account). At time 0, the futures price is F0 At time h, its price Fh can be equal to F0u or F0d (u > 1; d < 1) At time h, the payoff of a derivative is Cu or Cd, depending on the state of the futures

F0u Cu F0 C0 F0d Cd
How can we find C0? We again replicate the cash flow of the derivative: (a) Enter into number of futures contract and put B dollars in risk-free bond at time 0. (b) Recall that the payoff of a futures contract is the change in the futures prices. The replicating portfolio at time h is worth (Fh F0) + Berh. Cu C d = F (u d ) ( F0 u F0 ) + Be = C u 0 rh ( F0 d F0 ) + Be = C d B = e rh 1 d C + u 1 C u d ud u d
rh

The replicating portfolio at time 0 is worth B. If there are no arbitrage opportunities,

u 1 1 d C 0 = e rh Cu + Cd . ud u d

(3.7)

FINA 3250 | Derivatives for Actuaries I

72

C.Y. Ng

Chapter 3. Binomial Option Pricing Section 3. Options on Other Assets

Observations
(1) Since d < 1 < u (or else there is an arbitrage opportunity), both 0 and 1. Hence we let p* = and write C0 = erh [p*Cu + (1 p*)Cd] or even C0 = erh E*[CT] (3.9) 1 d ud 1 d u 1 and are between ud ud (3.8)

where CT = Cu and Cd with probabilities p* and (1 p*). (2) Observe that the risk premium of the stock is nowhere to be found in the pricing formula. Moreover, E*[Fh] = p*F0u + (1 p*) F0d = F0. Thus in the risk-neutral world, the expected growth rate of the futures price is zero. (3) Equations (3.8) and (3.9) are the same as (3.2) and (3.3) with = r. As a result, We can price futures options much the same as stock options: just replace by r.

Example

3. 2

An option has a gold futures contract as the underlying asset. The current 1-year gold futures price is $300/oz., the strike price is $290, the risk-free rate is 6%, volatility is 10%, and time to expiration is 1 year. Calculate the price of an American call on gold futures using a binomial tree with 3 time steps.

FINA 3250 | Derivatives for Actuaries I

73

C.Y. Ng

Chapter 3. Binomial Option Pricing Section 3. Options on Other Assets

Options on Stocks Paying Discrete Dividends (Out of MFE syllabus)

The Failure of Forward Tree Since most options expire in one year or less, the amount and timing of the dividends during the life of an option can be predicted with certainty. Now we consider how we can price options on a stock that pays known discrete dividends during the life of the option by building a tree. First we try to use the idea of forward tree. If no dividend will be paid between t and t + h, we create the binomial tree by (3.6). Suppose a dividend will be paid between t and t + h, and that its future value at time t + h is D. Then u= Ft ,t + h ( S ) St e
h

S t e rh D = e St

S t e rh D and d = e St

The problem is that the values of u and d depend on the current stock price. See how bad it would become when we build the tree using this idea.

Example

3. 3

Consider a stock whose current price is $41. The volatility of the stock is 30% per annum and the risk-free interest rate (compounded continuously) is 8% per annum. The stock pays a discrete dividend after 5 months, with future value at t = 8/12 of $5. Construct a binomial tree with 2 time steps, with each time step being 4 months long. Solution: During the first time step there are no dividends. So u = e rh + h = 1.221246 and d = 0.863692 and thus Su = 50.0711 and Sd = 35.4114 50.0711e 0.08 / 3 5 0.3 / 3 e = 1.102504, d = During the second time step, at the up node, u = 50.0711 0.779715, so that Suu = 55.2036 and Sud = 39.0412. At the down node, we have u = 1.053347, d = 0.744950, so that Sdu = 37.3005 and Sdd = 26.3797. 55.2036 50.0711 41 35.4114 26.3797 The tree does not recombine because Sud Sdu. While it is possible to price derivatives using this tree, the number of nodes is excessively large when h is small. Also, it can happen that S becomes negative at some point because there is no guarantee that St erh D > 0. 39.0412 37.3005

FINA 3250 | Derivatives for Actuaries I

74

C.Y. Ng

Chapter 3. Binomial Option Pricing Section 3. Options on Other Assets

The Escrowed Dividend Model We want to build a tree that is recombining. The key idea of the escrowed dividend model is that if we know for certain that a stock will pay a fixed dividend D at time TD, then we can view the stock price as being the sum of (1) the PV of the dividend (this is like a zero-coupon bond with no volatility) (2) the PV of the ex-dividend value of the stock, i.e., the prepaid forward price: St = Ft ,P ( S ) + D exp[r(TD t)] for t TD T forward component of the stock:
(3.10)

Since only Ft ,P ( S ) is random at time t, the volatility of the stock is attributed to the prepaid T

S0 F F0PT + 0 D exp(rTD), ,
yielding

S0 . S 0 D exp[ rTD ]

To build the binomial tree for the stock, we first build the binomial tree for the prepaid forward component using the methods introduced in the previous sections starting with
F0PT ( S ) = S0 D exp(rTD), ,

then add back the corresponding present value of the dividend using (3.10) to obtain the tree. The escrowed dividend model is only a method to approximate the price an option on a dividendpaying stock. In Section 8.4 we will illustrate the Roll, Geske and Whaley formula which was once thought to be an exact formula to this problem.

Example

3. 4

Consider a one-year American call option with a strike price of $40 on a stock whose current price is $41. The volatility of the stock is 30% per annum and the risk-free interest rate (compounded continuously) is 8% per annum. The stock pays a discrete dividend of $5 in 8 months. By constructing a binomial tree with 3 time steps, find the price of the call option. Solution: Step 1: PV of dividend at time 0 = 5 exp(0.08 Step 2: F0 = S0 PV of dividend at time 0 = 8 )= 12

F =
Step 3: u = exp(rh + F h ) =
d=

FINA 3250 | Derivatives for Actuaries I

75

C.Y. Ng

Chapter 3. Binomial Option Pricing Section 3. Options on Other Assets

Step 4: The binomial tree for the prepaid forward is

Step 5: Add back the PV of dividends to the tree on or before the dividend is paid

Step 6: Price the option:

Observations
(1) (2) Notice that if the option is European, then only the terminal value of the tree matters and hence we may not need to do Step 5. If we are pricing an American put, then Step 5 becomes Add back the PV of dividends to the tree before the dividend is paid. This is because for American call, it is only optimal to exercise immediately just before a dividend payment, but for American put, it is never optimal to exercise just before a dividend payment.

FINA 3250 | Derivatives for Actuaries I

76

C.Y. Ng

Chapter 3. Binomial Option Pricing Section 4. State Price Vector

State Price Vector

State price vector is another method to value options under binomial tree. We imagine a world where there is a risky stock S and a riskless bond B, with prices S0 and B0 at time 0. The model for the stock and the derivative in the single-period binomial tree is
Su Cu S0 C0 Sd Cd

Assume that we have two hypothetical assets: The first one pays $1 if at time h the state is up. The price of this security is Qu. The second one pays $1 if at time h the state is down. The price of this security is Qd. The two hypothetical assets are called Arrow-Debreu securities and (Qu, Qd) is called a state price vector. We can find the values of the state price vector from
1 Qu + 1 Qd = B0 h S u Qu + S d Qd = S 0 e 1 S u 1 Qu B0 = S d Qd S 0 e h S 0 e h B0 S d Qu 1 Q = S S h d u d B0 S u S 0 e (here Su Sd because the stock is risky). The price of the derivative is
C u Qu + C d Qd = = B S S 0 e h S 0 e h B 0 S d Cd Cu + 0 u Su S d Su S d

e h e rh d ue rh e h Cu + Cd ud ud e ( r ) h d u e ( r ) h = e rh Cu + Cd ud ud

which is equivalent to (3.1).

FINA 3250 | Derivatives for Actuaries I

77

C.Y. Ng

Chapter 3. Binomial Option Pricing Section 4. State Price Vector

Example

4. 1 [09 May MFE #7]

The following one-period binomial stock price model was used to calculate the price of a 1-year 10-strike call option on the stock.
Su = 12 S0 = 10 Sd = 8

You are given: (i) (ii) The period is one year. The true probability of an up-move is 0.75.

(iii) The stock pays no dividends. (iv) The price of the 1-year 10-strike call is $1.13. Upon review, the analyst realizes that there was an error in the tree construction and that Sd, the value of the stock on a down-move, should have been 6 rather than 8. The true probability of an up-move does not change in the new tree, and all other assumptions were correct. Recalculate the price of the call option. (A) 1.13 (B) 1.20 (C) 1.33 (D) 1.40 (E) 1.53 (Ans: D)

What is the relation between state price vector and risk-neutral probability? From risk-neutral valuation, Qu = erh[1 p* + 0 q*] and Qd = erh[0 p* + 1 q*], and B0 = erh, and hence
p* = Qu and Qu + Qd q* = Qd . Qu + Qd

The approach of state price vector and risk-neutral probability can be extended to models with many states.

FINA 3250 | Derivatives for Actuaries I

78

C.Y. Ng

Chapter 3. Binomial Option Pricing Section 4. State Price Vector

?
(i) (ii)

Example

4. 2 [MFE Sample #27]

You are given the following information about a securities market: There are two non-dividend-paying stocks, X and Y. The current prices for X and Y are both $100.

(iii) The continuously compounded risk-free interest rate is 10%. (iv) There are three possible outcomes for the prices of X and Y one year from now: Outcome 1 2 3
X $200 $50 $0 Y $0 $0 $300

Let CX be the price of a European call option on X, and PY be the price of a European put option on Y. Both options expire in one year and have a strike price of $95. Calculate PY CX. (A) $4.30 (B) $4.45 (C) $4.59 (D) $4.75 (E) $4.94 (Ans: A) Extra question 1: Can you find the risk-neutral probabilities for the three states of outcome? Extra question 2: What if X pays dividends continuously at a rate of 2% and Y pays a dividend of 4 after 6 months?

FINA 3250 | Derivatives for Actuaries I

79

C.Y. Ng

Chapter 3. Binomial Option Pricing Section 5. Pricing under P measure

Pricing under P measure

We have illustrated risk-neutral pricing in the previous sections of this chapter. In the hypothetical risk-neutral world (Q measure), we have E*(Sh) = p*S0u + (1 p*) S0d = S0 exp[(r )h]. Under the real world (P measure), we have E(Sh) = pS0u + (1 p) S0d = S0 exp[( )h]. Here p is the actual probability that the stock price would go up after a time period of h:
p= e ( ) h d ud

(3.11)

?
(i) (ii)

Example

5. 1 [07 May MFE #2]

For a one-period binomial model for the price of a stock, you are given: The period is one year. The stock pays no dividends.

(iii) u = 1.433, where u is one plus the rate of capital gain on the stock if the price goes up. (iv) d = 0.756, where d is one plus the rate of capital loss on the stock if the price goes down. (v) The continuously compounded annual expected return on the stock is 10%. (B) 0.57 (C) 0.62 (D) 0.67 (E) 0.72 (Ans: A) Calculate the true probability of the stock price going up. (A) 0.52

Can we use p to price the option? The answer is yes, but we have to use a risk-adjusted discount rate or weighted average cost of capital (WACC) . For example, to price the stock, we need to use as the risk-adjusted discounted rate: exp(h)E(Sh) = exp(h)[pS0u + (1 p) S0d] = exp(h)S0 = F0Ph ( S ) , But what is the appropriate for the option? Recall that in the derivation of equation (3.1), we replicate the options cash flow by purchasing shares of stock and investing B dollars in riskfree bonds. Thus satisfies
eh =
S0 B e h + e rh . S0 + B S0 + B

(3.12)

FINA 3250 | Derivatives for Actuaries I

80

C.Y. Ng

Chapter 3. Binomial Option Pricing Section 5. Pricing under P measure

Example

1. 2 (continue)

We have the following set-up: S0 = 40, Su = 42, Sd = 38, r = 8%, T = 1/12, K = 39. Recall that = 0.75 and B = 28.3106, and the following replicating portfolio:
0.75 shares Borrow 28.3106 Net Position Cost at time 0 0.75 40 = 30 28.3106 1.6894 Up at T 0.75 42 = 31.5 28.3106 erT = 28.5 3 Down at T 0.75 38 = 28.5 28.5 0

The value of the call was found to be $1.6894. (a) It is given that the true probability of up state is 0.6. Find the expected value of the spot price of S at T. Hence determine the expected return (continuously compounded) return on the stock. (b) What is the weighted average cost of capital (WACC) of the replicating portfolio? (c) Hence, price the call using (a) and (b). (d) Repeat by using P(up at T) = 0.8.

The observation of the above example is that depending on the value of P(up at T), the expected rate of return on S, the WACC of the replicating portfolio, and the risk-adjusted discounted rate of the call would change accordingly. But no matter what P(up at T) is, we always get the same call price. A mathematical proof is as follows:
exp( h)E(C h ) = exp( h)[ pC u + (1 p )C d ] = = S0 + B [ pC u + (1 p )C d ] S 0 e h + Be rh

e ( ) h d S0 + B u e ( ) h Cu + Cd C C d uC d dC u u d ud e ( ) h u + ud ud = S0 + B

FINA 3250 | Derivatives for Actuaries I

81

C.Y. Ng

Chapter 3. Binomial Option Pricing Section 5. Pricing under P measure

If we want = r, then
the expected return on the stock should be chosen to be r, thus by Observation (2) on p.64, P(up at T) should be set equal to p*.

While conceptually important, the idea of using P measure to price an option is not useful. In order to get the value of , we need to obtain and B. But if and B are available, the option price is simply S0 + B. When extended to multi-period tree, we have to calculate , B and at each node. (One such example is given on p.349-350 of Derivatives Markets, see also Exercise 3.19.)

is very hard to estimate statistically (see Section 5.4 for more on this), and there is no reason to estimate it because its effect will be exactly offset by the corresponding .
Relation between p and p*

Comparing (3.2) and (3.11), we can see that p and p* are calculated by the same type of formula. The only difference is the expected return of the stock is selected to be the actual and riskneutral r. Is there an explicit relation between p and p*? Actually they have a nice relation in terms of utility. We let Uu (Ud) be the utility value, expressed in terms of dollars today, that an investor attach to 1 dollar received in the up (down) state after 1 period. Because investors are risk-averse, Uu < Ud. A result from portfolio theory (you need not know the details) states that when every investor chooses their portfolio optimally, Uu and Ud would be the same across investors, and
Qu = pU u . Qd = (1 p )U d

By combining the above with the relation between state prices and risk-neutral probabilities, we have pU u . p* = pU u + (1 p )U d This equation shows the relation between p and p*: p* is the utility-weighted p.

Example

5. 2

Consider a one-period model for which Su = 180 and Sd = 30, with p = 0.52. The utility values of 1 today for the up and down states are 0.87 and 0.98, respectively. (a) Find the state price vector. Hence, find B0 and the effective interest rate for one period. (b) Find S0. (c) Find the price of a 130-strike call that matures after one period.

FINA 3250 | Derivatives for Actuaries I

82

C.Y. Ng

Chapter 3. Binomial Option Pricing Section 5. Pricing under P measure

Appendix 3: Proof of the Convergence of Binomial Tree

We need to calculate the limit of exp[ x(r )T xn h ][1 p * + p * exp(2 x h )]n as h 0. T 1 Since p* = and h = , the above equals h n 1+ e
xT e h lim exp[ x(r )T ] h 0 h 1 + e + e
2 x h h

1 + e
2T
(ln y ) 2

h .

Using the substitution y = e

, then the above becomes


x T y + y ] ln y 1 + y
2 2x

e x ( r )T lim exp[
y 1

We consider the log of the limit: x 2T 2T y + y 2 x + ln lim y 1 ln y (ln y ) 2 1 + y 1 y + y 2 x = 2T lim ln (1 + y ) y x y 1 (ln y ) 2 1 y 1 x + y x = T lim ln 1+ y y 1 (ln y ) 2
2

0 ( ) 0

By lHpitals rule, the above equals 1 (1 x)( y x y x ) + x( y x 1 y 1 x ) 2T lim y 1 2(ln y )( y 1 ) (1 + y )( y 1 x + y x )

= 2T lim

y (1 x)( y x y x ) + x( y x 1 y 1 x ) lim y 1 2(1 + y )( y 1 x + y x ) y 1 ln y

0 ( ) 0

(1 x)( xy x 1 xy x 1 ) + x[( x 1) y x 2 (1 x) y x ] 1 = 2T lim 8 y 1 y 1 1 = 2T 4 x( x 1) 8 So the original limit is e x ( r )T e 0.5


2

Tx ( x 1)

= exp[(r 0.5 2 ) xT + 0.5 x 2 2T ] .

FINA 3250 | Derivatives for Actuaries I

83

C.Y. Ng

Chapter 3. Binomial Option Pricing Exercise 3

Exercise 3

Unless otherwise stated, do NOT use the Option Pricing spreadsheet. Section 3.1 Binomial Tree 1. (Hull 11.4) A stock price is currently 50. At the end of 6 months it will be either 45 or 55. The continuously compounded risk-free interest rate is 10%. The stock is non-dividendpaying. Find the price of a 6-month 50-strike European put option. (DM 10.4) Consider a one-period binomial model for a non-dividend-paying stock. Assume S0 = 100, r = 8%, and T = 0.5. Let u = 1.3 and d = 0.8. (a) Suppose that you observe a 95-strike put price of 8. Construct an arbitrage opportunity. (b) Suppose that you observe a 95-strike put price of 6. Construct an arbitrage opportunity. 3. (Hull 11.14) A stock price is currently 25. It is known that at the end of 2 months it will be either 23 or 27. The continuously compounded risk-free interest rate is 10%. Find the price of a derivative that pays off the square of the stock price 2 months from now. A stock is currently priced at 50. Over each of the next two 3-month periods it is expected to go up by 6% or down by 5%. The continuously compounded risk-free interest rate is 5%. Find the price of the value of a 6-month 51-strike European call (a) if the stock pays no dividends; (b) if the stock pays dividends continuously at a rate of 1.5%; (c) if the stock pays dividends continuously at a rate of 5%. What pattern do you observe? 5. Consider the setting in the previous question. By using put-call parity, obtain the prices of the corresponding put, and observe what happen to the put price as dividend yield increases. If the put options were American, would it ever be optimal to exercise it early at any of the nodes on the tree? Consider the stock price tree in Example 1.5. A European lookback put pays max{Su: 0 u T} ST at maturity. Calculate the price of the lookback put. 7. Consider a two-period binomial tree for a non-dividend-paying stock. You are given: (i) (ii) The current stock price is 10. The time step over each period is 6 months.

2.

4.

6.

FINA 3250 | Derivatives for Actuaries I

84

C.Y. Ng

Chapter 3. Binomial Option Pricing Exercise 3

(iii) u / d = 1.3125. (iv) The risk-neutral probability of an up move is 0.84. (v) The continuously compounded risk-free interest rate is 2%. Calculate the price of a 1-year 9-strike American put option. 8. Consider a binomial tree stock price model. You are given: (i) (ii) The current stock price is 10. The time step over each period is 6 months.

(iii) u = 1.3 and d = 0.9. (iv) The stock pays no dividends. (v) The continuously compounded risk-free interest rate is 5%. The price of a 1-year K-strike American call option is 0.8876. Find K. Section 3.2 Log-normality 9. (DM 10.10) Consider a non-dividend-paying stock with S0 = 100 and = 30%. Let r = 8%. You are to construct binomial tree using the forward tree method. (a) Verify that the price of a 1-year 95-strike American call under a three-period binomial tree is 18.283, and that early exercise is not optimal at each node. (b) By using the same binomial tree, find the price of a 1-year 95-strike European put, and verify that the put-call parity for European options is satisfied. (c) By using the same binomial tree, find the price of a 1-year 95-strike American put. 10. (DM 10.11) Repeat the previous question by assuming that the stock pays continuous dividend with yield = 8%. Would the American option be early exercised? 11. For a two-period forward binomial tree model for a stock, you are given: (i) (ii) The current stock price is 10. The time step over each period is 6 months.

(iii) u / d = 1.55. (iv) The stock pays dividends continuously at a rate proportional to its price. The dividend yield is 2%. (v) The continuously compounded risk-free interest rate is 6%. Let p (P) be prices of a 1-year 10-strike European (American) put option. Calculate P p. 12. (a) Show that in all three types of tree introduced, we have (u d)2 p*q* 2h. (b) Show in the unusual situation when the number of time steps n is so small that < r h that the Cox-Ross-Rubinstein binomial tree gives negative probabilities.

FINA 3250 | Derivatives for Actuaries I

85

C.Y. Ng

Chapter 3. Binomial Option Pricing Exercise 3

Thus to use the Cox-Ross-Rubinstein tree we must make sure that the number of periods in the tree is large enough so that h is small. Questions 13 to 15 involve the use of the Option Pricing spreadsheet. 13. (DM 11.12, 11.13) Let S0 = 100, = 30%, r = 8%, T = 1, and = 0. Let h = T / n. (a) What is the risk-neutral probability that S1 = S0uidn i for i = 0, 1, 2, , n? (b) Using Excel, plot the risk-neutral probability mass function of S1 for n = 3 and n = 10. (c) Assume that n = 50. What is the risk-neutral probability that S1 < 80? Hence, price a European cash-or-nothing put that pays one dollar if S1 < 80 at maturity using a 50period tree. (Remark: The Black-Scholes price is 0.179789.) 14. (DM 11.16, 11.17) In this question we compare the option prices obtained from the CoxRoss-Rubinstein and Jarrow-Rudd lognormal tree. Let S0 = 100, = 0.30, r = 8%, T = 1, and = 5%. Let h = T / 8. By using Option Pricing, compute the prices of European and American 95-strike calls and puts using (a) the Cox-Ross-Rubinstein tree; (b) the Jarrow-Rudd lognormal tree. Are they close to the prices obtained from the forward tree? 15. Consider a binomial model with S0 = 41, = 35%, r = 7%, T = 0.5, and = 2%. By using Option Pricing, compute the price of a European put with K = 40 using h = T / n for n = 1, 10, 50, 100, 500, 1000 and 5000 using three different types of tree. Do they converge? Section 3.3 Options on Other Assets 16. Suppose that (i) (ii) the spot price of the Canadian dollar is $0.75 US; the Canadian-dollar US-dollar exchange rate has a volatility of 10%;

(iii) the continuously compounded risk-free interest rates of Canada and the United States are 8% and 10%, respectively. (a) Calculate the no-arbitrage price (in Canadian dollars) of a call option to buy 100 USD with 140 Canadian dollars six months from now. Use a one-period forward tree to model the movement of the exchange rate. (b) The market price of the call in (a) is 0.7 Canadian dollars. Suppose that the one-period forward tree model is correct. Demonstrate an arbitrage opportunity. (c) Calculate the no-arbitrage price (in US dollars) of a put option to sell 70 Canadian dollars for 50 USD six months from now. How is this related to (a)?

FINA 3250 | Derivatives for Actuaries I

86

C.Y. Ng

Chapter 3. Binomial Option Pricing Exercise 3

17. Company A is a U.S. international company, and Company B is a Japanese local company. Company A is negotiating with Company B to sell its operation in Tokyo to Company B. The deal will be settled in Japanese yen after 6 months. To avoid a loss at the time when the deal is closed due to a sudden devaluation of yen relative to dollar, Company A has decided to buy at-the-money dollar-denominated yen put of the European type to hedge this risk. You are given: (i) (ii) The sale price of the Tokyo operation has been settled at 120 billion Japanese yen. The continuously compounded risk-free interest in the US and Japan are 3.5% and 1.5%, respectively.

(iii) The current dollar-yen exchange is 120 yen per dollar. (iv) The exchange rate volatility is 5%. By constructing a 2-period forward tree for the exchange rate, calculate Company As option cost. 18. (DM 10.16) Suppose that the current exchange rate is 1 dollar for 120 yen. The dollar interest rate is 5% and the yen rate is 1%. Consider a 1-year at-the-money American dollar call that is yen-denominated. The exchange rate volatility is 10%. By constructing a 3period binomial tree, find the price of the call. 19. Consider a two-period binomial tree for a futures index. You are given: (i) (ii) The current futures price is 75. The time step over each period is 3 months.

(iii) u / d = 1.3. (iv) The risk-neutral probability of a down move is 0.62963. (v) The continuously compounded risk-free interest rate is 7%. Calculate the price of a 6-month 80-strike American call option. 20. (Hull 16.12) A futures price is currently 60. It is known that over each of the next two three-month periods it will either rise by 10% or fall by 10%. The risk-free interest rate is 8% per annum. (a) What is the value of a six-month European call option on the futures with a strike price of 60? What about a six-month 60-strike European put option? (b) If the call were American, would it ever be worth exercising it early? (c) Calculate the price of a six-month 60-strike American put. Would it be ever worth exercising early?

FINA 3250 | Derivatives for Actuaries I

87

C.Y. Ng

Chapter 3. Binomial Option Pricing Exercise 3

21. (07 FIN3250 Final) Gold futures option has a gold futures contract as the underlying asset. The current 1-year gold futures price is $300/oz. The continuously compounded risk-free interest rate is 6%. The volatility of futures price is = 12%. (a) A two-period binomial tree is used to replicate the cash flows of a European 305-strike put on gold futures using the underlying futures contract and cash invested at 6%. (i) (ii) The replicating portfolio is rebalanced every half year. and B are the number of shares of future contracts held and the amount of cash in the hedge portfolio: u Bu d Bd

0 B0

Calculate 0 d. (b) Calculate the price of an American 1-year 305-strike put on gold futures. 22. (DM 11.20) Let S0 = 50, = 30%, r = 8% and T = 1. The stock will pay a 4 dividend in exactly 3 months. Let h = T / 4. (a) Construct a 4-period binomial tree for the prepaid forward price. Remember to adjust . (b) Hence, calculate the price of a European 45-strike call. (c) Add back the present values of dividends to the tree. Hence, compute the price of an American 45-strike call. 23. Consider a 5-month 50-strike European put on a stock that is expected to pay a single dividend of 2.06 during the life of the option. It is given that S0 = 52, r = 10%, the volatility of the prepaid forward price is 40%, and the ex-dividend date is in 3.5 months. Use the Cox-Ross-Rubinstein method to construct a 5-period tree and calculate the price of the put option. Repeat for an otherwise identical American put. Section 3.4 State Price Vector 24. Consider a 1-period binomial tree model for a non-dividend-paying stock with u = 1.2 and d = 0.9. The time period is 1 year and the effective annual risk-free interest rate is 0.05. Calculate the absolute difference between state price of the up and down state. 25. Suppose S0 = 100, u = 1.2 and d = 0.8. The one-period effective annual risk-free rate is 10%. (a) Find the state price vector. (b) Price a 110-strike call using the state price vector.
88

FINA 3250 | Derivatives for Actuaries I

C.Y. Ng

Chapter 3. Binomial Option Pricing Exercise 3

26. Consider the following one-period binomial stock price model. 6.25 S0 You are given: (i) (ii) The period of the tree is T years. The price of a T-year 5-strike European put is 0.54784 3.75

(iii) The price of a T-year 25-strike European call on the square of the stock is 7.55205. Calculate the price of a T-year option that pays | ST 5 | at maturity. 27. Consider Example 1.5. Calculate the state price vector for the eight states UUU, UUD etc. 28. Show that the condition in Example 1.1 is the same as the state price vector being positive. 29. A one-period model has two assets with the following price evolution:
Asset 1 2 Price at t = 0 10.4 10 Up state 12 unknown Down state 8 2.5

A call option on asset 1 with a strike price of 9 has a price of 1.8. (a) Compute the state price vector implicit in this model. (b) Compute the implied effective interest rate for this model. (c) The price of a call option on asset 2 is 2.4. What is the strike price of this call? 30. Consider the following trinomial tree model for stock S: 17 S0 15 13 You are given that (i) (ii) The time period of the tree is 1 year. The continuously compounded risk-free interest is 4%.

(iii) A 1-year 15-strike call option on the stock has a price of 0.6. (iv) A 1-year 14-strike put option on the stock has a price of 0.5. (a) Calculate the risk-neutral probability for the stock price to become15 after 1 year. (b) If the stock pays dividends at rate of 2% continuously, calculate S0.

FINA 3250 | Derivatives for Actuaries I

89

C.Y. Ng

Chapter 3. Binomial Option Pricing Exercise 3

31. You are given the following information about a securities market: (i) (ii) There are two non-dividend-paying stocks, X and Y. The current prices for X and Y are 67 and 63, respectively.

(iii) The effective annual risk-free interest rate is 10%. There are three possible outcomes for the prices of X and Y one year from now:
Outcome 1 2 3 X 90 77 60 Y 33 66 99

(a) Compute the state price vector implicit in this model. (b) Compute the risk-neutral probabilities for the three outcomes. (c) Find the price of a one-year at-the-money call option on X. (d) Find the price of a spread option on X and Y that pays (X Y 4)+ one year from now. 32. You are given the following information about a securities market: (i) (ii) There are two non-dividend-paying stocks, X and Y. The current price for Y is 65.

(iii) The annual effective risk-free interest rate is 10%. There are three possible outcomes for the prices of X and Y one year from now:
Outcome 1 2 3 X 90 80 60 Y 33 66 99

(a) Find the range of possible values of the price of X so that the market has no arbitrage opportunities. [Hint: Let the price of X be x, and solve for the state price vector.] (b) Suppose that the price of X is 70. Construct an arbitrage portfolio. Section 3.5 Pricing under P Measure 33. For a one-period binomial model for a non-dividend-paying stock, you are given: (i) (ii) The period is 6 months. The following binomial tree: 54 50 48 (iii) E(S0.5) = 53

FINA 3250 | Derivatives for Actuaries I

90

C.Y. Ng

Chapter 3. Binomial Option Pricing Exercise 3

(iv) The continuously compounded risk-free rate of return of the stock is 4%. Find p p*. 34. Consider Example 1.3 and assume that = 5%. Find the risk-adjusted discounted rate for nodes 0, u and d and hence use the actual probability measure to price the put. Repeat for = 6%.

35. Consider the following one-period binomial stock price model 12 9 You are given: (i) (ii) The time period of the tree is 1. The stock pays no dividend. 8

(iii) The true probability of an up move is p = 0.75. (iv) The utility value of 1 today for the up state is 0.70. Find the utility value of 1 today for the down state. 36. Consider the following one-period binomial stock price model: 100 S0 60 You are given: (i) (ii) The time period of the tree is 1. The stock pays no dividend.

(iii) The price of a 1-year 85-strike call is 5.789. (iv) The utility value of 1 today for the up state is 0.70. (v) The utility value of 1 today for the down state is 1.1667. Find the ratio of the true and risk-neutral probability of an up move after 1 year. Numerical Answers: 1. 3. 4. 1.16 639.3 1.635071, 1.443013, 1.043823. Call price decreases as increases.

FINA 3250 | Derivatives for Actuaries I

91

C.Y. Ng

Chapter 3. Binomial Option Pricing Exercise 3

5. 6. 7. 8. 9. 10. 11. 13. 14. 15.

16. 17. 18. 19. 20. 21. 22. 23. 24. 25. 26. 27. 29. 30. 31. 32.

33. 35. 36.

For put, 1.375877, 1.557416, 2.019133. Put price increases as increases. Yes. E.g. when the stock pays no dividends, early exercise is optimal at node Sd and the put price is 1.645603. 1.376 0.23745 10.9 (c) 6.678 European call: 13.941, put: 9.326, American call: 14.183, put: 9.5046, both early exercise 0.02 (c) 0.2006, 0.1852 (a) European call: 15.13999, put: 7.713102, American call: 15.15327, put: 7.979101 (b) European call: 15.02898, put: 7.602094, American call: 15.04213, put: 7.877024 Forward tree: 15.2825, 7.855364, 15.29543, 8.113203 Forward tree: 3.892478, 3.060905, 3.006939, 2.985337, 2.995581, 2.994839, 2.994655 CRR tree: 3.941363, 2.996812, 3.010037, 3.004116, 2.994336, 2.995608, 2.994900 Jarrow-Rudd tree: 3.93872, 2.977087, 3.005296, 3.003018, 2.994606, 2.995042, 2.994824 All of them converge to a value that is close to 2.995. The limiting value is the Black-Scholes put price, which is 2.994708. (a) 0.778 (c) 0.29176, 0.778 = 0.29176 2 / 0.75 9.875 million 3.1257 3.00 (a) 3.0265 (b) 3.0265, no (c) 3.0265 for European put 3.0847 for American put, yes. (a) 0.4898 (b) 16.0484 (b) 8.4338 (c) 8.4551 (a) F = 32.55% 4.31932, 4.40971 0 (a) (30/44, 10/44) (b) 75/11 1.21913 The state prices are all 0.064. The state price vector is 0.064(1, 1, ..., 1). (a) (0.6, 0.4) (b) 0% (c) 11 (a) 0.16735 (b) 14.295 (a) (0.36364, 0.09091, 0.45455) (b) (0.4, 0.1, 0.5) (c) 9.27273 (d) 19.90909 (a) The state price vector is (230/33 x/10, x/5 435/33, 235/33 x/10). 2175/33 < x < 2300/33 (65.9091 < x < 69.697 if you want 4 decimal places.) (b) For example, short 914 units of X, 555 units of Y to earn $100,055, and invest the proceedings at the risk-free rate. After 1 year, the net worth is 9485.5, 310.5 or 275.5 after closing out the short position. 0.33 1.35 1.3

FINA 3250 | Derivatives for Actuaries I

92

C.Y. Ng

Chapter 4. Stock Price Process Section 1. Brownian Motion

CHAPTER 4
Related Readings

Stock Price Process

Derivatives Markets (2nd edition)


Sections 11.3, 18.1 18.4, 20.1 20.3, 20.6, Appendix 13A

Introduction to Probability Models (8th edition)


Sections 4.5.1, 10.1 10.3, 10.5 10.6

Stochastic Calculus for Finance II Continuous-Time Models


Sections 3.1 3.4

L earning Objectives Random walk, Brownian motion, stochastic calculus, Its lemma, the BlackScholes framework, the lognormal distribution

In this chapter, we study the most important component of the classical continuous-time model of stock price: the Brownian motion (), and the calculus that operates on it. This chapter is extremely mathematical and requires a solid knowledge in FIN2220. Let T be a set. For example, T = [0, ). A stochastic process () {X(t): t T } is a collection of random variables: for each fixed t T, X(t) is a random variable. The index set T is often interpreted as time. A stochastic process is discrete if T is countable, say, {0, 1, 2, }. It is continuous if T can fill in an interval on the real line. 1 Brownian Motion

We want to model the wandering of a drunken man who walks along a straight line. Zn = the position of the man on the line at time n, with Z0 = z0 being non-random. At each time point, the man may walk one step forward or backward.

To model {Zn}, let

Yi =
be the increments and let Z n = z 0 + Yi .
i =1 n

+1 with probability p 1 with probability q

FINA 3250 | Derivatives for Actuaries I

93

C.Y. Ng

Chapter 4. Stock Price Process Section 1. Brownian Motion

The stochastic process so constructed is called a random walk. After n steps, there can be k up moves and (n k) down moves, for k = 0, 1, 2, ..., n, and hence P(Zn = z0 + 2k n) = nCk pkqnk. When p = q = 0.5, the random walk is symmetric. The following figure shows a sample path of a symmetric random walk {Zn} for n = 0 to 100, using z0 = 25.
45 40 35 30 25 20 15 10 5 0 0 50 100 150 200 250 300 350 400 450 500

Example

1. 1 (The Gamblers Ruin Problem)

Let N N be fixed and Pi (i = 0, 1, 2, , N) be the probability that starting with z0 = i, {Zn: n 0} reaches N before 0. (a) Show that Pi = pPi+1 + qPi1 for i = 1, 2, , N 1. (b) Solve the difference equation for the case when p = q = 0.5.

FINA 3250 | Derivatives for Actuaries I

94

C.Y. Ng

Chapter 4. Stock Price Process Section 1. Brownian Motion

Let A, B N be fixed. For a symmetric random walk starting from z0 = 0, it follows from Example 1.1(b) that B P(hits A before B) = . A+ B Can we model stock price using a symmetric random walk? (a) Stock price cannot be negative because shareholders have limited liability. (b) The magnitude of the move should depend on the level of the stock price. We may expect the magnitude of the next jump to be 1 when the stock price is now $25. But when the stock price has descended to $10, the magnitude of the next jump should be less than 1. (c) The stock should have a positive mean return, but a random walk is not because EZn = 0. To build a better stock price model, we first build a continuous-time analogy of the random walk. From Symmetric Random Walk to Standard Brownian Motion: Rescaling First we let the drunken man move more frequently: Original: At t = 1, 2, 3, , the drunken man takes a step of size 1 or 1, with probabilities 0.5 New: At t = h, 2h, 3h, , the drunken man takes a step of size y or y, with probabilities 0.5 Let Z(t) denote the position at time t and take that Z(0) = 0. Then

Z(t) = y (Y1 + Y2 + + Y[t/h])

(4.1)

where the Yis are defined as in the symmetric random walk. (Here, [t/h] is the greatest integer less than or equal to t/h.) By a simple calculation, E[Z(t)] = 0 and Var[Z(t)] = (y)2[t/h] . Then we take limit: (1) h 0 so that the drunken man walks continuously; (2) y 0 so that the path of the man becomes continuous. However, we must do it in a way such that the resulting {Z(t): t T } is non-trivial. What about y = h and then h 0? What about y = h1/3 and then h 0? From the above, we can see that if we take y = h and let h 0, then Var[Z(t)] t From (4.1), the central limit theorem and y 0, the following are reasonable:

FINA 3250 | Derivatives for Actuaries I

95

C.Y. Ng

Chapter 4. Stock Price Process Section 1. Brownian Motion

1. Z(t) ~ N(0, t ) for each fixed t. 2. {Z(t): t 0} is continuous in t. The process constructed is called a standard Brownian motion (BM). The figure below shows a sample path of {Z(t): t 0} from t = 0 to 100. The path is quite jagged because the size of a movement in Z in time h roughly equals h and h is much bigger than h for small h. Actually, the path is so irregular that it is almost everywhere non-differentiable ( ).
40

30

20

10

10
-10

20

30

40

50

60

70

80

90

100

-20

-30

Rescaling of random walk is only one of the many ways to construct a Brownian motion. Any stochastic process satisfying the four properties below is a standard Brownian motion: Properties of Standard Brownian Motion 1. Z(0) = 0. 2. Z(t) ~ N(0, t ). 3. {Z(t)} has independent increments, i.e., for all t1 < t2 < < tn,

Z(tn) Z(tn1), Z(tn1) Z(tn2), , Z(t2) Z(t1), Z(t1)


are independent. 4. {Z(t)} has stationary increments, i.e. the distribution of Z(t + s) Z(t) depends only on s. Properties 2 and 4 implies that Z(t + s) Z(t) ~ N(0, s). Another very useful fact is

FINA 3250 | Derivatives for Actuaries I

96

C.Y. Ng

Chapter 4. Stock Price Process Section 1. Brownian Motion

Predicting the Future

Z(t + s) | {Z(u): 0 u t} ~ N(Z(t), s)


The fact that E[Z(t + s) | {Z(u): 0 u t}] = Z(t) is called the martingale property.

Example

1. 2

Let {Z(t): t 0} be a standard Brownian motion. Find (a) Cov[Z(2), Z(4)]; (b) E[Z(2)Z(4)]; (c) Var[Z(2)Z(4)].

Example

1. 3

Find Cov[Z(s), Z(t)] for s t. Hence, find the variance-covariance matrix of (Z(t1), Z(t2), ..., Z(tn)).

FINA 3250 | Derivatives for Actuaries I

97

C.Y. Ng

Chapter 4. Stock Price Process Section 1. Brownian Motion

Mean and Covariance Function For a stochastic process {X(t): t 0}, we define the mean and covariance function by

M(t) = E[X(t)], C(s, t) = Cov[X(s), X(t)].


For example, a standard Brownian motion has (i) M(t) = E[Z(t)] = 0 for all t 0, and (ii) C(s, t) = Cov[Z(s), Z(t)] = s for 0 s t.

Example

1. 4 [Brownian Bridge]

Let X(t) = Z(t) tZ(1) for 0 t 1. Compute the mean and covariance function of X.

Suppose that 0 t1 < t2 < ... < tn. To obtain the joint pdf of (Z(t1), Z(t2), ..., Z(tn)), we use predicting the future recursively to obtain the Multidimensional Density of Standard Brownian Motion 1 z12 ( z 2 z1 ) 2 ( z n z n 1 ) 2 exp + +K+ t 2 t1 t n t n 1 2 t1 . f Z (t1 ),Z (t2 ),K, Z (tn ) ( z1 , z 2 , K, z n ) = n/2 1/ 2 (2 ) [t1 (t 2 t1 ) L (t n t n 1 )]

Proof:

FINA 3250 | Derivatives for Actuaries I

98

C.Y. Ng

Chapter 4. Stock Price Process Section 2. Variations on Brownian Motion

Example

1. 5 [Inferring the past]

Suppose that 0 t1 < t2. Find the conditional distribution of Z(t1) given Z(t2) = h.

Variations on Brownian Motion

Brownian Motion with Drift

We say that {X(t): t 0} is a Brownian motion with drift with drift coefficient and variance parameter 2 (or volatility ) if X(t) = t + Z(t). It is easy to see that (i) {X(t): t 0} has stationary and independent increments; (ii) X(t) ~ N( t, 2 t).

Example

2. 1

Find the joint density function of X(s) and X(t) for 0 < s < t.

FINA 3250 | Derivatives for Actuaries I

99

C.Y. Ng

Chapter 4. Stock Price Process Section 2. Variations on Brownian Motion

Example

2. 2 [02 Nov Exam M #08]

The value of currency in country M is currently the same as in country N (i.e., 1 unit in country M can be exchanged for 1 unit in country N). Let C(t) denote the difference between the currency values in country M and N at any point in time (i.e., 1 unit in country M will exchange for 1 + C(t) at time t). C(t) is modeled as a Brownian motion process with drift 0 and variance parameter 0.01. An investor in country M currently invests 1 in a risk free investment in country N that matures at 1.5 units in the currency of country N in 5 years. After the first year, 1 unit in country M is worth 1.05 in country N. Calculate the conditional probability after the first year that when the investment matures and the funds are exchanged back to country M, the investor will receive at least 1.5 in the currency of country M. (A) 0.3 (B) 0.4 (C) 0.5 (D) 0.6 (E) 0.7 (Ans: B)

Geometric Brownian Motion (GBM)

Let {X(t): t 0} be a Brownian motion with drift and Y(0) be a constant. Then Y(t) = Y(0)eX(t) = Y(0)exp{ t + Z(t)} is called a geometric Brownian motion. Now we study properties of {Y(t): t 0}. To calculate probabilities involving Y(t), we notice that by taking logarithm, ln Y(t) = ln Y(0) + X(t) and hence Y(t) is lognormal (see Section 4.6 for a detailed description) and ln Y(t) ~ N( ln Y(0) + t, 2 t ). Recall that for a normal U, then MU(h) = exp{E(U)h + Var(U)h2/2}. Since X(t) ~ N( t, 2 t ), E[ekX(t)] = exp{ t k + 2 t k2/2}.

FINA 3250 | Derivatives for Actuaries I

100

C.Y. Ng

Chapter 4. Stock Price Process Section 2. Variations on Brownian Motion

As a result,
The k-th Moment of a Geometric Brownian Motion

E[Y k(t)] = Y k(0)exp[(k + k 2 2 /2) t]. In particular, E[Y(t)] = Y(0)exp[( + 2/2)t] and E[Y 2(t)] = Y 2(0)exp[(2 + 2 2 )t] = E2[Y(t)] e t , from which the variance of Y(t) can be computed. Another very useful fact for GBM is that we have the following relation between Y(t) and Y(s) for s < t: Y(t) = Y(s)e (t s) + [Z(t) Z(s)]. Notice that Y(s) and Z(t) Z(s) are independent.
2

Example

2. 3 [09 FIN3250 Midterm]


S1 (t ) = 100 exp[0.035t + 0.3Z 1 (t )] .

Let {Z1(t): t 0} be a standard Brownian motion and S1 be a stock following

(a) Find P(98 S1(2) 103). (b) Calculate the mean and variance of S1(2). Suppose that S2 is a stock following S2(t) = 95exp[0.08t + 0.25Z2(t)] where {Z2(t): t 0} is a standard Brownian motion that has a correlation of 0.4 with {Z1(t): t 0}. (c) Find (i) P(S2(2) > 103 | S2(1) = 100); (ii) P(S2(1) > 100 | S2(2) = 103);

FINA 3250 | Derivatives for Actuaries I

101

C.Y. Ng

Chapter 4. Stock Price Process Section 2. Variations on Brownian Motion

Correlated Brownian Motions

The Multivariate Normal Distribution Let Z1, Z2, ..., Zn be a set of n independent N(0, 1) random variables. Define X 1 = 1 + a11 Z 1 + a12 Z 2 + ... + a1n Z n X 2 = 2 + a 21 Z 1 + a 22 Z 2 + ... + a 2 n Z n
M X i = i + ai1 Z 1 + ai 2 Z 2 + ... + ain Z n M X m = m + a m1 Z 1 + a m 2 Z 2 + ... + a mn Z n

or in matrix notation
X = + AZ
X1 X where X = 2 , Z = M X m Z1 Z 2, = M Z n 1 2 , and A = M m a11 a 21 M a m1 a12 a 22 M a2m
n

L a1n L a2n . O M L a mn

While you are not required to know the joint pdf of X, you should know the following:
2 (1) Marginal distribution: each Xi is N( i, i2 ) with i2 = aik . k =1

(2) Dependence structure: Cov(Xi, Xj) = aik a jk . As a result, the variance-covariance matrix
k =1

is AA. We also have ij = Corr(Xi, Xj) = We say that X follows multivariate normal, denoted by X ~ Nm(, ), where = AA. is symmetric and positive semi-definite (that is, ww 0 for all non-zero vectors w Rm). Now suppose that m = n and = 0. In this case, each Xi ~ N(0, i2 ), but the Xis are (generally) correlated. If all ij are 0, then the Xis are not only uncorrelated, but independent.
The Cholesky Decomposition

Usually we are interested in the reverse process of creating n correlated standard normals X from n independent standard normals Z. That is, given a variance-covariance matrix , we want a matrix A such that X = AZ would follow Nn(0, ). This is equivalent to finding A such that = AA.

FINA 3250 | Derivatives for Actuaries I

102

C.Y. Ng

Chapter 4. Stock Price Process Section 2. Variations on Brownian Motion

It turns out that as long as is positive semi-definite (as it must be), such A exists (though not necessarily unique). Actually, A can even be chosen to be lower-triangular. To find the elements in A, we note
2 (1) variance constraint: Var ( X i ) = aik = ii = i2 k =1 i

(2) covariance constraint: For i < j, Cov( X i , X j ) = aik a jk = ij


k =1

The set of equations can be solved by starting from the top of the triangle and proceed down the row in the order a11, a21, a22, a31, a32, . This decomposition is implemented in the Option Pricing spreadsheet by the user-defined function Cholesky.

Example

2. 4

0.09 0.072 0.045 Find the Cholesky decomposition of = 0.072 0.16 0.02. Hence, explain how you can 0.045 0.02 0.25 form three dependent normal variables with mean vector (0, 0.3, 0.3) and variance-covariance matrix .

FINA 3250 | Derivatives for Actuaries I

103

C.Y. Ng

Chapter 4. Stock Price Process Section 2. Variations on Brownian Motion

Correlated Brownian Motions

From n independent standard Brownian motions {Zi(t)} for i = 1, ..., n, we want to create n correlated standard Brownian motions {Xi(t)} with a given correlation structure , where
1 = 21 M n1

12 L 1n 1 L 2n n2
M O L M 1

That is, we want (i) each {Xi(t)} to be a standard Brownian motion, (ii) the correlation is Corr[Xi(t), Xj(t)] = ij for any t. Suppose we obtain from Cholesky decomposition an A such that = AA. Then we set
X(t) = AZ(t).

Since Z(t) ~ Nn(0, tI), where I is an n n identity matrix, it follows that (1) Xi(t) is normal with mean 0, and Xi(0) = 0. (2) the variance-covariance matrix of X(t) is Cov[X(t)] = t , so that
Var[Xi(t)] = t (and then it is easy to prove that they are standard Brownian motions); Corr[Xi(t), Xj(t)] = Cov[Xi(t), Xj(t)] / t = ij t / t = ij (correct correlation structure).

Example

2. 3 (continue)

Find (iii) P(S1(t) > S2(t)) in terms of t.

FINA 3250 | Derivatives for Actuaries I

104

C.Y. Ng

Chapter 4. Stock Price Process Section 3. Variation and Stochastic Integral

Variation and Stochastic Integral

Variation of Standard Brownian Motion (Extremely Hard)

Let T > 0. By a partition P of [0, T] we mean a collection of points {t0, t1, , tn} where 0 = t0 < t1 < t2 < < tn = T . Corresponding to a partition P of [0, T] we put || P || = max(t i t i 1 ) .
i

The k order variation of a stochastic process {X(t)} in [0, T] for a partition P of [0, T] is
VTk ( X , P ) = X (t i ) X (t i 1 ) .
k i k T

th

The k order variation of {X(t)} in [0, T] is V ( X ) = lim VTk ( X , P) .


P 0

th

Example

3. 1

Let X(t) = t2/2. (a) Find the total variation over [0, T]. (b) Find the quadratic variation over [0, T]. (c) Find the length of the curve of X over [0, T].

FINA 3250 | Derivatives for Actuaries I

105

C.Y. Ng

Chapter 4. Stock Price Process Section 3. Variation and Stochastic Integral

When {X(t)} is a stochastic process, generally speaking, its variation is a function of the sample path and is thus random. For a standard Brownian motion, two special cases deserve some attention: Total variation (k = 1): the total variation of a process is the sum of the absolute increments of the process. Quadratic variation (k = 2): the quadratic variation of a process is the sum of the squared increments of the process. Quadratic Variation: Step 1: For a partition P = {t0, t1, , tn}, we first compute the mean and variance of VT2 ( Z , P) = [ Z (t i ) Z (t i 1 )]2 .
i

By property 4, Z(ti) Z(ti1) ~ N(0, ti ti1). By using the MGF of normal distribution, E[Z(ti) Z(ti1)] = 0, Var[Z(ti) Z(ti1)] = ti ti1, and E[Z(ti) Z(ti1)]4 = 3(ti ti1)2. Thus E[VT2 ( Z , P)] = E[ Z (t i ) Z (t i 1 )]2 = Var[ Z (t i ) Z (t i 1 )] = (t i t i 1 ) = T ,
i i i

which does not depend on P. By property 3, the [Z(ti) Z(ti1)]s are independent of each other for different is. Thus, Var[VT2 ( Z , P)] = Var[ Z (t i ) Z (t i 1 )]2
i

= E[ Z (t i ) Z (t i 1 )]4 {E[ Z (t i ) Z (t i 1 )]2 }2


i i

= 3(t i t i 1 ) (t i t i 1 ) 2 = 2 (t i t i 1 ) 2
2

2 P (t i t i 1 )
i

=2PT Step 2: Now let ||P|| 0 and calculate the limits of the mean and variance in Step 1. For mean, E[VT2 ( Z )] = lim E[VT2 ( Z , P)] = T .
P 0

For variance, Var[VT2 ( Z )] 2 lim P T = 0 . But since the variance obviously is nonP 0

negative, it follows from squeeze theorem that Var[VT2 ( Z )] = 0. Step 3: This means that VT2 ( Z ) is not random but constant! (Why?) The value of this constant is equal to the mean of VT2 ( Z ), which is T. We write

VT2 ( Z ) = lim [ Z (t i ) Z (t i 1 )]2 = [dZ (t )]2 = T .


T P 0 i 0

The calculation of variations of Brownian motion on p.653 of Derivatives Markets is wrong.

FINA 3250 | Derivatives for Actuaries I

106

C.Y. Ng

Chapter 4. Stock Price Process Section 3. Variation and Stochastic Integral

Quadratic Variation

The quadratic variation of a standard Brownian motion on [0, T ] is non-random and is equal to T. In particular, [dZ(t)]2 = dt. Total Variation: Notice that VT2 ( Z , P ) = Z (t i ) Z (t i 1 )
i 0<i n 2

max Z (t i ) Z (t i 1 ) Z (t i ) Z (t i 1 )
i

= max Z (t i ) Z (t i 1 ) VT1 ( Z , P)
0< i n

Letting ||P|| 0, we have


VT2 ( Z ) lim max Z (t i ) Z (t i 1 ) VT1 ( Z ) .
P 0 0< i n

The LH above equals T, while the limit in RH is 0 by continuity of standard Brownian motion. For RH to be greater than LH, The total variation of a standard Brownian motion is infinite. This result implies that the length of any path of a Brownian motion is infinite over any finite interval. This in turn implies the infinite crossing property: the expected number of times that the Brownian motion will cross any particular level in a time interval of any positive length is infinite (proof omitted). Higher-order Variation: By amending the proof above, it can easily be shown that the kth order variation is 0 for k > 2.

Stochastic Integral

For a real-valued function f, we define the Riemann integral

b a

f (t )dt as the limit

b a

f (t )dt = lim

P 0

f ( )(t
i i

t i 1 )

where P = {t0, t1, , tn} is a partition of [a, b] and ti1 i ti. If the limit does not exist, the integral is not defined.

FINA 3250 | Derivatives for Actuaries I

107

C.Y. Ng

Chapter 4. Stock Price Process Section 3. Variation and Stochastic Integral

Similarly, the Riemann-Stieltjes integral

b a

f (t )dg(t ) (where g is real-valued) is defined as

b a

f (t )dg(t ) = lim

P 0

f ( )[g(t ) g(t
i i i b

i 1

)] ,

whenever the limit exists. In the special case when g is differentiable, then the Riemann-Stieltjes integral reduces to the Riemann integral

b a

f (t )dg(t ) = f (t )g (t )dt.
a

When we replace g(t) by a stochastic process X(t), or f (t) by a function depending on t and X(t), we obtain integrals such as

b a

f (t , X (t ))dt and

b a

f (t , X (t ))dX (t ) .

The definition of the integrals above is similar to the Riemann-Stieltjes case, except that we always take i = ti1 (the left end-point). Notice that the integrals above depend on {X(t): 0 t T} and hence they are random variables in general. In the case when X(t) = Z(t),

b a

f (t , Z (t ))dZ (t ) is called an It integral.

Example

3. 2

Let f (t, x) = 1 for 0 t 1 and 2 for 1 t 2. Calculate and variance.

2 0

f (t , Z (t ))dZ (t ) and compute its mean

Mean and Variance of It Integral


b b b E f (t , Z (t ))dZ (t ) = 0 and Var f (t , Z (t ))dZ (t ) = E f 2 (t , Z (t ))dt a a a

The second formula is called It isometry. The proof of the above is given in Appendix 4A.

FINA 3250 | Derivatives for Actuaries I

108

C.Y. Ng

Chapter 4. Stock Price Process Section 4. Stochastic Differential Equation

Heuristic proof: ? Example (a)

3. 3
(b)

Compute the mean and variance of

2 0

t 2 dZ (t ) ;

1 0

Z 2 (t )dZ (t ) ;

[Hint: Recall from FIN2220 that for U ~ N(0, 1), EU 2 k =


Hence, show that

( 2k ) ! and EU 2k+1 = 0 for k N.] k 2 k!

1 0

Z 2 (t )dZ (t )

1 3 Z (1) . [See Exercise 4.30 for what the result should be.] 3

Stochastic Differential Equation

dy = a(t , y ) . From a modeling point of view, the rate of change of y dt respect to t is a deterministic function a(t, y). Another way to write the ODE is in differential form Consider the 1st-order ODE dy = a(t, y)dt, which can be interpreted as

y(ti+1) y(ti) a(ti, y(ti)) (ti+1 ti).


Not all models and systems are completely deterministic. What if the rate of change of y is perturbed by a noise? In a vague sense,

dy = a (t , y )dt + b(t , y ) noise. An It process / diffusion is a stochastic process that satisfies a stochastic differential equation (SDE, ) (4.2) Here, a(x, t) and b(x, t) are respectively the expected drift rate (or simply drift) and volatility coefficient of the SDE. b2(x, t) is the variance rate of the SDE. dX(t) = a(t, X(t))dt + b(t, X(t))dZ(t).

FINA 3250 | Derivatives for Actuaries I

109

C.Y. Ng

Chapter 4. Stock Price Process Section 4. Stochastic Differential Equation

For a differentiable function f : R R, df (t) can be treated as f (t + dt) f (t) = f (t) dt. But since {Z(t): t 0} is not differentiable, we interpret dZ(t) as Z (t + dt) Z (t) ~ N(0, dt), a normal random variable independent of the history {Z(u): 0 u t}. If a(x, t) 0, then {X(t)} is driftless and is a martingale (see Exercise 4.32 for more detail). To understand (4.2), consider a time grid P = {s0, s1, s2, , sn} with s0 = 0 and sn = t: X(si+1) X(si) a(si, X(si)) (si+1 si) + b(si, X(si)) [Z(si+1) Z(si)] Summing i from 0 to n 1 on both sides,

[ X (si +1 ) X (si )] a(si , X (si ))(si +1 si ) + b(si , X (si ))[Z (si+1 ) Z (si )].
i =0 i =0 i =0

n 1

n 1

n 1

Taking limit on n so that || P || = max( s i s i 1 ) 0,


i

Stochastic Integral Representation


X (t ) X (0) = a ( s, X ( s ))ds + b( s, X ( s ))dZ ( s )
0 0 t t

(4.3)

Observations
It follows from the fundamental theorem of calculus that d t a ( s, X ( s ))ds = a (t , X (t )) . So dt 0
t d a( s, X ( s ))ds = a(t , X (t ))dt . 0

We let the drift term a(s, X(s)) in (4.2) and (4.3) be 0. It then follows from dX(t) = b(t, X(t))dZ(t) X (t ) X (0) = b( s, X ( s ))dZ ( s )
0 t

that
Differential of an It Integral
t d b( s, X ( s ))dZ ( s ) = b(t , X (t ))dZ (t ) . 0

In a vague sense, the fundamental theorem of calculus holds for stochastic integrals. Given (4.2), we may be able to solve for X(t). A solution of (4.2) is a function dependent on X(0) and {Z(t)} so that we can calculate, for each t, the value of X(t) using X(0) and {Z(s): 0 s t}. For Exam MFE, you need not know how to solve SDEs. Instead, you should memorize the solution of three SDEs given below.

FINA 3250 | Derivatives for Actuaries I

110

C.Y. Ng

Chapter 4. Stock Price Process Section 4. Stochastic Differential Equation

Brownian motion with drift / Arithmetic Brownian motion (a(t, x) = , b(t, x) = )

The SDE dX(t) = dt + dZ(t) has stochastic integral representation


X (t ) X (0) = ds + dZ ( s ) .
0 0 t t

The solution of the SDE is the Brownian motion with drift (including an intercept) X(t) = X(0) + t + Z(t).
Ornstein-Uhlenback process (a(t, x) = ( x), b(t, x) = )

The Ornstein-Uhlenback (OU) process has SDE dX(t) = [ X(t)] dt + dZ(t) where > 0. The drift term for the SDE incorporates mean reversion. is the speed of the reversion, and is the equilibrium level: E[dX(t) | X(t)] = [ X(t)] dt when X(t) < , E[dX(t) | X(t)] > 0, and hence on average X(t) drifts up. when X(t) = , E[dX(t) | X(t)] = 0, and hence on average X(t) does not have drift. when X(t) > , E[dX(t) | X(t)] < 0, and hence on average X(t) drifts down. The solution of the SDE is
X (t ) = e t X (0) + (1 e t ) + e ( t s ) dZ ( s ) .
0 t

Like a BM with drift, the OU process can be negative. While not a useful model for stock prices, it illustrates non-deterministic drift and is a good model for interest rates.
Geometric Brownian motion GBM (a(t, x) = x, b(t, x) = x)

The SDE dX(t) = X(t) dt + X(t) dZ(t) has solution in the form of a geometric Brownian motion: X(t) = X(0) exp[( 2/2) t + Z(t)]. GBM can be used to model a non-dividend-paying stock. Let S(t) be the time-t price and note dS (t ) that is the percentage change or instantaneous return of the stock. We consider S (t ) dS (t ) = dt + dZ(t). S (t )

FINA 3250 | Derivatives for Actuaries I

111

C.Y. Ng

Chapter 4. Stock Price Process Section 4. Stochastic Differential Equation

This says that the instantaneous return over (t, t + dt) is normally distributed with mean dt and variance 2 dt. The solution of the SDE is For a non-dividend-paying stock, S(t) = S(0) exp[( 2/2) t + Z(t)] where is the expected rate of return and is the volatility per annum. If the stock pays proportional dividends continuously at rate , then the instantaneous return on dS (t ) the stock is reduced by dt. As a result, S (t ) dS (t ) = ( ) dt + dZ(t) S (t ) and hence
The Black-Scholes Model

For a stock that pays dividends proportional to its price, with a dividend yield , dS(t) = ( ) S(t) dt + S(t) dZ(t) and S(t) = S(0) exp[( 2/2) t + Z(t)]. (4.4)

In the above, is the expected total rate of return and is the expected rate of appreciation.

Example

4. 1

Let 0 t1 t2. Find E[S(t2) | S(t1)] and E[S 2(t2) | S(t1)] for the model above.

The Black-Scholes model is the model that we shall study in depth in Chapters 5 and, 6. We shall extend this model to accommodate for multiple correlated underlying assets in Chapter 7.

FINA 3250 | Derivatives for Actuaries I

112

C.Y. Ng

Chapter 4. Stock Price Process Section 4. Stochastic Differential Equation

Example

4. 2 [MFE Sample #10]

Consider the Black-Scholes framework. Let S(t) be the stock price at time t, t 0. Define X(t) = ln[S(t)]. You are given the following three statements concerning {X(t)}. (i) (ii) {X(t), t 0} is an arithmetic Brownian motion. Var[X(t + h) X(t)] = 2h, t 0, h > 0.
n n j =1

(iii) lim [ X ( jT / n) X (( j 1)T / n)]2 = 2T . Which of the statements is / are correct? (Do (iii) after you have learnt Its lemma.)

Example

4. 3 [MFE Sample #11]

Consider the Black-Scholes framework. You are given the following three statements on variances, conditional on knowing S(t), the stock price at time t. (i) Var[ln S(t + h) | S(t)] = 2 h, h > 0 dS (t ) (ii) Var S (t ) = 2 dt S (t )

(iii) Var[S(t + dt) | S(t)] = S 2(t) 2dt

Which of the statements is / are correct?

FINA 3250 | Derivatives for Actuaries I

113

C.Y. Ng

Chapter 4. Stock Price Process Section 5. Its Lemma

Its Lemma

Its lemma is a generalization of Taylors theorem in ordinary calculus. Let f be a function of t and x. Suppose that the value of f (t, x) is known and that t and x have both changed by small amounts dt and dx. Then Taylors theorem says that
f (t + dt , x + dx) f (t , x) + f t (t , x)dt + f x (t , x)dx + 1 [f tt (t , x)(dt ) 2 + 2f tx (t , x)dtdx + f xx (t , x)(dx) 2 ]. 2!

But since typically (dt)2, (dx)2 and (dt)(dx) are so small when compared with dt and dx, f (t + dt , x + dx) f (t , x) + f t (t , x)dt + f x (t , x)dx , or in differential form, df (t , x) = f t (t , x)dt + f x (t , x)dx . But what if we replace x by a diffusion process X(t)?
From Taylors Theorem to Its Lemma

More specifically, for dX(t) = a(t, X(t))dt + b(t, X(t))dZ(t), [dX(t)]2 is the sum of a(t, X(t))dt a(t, X(t))dt b(t, X(t))dZ(t) a2(t, X(t)) dt dt a(t, X(t)) b(t, X(t)) dt dZ(t) b(t, X(t))dZ(t) a(t, X(t)) b(t, X(t)) dt dZ(t) b2(t, X(t)) dZ(t) dZ(t)

Moreover, it follows from Exercise 4.19 that dZ(t) dt = 0. Gathering all these, we have [dX(t)]2 = b2(t, X(t))dt. As a result, while (dt)2 is still very small when compared to dt, [dX(t)]2 is comparable to dt! This important observation gives rise to
The 1-dimensional Its Lemma

Let f : [0, ) R R be twice continuously differentiable and Y(t) = f (t, X(t)). Then 1 dY(t) = f t (t, X(t))dt + f x (t, X(t))dX(t) + f xx (t , X (t ))[dX (t )]2 , 2 2 where [dX(t)] = dX(t) dX(t) is computed according to the rules dt dt = 0, dt dZ(t) = 0, dZ(t) dZ(t) = dt.

This result was first formulated and proven by Kiyoshi It () in 1942. The simplest case for the use of Its lemma is when Y(t) = f(t, Z(t)). In this case [dX(t)]2 = dt.

FINA 3250 | Derivatives for Actuaries I

114

C.Y. Ng

Chapter 4. Stock Price Process Section 5. Its Lemma

Example

5. 1
(ii) Y(t) = 2 + t + eZ(t); (iii) Y(t) = t sin[tZ(t)].

(a) Find dY(t) for each of the following: (i) Y(t) = Z2(t);
t

(b) Let Y(t) = sZ ( s )dZ ( s ) . Find dY(t).


0

Example

5. 2 [MFE Sample #13]

Let {Z(t): t 0} be a standard Brownian motion. You are given (i) U(t) = 2Z(t) 2 (ii) V(t) = Z(t)2 t (iii) W(t) = t2 Z(t) 2 sZ ( s )ds
0
t

Which of the processes above has / have zero drift?

FINA 3250 | Derivatives for Actuaries I

115

C.Y. Ng

Chapter 4. Stock Price Process Section 5. Its Lemma

Example

5. 3 [09 May MFE #6]


dX(t) = 2[4 X(t)]dt + 8dZ(t),

X(t) is an Ornstein-Uhlenbeck process defined by where Z(t) is a standard Brownian motion. Let Y(t) = 1/X(t). You are given that dY(t) = (Y(t))dt + (Y(t))dZ(t) for some functions (y) and (y). Determine (0.5). (A) 9 (B) 1 (C) 4 (D) 7 (E) 63 (Ans: D)

Example

5. 4

(a) Suppose {X(t): t 0} follows dX(t) = X(t) dt + X(t) dZ(t), X(0) = x0. Let Y(t) = ln X(t). Find dY(t) and hence solve for X.

Three Equivalent Representations of Geometric Brownian Motion

dX(t) = X(t) dt + X(t) dZ(t)

d[ln X (t )] = (

2
2

)dt + dZ (t )

X(t) = X(0) exp[( 2/2) t + Z(t)]

FINA 3250 | Derivatives for Actuaries I

116

C.Y. Ng

Chapter 4. Stock Price Process Section 5. Its Lemma

(b) Suppose {X(t): t 0} follows dX(t) = [ X(t)] dt + dZ(t), X(0) = x0. Let Y(t) = e t X(t). Find dY(t) and hence solve for X.

Example
t

5. 5

Let Y(t) = tZ ( s )dZ ( s ) . Find dY(t). Compare this with Example 5.1(b).
0

FINA 3250 | Derivatives for Actuaries I

117

C.Y. Ng

Chapter 4. Stock Price Process Section 5. Its Lemma

The Multivariate Its Lemma

More generally, let f (t, x1, x2, ..., xn) = f (t, x) be twice continuously differentiable on [0, ) Rn and Y(t) = f (t, X1(t), X2(t), ..., Xn(t)) = f (t, X(t)), where {X(t): t 0} is a correlated It process constructed from correlated Brownian motions {Z(t): t 0} with correlation matrix [ij].
f x1 (t , x) dX 1 (t ) f (t , x) x2 , H(f ) = [f (t , x)] , and dX(t ) = dX 1 (t ) . Then Define f x (t , x) = xi x j M M f xm (t , x) dX n (t )

The Multivariate Its Lemma

dY (t ) = f t (t , X(t ))dt + f xi (t , X(t ))dX i (t ) +


i

1 f x x (t , X(t ))dX i (t )dX j (t ) 2 i, j i j

1 = f t (t , X(t ))dt + f x (t , X(t )) dX(t ) + [dX(t )]H (f )dX(t ) 2 where dXi(t) dXj(t) is computed according to the rules dt dt = 0, Proof: dt dZi(t) = 0, dZi(t) dZj(t) = ij dt.

H is the Hessian of f with the first row and first column (which corresponds to the variable t) deleted.

FINA 3250 | Derivatives for Actuaries I

118

C.Y. Ng

Chapter 4. Stock Price Process Section 6. The Black-Scholes Framework

?
Let

Example

5. 6
dS(t) = (r S ) S(t) dt + S S(t)dZS(t), dQ(t) = (r Q ) Q(t) dt + Q Q(t) dZQ(t)

where {ZS(t): t 0} and {ZQ(t): t 0} are standard Brownian motions with correlation . Find dY(t) for (i) Y(t) = ln[S(t)/Q(t)] and (ii) Y(t) = S(t)Q2(t).

The Black-Scholes Framework

By means of the Black-Scholes framework, we mean the following: the underlying asset follows (4.4) the underlying asset pays dividends continuously at a level proportional to its price the risk-free interest rate is constant there are no transaction cost and taxes borrowing and short selling of any amount of cash and underlying asset is allowed the borrowing and lending rates are both equal to the risk-free interest rate there are no arbitrage opportunities trading is continuous

When the underlying asset S follows (4.4), S(T) is lognormal. So we start with the property of a lognormal distribution.

FINA 3250 | Derivatives for Actuaries I

119

C.Y. Ng

Chapter 4. Stock Price Process Section 6. The Black-Scholes Framework

The Lognormal Distribution

A random variable X is said to follow the lognormal distribution LN(m, s2) if ln X ~ N(m, s2). To find the cdf and pdf of X, we note ln x m FX (x) = P(X x) = P(ln X ln x) = N s z2 d 1 ln x m 1 FX ( x) = N , where N ( z ) = f X (x) = exp . 2 dx sx s 2 Recall that for Y ~ N(m, s2), MY(t) = E(etY ) = exp(mt + s 2t 2/2 ). Putting Y = ln X and t = k, For X ~ LN(m, s2), the kth raw moment of X is EX k = exp(mk + s 2k2/2). Letting k = 1 and 2, EX = exp(m + s 2/2) Thus, and EX 2 = exp(2m + 2s 2) = (EX)2exp(s2).

Var(X) = (EX)2 [exp(s 2 ) 1].

The following, whose proof can be found in Appendix 4B, is one of the keys in Chapter 5.
Partial Expectations of X

Let X ~ LN(m, s2) and v =

ln[E ( X ) / K ] + s 2 / 2 . Then s E[X I(X < K)] = E(X) N(v).

E[X I(X > K)] = E(X) N(v) and


The Behavior of S(t)

Now we apply the above to derive the properties of S(t) = S(0) exp[( 2/2) t + Z(t)]. Firstly, since Z(t) ~ N(0, t), and the 100p-th percentile of Z is
Percentiles of S(t)

t N1(p), and hence

1 Median of S(t) = S (0) exp ( 2 t 2 1 100p-th percentile of S(t) = S (0) exp 2 t + t N 1 ( p) 2 In particular, the 100(1 )% lognormal confidence interval for S(t) is given by S (0) exp[( 2 / 2)t z / 2 t ] .
Actually it is a log-transformed probability interval because S(t) is truly random. For MFE, just stick to this wrong terminology.

FINA 3250 | Derivatives for Actuaries I

120

C.Y. Ng

Chapter 4. Stock Price Process Section 6. The Black-Scholes Framework

Example

6. 1

Suppose that the stock price today is 100, the expected rate of return on the stock is 10%, the volatility of the stock is 30%, and the stock does not pay dividends. Find the 90% logtransformed probability interval of S(0.5).

Now it is about time to define the actual version of d1 and d2:


2 2 = ln[ S (0) / K ] + ( + / 2)t , d = d t = ln[ S (0) / K ] + ( / 2)t d1 2 1 t t

Then
The Cumulative Distribution and Moments of S(t)

P(S(t) K) = N (d 2 ) = 1 N (d 2 ) E[S k (t)] = S k (0)exp[k( )t + k(k 1) 2t/2] In particular, E[S(t)] = S(0)exp[( )t] and Var[S(t)] = E2[S(t)] [exp( 2t) 1].

Example

6. 2
(ii) E[S(t)I(S(t) > K)] = E[S(t)] N (d1 ) .

Show the decomposition (i) E[S(t)I(S(t) < K)] = E[S(t)] N ( d1 ) , Hence, evaluate E[S(t) | S(t) < K] and E[S(t) | S(t) > K].

FINA 3250 | Derivatives for Actuaries I

121

C.Y. Ng

Chapter 4. Stock Price Process Section 6. The Black-Scholes Framework

Example

6. 3

A European asset-or-nothing put can be exercised after 2 years, and the payoff is the stock price S(2) if it is less than 46. Suppose that the dynamics of {S(t): t 0} follows dS(t) = 0.08S(t)dt + 0.25S(t)dZ(t), S(0) = 50. (a) What is the probability that the payoff after 2 years is strictly positive, as viewed at t = 0? (b) Find the expected payoff of the option, as viewed at t = 0?

Example

6. 4 [MFE Sample #56]

Assume the Black-Scholes framework. For a stock that pays dividends continuously at a rate proportional to its price, you are given: (i) (ii) The current stock price is 5. The stocks volatility is 0.2.

(iii) The continuously compounded expected rate of stock-price appreciation is 5%. 1 Consider a 2-year arithmetic average strike option. The strike price is A(2) = [ S (1) + S (2)] . 2 Calculate Var[A(2)]. (A) 1.51 (B) 5.57 (C) 10.29 (D) 22.29 (E) 30.57 (Ans: A)

FINA 3250 | Derivatives for Actuaries I

122

C.Y. Ng

Chapter 4. Stock Price Process Section 6. The Black-Scholes Framework

The Behavior of Returns

Let R(t1, t2) = ln [S(t2) / S(t1)] be the (non-annualized) continuously compounded return of S from t1 to t2. (1) Continuously compounded returns are additive. Suppose t0 < t1 < t2. Then
R(t0, t2) = R(t0, t1) + R(t1, t2).

(2) Let {S(t): t 0} be a GBM in (4.4), then the mean and variance of the continuously compounded returns are proportional to time. In particular,
R(t1, t2) ~ N[( 2 /2) (t2 t1), 2 (t2 t1) ].

(4.5)

Statement (2) justifies the claim of the property of volatility in Section 3.2.
Proof: For (1), since S(t1) = S(t0) exp[R(t0, t1)] and S(t2) = S(t1) exp[R(t1, t2)], S(t2) = S(t0) exp[R(t0, t1)] exp[R(t1, t1)] = S(t0) exp[R(t0, t1) + R(t1, t2)] .

Comparing with S(t2) = S(t0) exp[R(t0, t2)], we have R(t0, t2) = R(t0, t1) + R(t1, t2) . For (2), observe that S(t2) / S(t1) = exp[( 2/2) (t2 t1) + (Z(t2) Z(t1))] and hence
R(t1, t2) = ( 2/2) (t2 t1) + [Z(t2) Z(t1)].

One consequence of (4.5) is that the 100(1 )% probability interval for R(0, t) is ( 2 / 2)t z / 2 t . Assume that = r in (4.5). By further setting t1 = 0 and t2 = T, we see that R(0, T) has the same distribution as the limit of the binomial tree in Section 3.2. Thus, the binomial tree is an approximation of the GBM. The reason why we need to change to r is that we need to change the probability measure from the actual to risk-neutral. This transformation will be studied in Section 5.3.
Appendix 4A: Mean and Variance of It Integral

By definition,

b a

f (t , Z (t ))dZ (t ) = lim f (t i 1 , Z (t i 1 ))[Z (t i ) Z (t i 1 )] . By the properties of BM,


P 0 i =1

Z(ti) Z(ti1) ~ N(0, ti ti1) and is independent of f (ti1, Z(ti1)). As a result,

E{f(ti1, Z(ti1))[Z(ti) Z(ti1)]} = 0. Similarly, for i j, using Cov(X, Y) = E(XY) EX EY, we have Cov{f (ti1, Z(ti1))[Z(ti) Z(ti1)], f (tj1, Z(tj1))[Z(tj) Z(tj1)]} = 0. Also, using Var(X) = E(X 2) (EX )2, we have Var{f(ti1, Z(ti1))[Z(ti) Z(ti1)]} = E[f 2(ti1, Z(ti1))](ti ti1). By assuming that the order of limit and the expectation sign can interchange, we get the results.

FINA 3250 | Derivatives for Actuaries I

123

C.Y. Ng

Chapter 4. Stock Price Process Exercise 4

Appendix 4B: Calculating E[X I(X > K)] when X is Lognormal

The density of X is f ( x) =
2

1 sx

1 2

exp(

(ln x m) 2 ) , for x > 0. 2s 2

Since E(X) = exp(m + s /2), we have m = ln E(X) s 2/2. So 1 (ln x m) 2 E[ XI( X > K )] = xf ( x)dx = exp( ) dx . K K 2s 2 s 2 ln x m ln K m Letting y = (so that x = e m + sy ) and L = , we have s s 1 (ln x m) 2 1 y2 exp( ) se m + sy dy exp( )dx = K s 2 L 2 2s 2 s 2
=

1 2
2

exp(

y 2 2sy + s 2 m + s 2 / 2 )e dy 2 exp(

( y s) 2 ) dy L 2 2 = E ( X )[1 N ( L s )] = E ( X ) N ( s L) = E( X ) N (v) = e m+ s
/2

Exercise 4

The level of difficulty of questions marked by (*) are beyond SoA. Section 4.1 Brownian Motion 1. Consider the random walk model Zn = Y1 + Y2 + + Yn where Yi = 1 or 1 with respective probabilities p and q = 1 p. (a) Find EZn and Var(Zn) in terms of p. (b) Find EZn and Var(Zn) for a symmetric random walk. 2. 3. Consider Example 1.1. Solve (a) for the case when p q. (a) What is the distribution of Z(t) Z(s) for s t? (b) What is the distribution of Z(t) + Z(s) for s t? 4. 5. 6. Calculate P(1 < Z(5) < 1 | Z(1) = 1). Let 0 < t1 < s < t2. Find the conditional distribution of Z(s) given Z(t1) = A and Z(t2) = B. Calculate Cov[Z(2) Z(1), Z(3)].
FINA 3250 | Derivatives for Actuaries I 124

C.Y. Ng

Chapter 4. Stock Price Process Exercise 4

7. 8.

Find E[Z(2)Z(3)] and E[Z(1)Z(2)Z(3)] . Let {Z1(t)} and {Z2(t)} be two independent standard Brownian motions and X(t) = 0.01 + 0.005t + 0.1Z1(t) 0.05Z2(2t) for all t 0. Find the distribution of X(t).

9.

Let {Z1(t)} and {Z2(t)} be two independent standard Brownian motions. Find P(Z1(5) > Z2(3)) and P(Z1(5) > Z2(3) | Z1(1) = 1).

Section 4.2 Variations on Brownian Motion 10. 11. Let X(t) be a Brownian motion with drift with = 0.05 and = 0.1. Find the probability that 0.5 < X(20) < 1, given that X(10) = 0.7. (a) Calculate the mean and variance of exp[3Z(0.1) + 2Z(0.2) + Z(0.3)], condition on the value of Z(0.1). (b) Hence, or otherwise, compute the mean and variance of exp[3Z(0.1) + 2Z(0.2) + Z(0.3)]. 12. Show that the joint MGF of the multivariate normal distribution is
n 1 n n MX(t1, t2, ..., tn) = exp i t i + t i t j Cov( X i , X j ) . 2 i =1 j =1 i =1

13.

Let {X(t): t 0} be a Brownian motion with drift coefficient and variance parameter 2, with X(0) = 0. Find E[ X(t) | X(s) = c] for (a) t > s, (b)* t < s.

14. (07 FIN3250 Final) Consider the following geometric random walk model for stock price: Sn = 2 exp{Y1 + Y2 + + Yn} where Yi = 1 or 1 with probabilities 0.3 and 0.7. Which of the following statements concerning the stock price process is / are correct? (i) (ii) E(Sn) does not increases with n. Var(S3) = 43.3 (corrected to 1 decimal place).

(iii) The expected annual rate of return (not continuously compounded) is 9%. 15. (a) Let c < 0 and Y(t) = Z(c2t) / c. Show that Y is a standard Brownian motion. t (b) Let {X(t): 0 t 1} be a Brownian bridge. Let Y (t ) = (1 + t ) X for t 0. 1+ t Compute the mean and covariance function of Y and compare them with that of a standard Brownian motion.

FINA 3250 | Derivatives for Actuaries I

125

C.Y. Ng

Chapter 4. Stock Price Process Exercise 4

Section 4.3 Variation and Stochastic Integral


t 1 0 Let Y (t ) = 2 1 < t < 3 . Find the total and quadratic variation of Y(t) over [0, 4]. 2 t 3

16.

17.

Consider {X(t)}, a Brownian motion with drift coefficient and variance parameter 2 . Show that the quadratic variation of X over [0, T] is 2T. [Hint: you can (i) use definition of quadratic variation, or (ii) integrate [dX(t)]2.]

18.

Show that for a correlated standard Brownian motion {Z1(t), Z2(t)} with correlation , lim
P 0

[Z (t ) Z (t
1
i

i 1

)][ Z 2 (t i ) Z 2 (t i 1 )] = [dZ 1 (t )dZ 2 (t )] = T .


0
i 1

19. (a) Prove that lim

P 0

[Z (t ) Z (t
i i

)](t i t i 1 ) = 0 .

Symbolically, we have (b) Prove also that lim


P 0

[dZ (t )dt ] = 0 for any T 0 and hence we write dZ(t) dt = 0.

(t
i

t i 1 ) 2 = 0, which gives dt dt = 0.

Section 4.4 Stochastic Differential Equation 20. Write down dY(t) for each of the following SDEs. Do NOT use Its lemma. (a) Y(t) = 12 2.5t + 5Z(t) (c) Y(t) = 2e0.5 + 0.3t 0.2Z(t)
t

(b) Y(t) = 12e2.5t + 5Z(t) (d) Y(t) = 20.5 + 0.3t 0.2Z(t)


t

(e) Y(t) = e t + 3(1 e t ) + 2 e s t dZ ( s ) (f) Y(t) = 2 e 2t + 2 e 2( t s ) dZ ( s )


0 0

21.

Write down the solutions of the following stochastic differential equations: (a) dY(t) = 10dt + dZ(t), Y(0) = 1 (c) dY(t) = aY(t)dt + bdZ(t), Y(0) = y0 (b) dY (t ) = 7dt dZ(t), Y(0) = 1 Y (t )

(d) dY(t) = [0.5 1.5Y(t)]dt + 3dZ(t), Y(0) = 0.4

22. 23.

(Hull 12.5) Consider the process dS(t) = (t)dt + (t)dZ(t) with S(0) = 5. For the first three years, (t) = 2 and (t) = 3; afterwards, (t) = 3 and (t) = 4. Find the distribution of S(6). Let S(t) be the stock price at time t in years, where dS(t) = 0.08S(t)dt + 0.3S(t)dZ(t), S(0) = 100. Calculate the probability (as viewed from time 0) that the payoff of a 100-strike 6-month European put option is greater than 1.5.
FINA 3250 | Derivatives for Actuaries I 126

C.Y. Ng

Chapter 4. Stock Price Process Exercise 4

24. (Hull 12.7) Stock A and B both follow geometric Brownian motion. Changes in any short interval of time are uncorrelated with each other. Does the value of a portfolio consisting of one of stock A and one of stock B follow geometric Brownian motion? [This observation will be useful in Chapter 7 when we discuss Asian options.] 25. (07 FIN3250 Final) Let S(t) = S(0)exp[( 0.5 2)t + Z(t)] and S(0) > 0. Let Y(t) = S1(t)ert for t 0. Which of the following statements is/are true? (i) (ii) {Y(t): t 0} is a geometric Brownian motion. dY (t ) Var Y (t ) = 2 dt Y (t )

dY (t ) dS (t ) (iii) Cov , S (t ) = 2 dt . Y (t ) S (t )

26.

Let 0 t0 t1 t2. Find Cov[S(t1), S(t2) | S(t0)] for S(t) following (4.4).

Section 4.5 Its lemma 27. Let Y(t) = t Z 3 (t ) . If dY(t) = a(t, Y(t))dt + b(t, Y(t))dZ(t), find a(t, y) and b(t, y). t . X (t ) If dY(t) = a(t, Y(t))dt + b(t, Y(t))dZ(t), find a(t, y) and b(t, y).
t t Let U(t) = t 2 Z ( s )dZ ( s ) and V(t) = t 2 Z ( s )dZ ( s ) . Find dU(t) and dV(t). 0 0
2

28. Suppose that dX(t) = 2e2tX(t)dt X(t)dZ(t) and Y(t) =

29. 30. 31.

(Relate to Example 3.3.) By considering d[Z3(t)], find (a) Show that X(t) = Z(t) / (1 + t) solves dX (t ) = (b) Show that X(t) = sin Z(t) solves dX (t ) =

1 0

Z 2 (t )dZ (t ).

X (t ) 1 dt + dZ (t ) , X(0) = 0. 1+ t 1+ t

1 X (t )dt + 1 X 2 (t )dZ (t ) , X(0) = 0. 2

32.* A stochastic process {Y(t): t 0} is a martingale if for any s < t, E[Y(t) |{Y(u): 0 u s}] = Y(s). (Strictly speaking, the definition on p.661 of Derivatives Markets is wrong.) (a) Let X(t) = Z2(t) t, show that {X(t)} is a martingale. (b) By using the stochastic integral representation, show that if the drift coefficient of the SDE followed by an It process {Y(t)} is 0, then {Y(t)} is a martingale. (c) Hence, use Its lemma to give a quick proof of (a).

FINA 3250 | Derivatives for Actuaries I

127

C.Y. Ng

Chapter 4. Stock Price Process Exercise 4

33. Find the value of c so that Y(t) = Z3(t) ctZ(t) is a driftless. 34.* By using multivariate Its lemma, prove the It product rule d[X(t)Y(t)] = X(t)dY(t) + Y(t)dX(t) + dX(t)dY(t). Hence, show the integration by parts formula

b a

X ( s )dY ( s ) = [ X (t )Y (t )]b Y ( s )dX ( s ) dX ( s )dY ( s ) . a


a a

What happens if X(s) is a deterministic function? 35.* Let a: R R R and b: R R be continuous functions. Let {X(t): t 0} follows dX(t) = a(t, X(t))dt + b(t)X(t)dZ(t), We illustrate a method to solve the SDE as follows: 1 t t (a) Define the integrating factor F (t ) = exp b( s )dZ ( s ) + b 2 ( s )ds . 2 0 0 Find dF(t) and hence show the SDE is equivalent to d[F(t)X(t)] = F(t) a(t, X(t))dt. (b) Now define Y(t) = F(t) X(t), so that X(t) = Y(t) / F(t). Show that dY(t) = F(t) a(t, Y(t) / F(t))dt, 1 dt + X(t)dZ(t), X(0) = x0 > 0. X (t ) Y(0) = x0. The equation enables us to find the solution of Y(t), from which we can obtain X(t). (c) Solve dX(t) = X(0) = x0.

36.* (09 FIN3250 Final) (a) Let {y(t): t 0} satisfy the Bernoulli differential equation dy (t ) = y (t )[h g (t ) y (t )], y(0) = y0, dt 1 . y (t ) (i) Show that u(t) satisfy a 1st-order linear ODE that can be solved using integrating factor. where h is a real constant and g is a function of t. Let u (t ) = (ii) Hence, express y(t) in terms of functions and integrals involving h, g, and y0. (b) Let {X(t): t 0} follow the stochastic differential equation dX(t) = rX(t)[K X(t)]dt + X(t) dZ(t), X(0) = x0 where r, K, are real constants, and {Z(t): t 0} is a standard Brownian motion. Let 1 Y (t ) = X (t ) exp[ 2 t Z (t )] . 2 By using Its lemma, show that (*)

FINA 3250 | Derivatives for Actuaries I

128

C.Y. Ng

Chapter 4. Stock Price Process Exercise 4

dY (t ) = Y (t )[rK g (t )Y (t )]dt for some function g that is a function of r, , t and the standard Brownian motion Z. (c) By using (a) and (b), show that the solution for (*) is
X (t ) = exp[(rK 0.5 2 )t + Z (t )] 1 / x0 + r exp[(rK 0.5 ) s + Z ( s )]ds
2 0 t

Section 4.6 The Black-Scholes Framework 37. (Hull 13.7) A stock price is currently 40. Assume that the expected return from the stock is 15% and that its volatility is 25%. What is the probability distribution for the rate of return (with continuous compounding) earned over a two-year period? Assume the Black-Scholes framework and that the stock does not pay dividends. (Hull 13.8) A stock price follows a geometric Brownian motion with an expected return of 16% and a volatility of 35%. The current price is 38. What is the probability that a European call option on the stock with an exercise price of 40 and a maturity date in six months will be exercised?

38.

39. Consider a non-dividend-paying stock with an initial price of 40, expected rate of return of 16% per annum, and a volatility of 20% per annum. Find the 95% probability interval of the stock return price in six months time and the 95% log-transformed probability interval of S(0.5). Find also the one standard deviation move in the stock price. Note 1: A one SD move here (p.598 and p.600 of Derivatives Markets) correspond to a movement of + and from the mean for the return. Note 2: We have still another definition of one SD move of the stock price in Section 6.2; see also p.429 of Derivatives Markets. These two definitions contradict with each other if T is not infinitesimally small. 40. Assume the Black-Scholes framework and a stock that follows the SDE dS(t) = 0.05S(t)dt 0.4S(t)dZ(t) Find the probability that S(1.5) is less than 0.6E[S(1.5)]. 41. Let S(t) follows (4.4) with S(0) = 10, = 0.02, = 0, and = 0.2. An option pays the geometric average 3 S (1) S (2) S (3) at time 3. Calculate the expected payoff.

Numerical Answers 1. 2. 3. (a) (2p 1)n and 4p(1 p)n 1 (q / p) i Pi = 1 (q / p) N (a) N(0, t s) (b) 0 and n

(b) N(0, 3s + t)

FINA 3250 | Derivatives for Actuaries I

129

C.Y. Ng

Chapter 4. Stock Price Process Exercise 4

4. 5. 6. 7. 8. 9. 10. 11. 13. 14. 16. 20.

0.3413 Normal with mean


A(t 2 s ) + B ( s t1 ) ( s t1 )(t 2 s ) and variance t 2 t1 t 2 t1

21.

22. 23. 24. 25. 26. 27. 28. 29. 30. 33. 34. 35. 37. 38. 39. 40. 41.

1 2, 0 N(0.01 + 0.005t, 0.015t) 0.5, N 1 (1 / 7 ) 0.2507 (a) e0.5 + 6Z(0.1), (e2 e)e12Z(0.1) (b) e2.3, e9.2 e4.6 (a) (t s) + c (b) ct/s (ii) only 6, 20 (a) dY(t) = 2.5dt + 5dZ(t) (b) dY(t) = 10Y(t)dt + 5Y(t)dZ(t) (c) dY(t) = 0.32Y(t)dt 0.2Y(t)dZ(t) (d) dY(t) = [0.3 ln 2 + 0.02(ln 2)2]Y(t)dt 0.2(ln 2)Y(t)dZ(t) (e) dY(t) = [3 Y(t)]dt + 2dZ(t) (f) dY(t) = 2[2 Y(t)]dt + dZ(t). (a) Y(t) = 1 10t + Z(t) (b) Y(t) = e6.5t Z(t) t t 1 (c) Y (t ) = y 0 e at + b e a (t s ) dZ ( s ) (d) Y (t ) = 0.4e 1.5t + (1 e 1.5t ) + 3 e 1.5( t s ) dZ ( s ) 0 0 3 N(20, 75) N(0.1537) No (i) and (ii) E[S(t2)|S(t0)] E[S(t1)|S(t0)] [exp( 2(t1 t0)) 1] [By using covariance decomposition and Example 4.1, this result is immediate.] y a(t, y) = + 3(ty )1 / 3 , b(t, y) = 3(ty 4 )1 / 6 2t 1 + t 2te 2t a(t , y ) = y , b(t, y) = y t 2U (t ) 2V (t ) 2 2 + t Z (t ) dt + 2t V (t ) Z (t )dZ (t ) dt + t 2 Z (t )dZ (t ) , dV(t) = dU(t) = t t 1 1 1 3 2 0 Z (t )dZ (t ) = 3 Z (1) 0 Z (t )dt 3 The final term is 0, and the ordinary integration by parts formula drops out.
2 (c) X (t ) = exp(Z (t ) 0.5 2 t ) x0 + 2 exp(2Z ( s ) + 2 s )ds 0 t

N(0.2375, 0.125) 0.4968 For return, (0.20719, 0.34719), for stock price, (32.515, 56.603); (37.243, 49.417) N(0.79777) 10.316

FINA 3250 | Derivatives for Actuaries I

130

C.Y. Ng

Chapter 5. The Black-Scholes Model Section 1. Change of Probability Measure

CHAPTER 5
Related Readings

The Black-Scholes Model

Derivatives Markets (2nd edition)


Sections 20.4 20.7, 12.1 12.2, 12.5, 21.2 21.3, 22.1 22.2, 18.5 18.6, 11.4

Options, Futures & Other Derivatives (7th edition)


Sections 13.1 13.4, 13.7, 13.8, 13.11, 15.1 15.4, 16.1 16.8, 18.5

Stochastic Calculus for Finance II Continuous-Time Models


Chapter 1

L earning Objectives Radon-Nikodm derivative, market price of risk, the Black-Scholes equation, risk-neutral valuation, the Black-Scholes formula and its variants, estimation of mean return and volatility

In this chapter we study derivative pricing under the Black-Scholes framework using the apparatus in Chapter 4. We shall show that the price of every European derivative must satisfy the Black-Scholes equation, from which we can derive the risk-neutral valuation formula for European derivatives. Then we shall focus on some special terminal payoff patterns and develop a variety of pricing formulas. 1

Change of Probability Measure

This section is a prelude to the theoretical part in Section 5.3. Consider a discrete sample space with finitely many sample points on which we have two probability measures defined on it. E.g. for the coin-tossing experiment, a coin is thrown three times. Let H = head and T = tail, then S = {HHH, HHT, HTH, HTT, THH, THT, TTH, TTT} Alfred thinks that the coin is biased, and he thinks that P(H) = 2/3. He defines the probability measure P accordingly. Andrew thinks that the coin is fair, and he defines the probability measure Q. P() Q() HHH 8/27 1/8 HHT 4/27 1/8 HTH 4/27 1/8 HTT 2/27 1/8 THH 4/27 1/8 THT 2/27 1/8 TTH 2/27 1/8 TTT 1/27 1/8

FINA 3250 | Derivatives for Actuaries I

131

C.Y. Ng

Chapter 5. The Black-Scholes Model Section 1. Change of Probability Measure

Any random variable defined on S may take different probabilities on P and Q. Let N() = number of heads. For example, N(HHH) = 3, N(TTH) = 1. Then P(N = 2) = 4/9 and Q(N = 2) = 3/8, and EP(N) = 2 and EQ(N) = 1.5. Q() . P() Observe that for all S, P() > 0 and Q() > 0. So we can define a random variable Z () =

Z is called the Radon-Nikodm derivative of Q with respect to P. We write Z =

dQ . dP

The random variable Z has the following three properties: (i) (ii) P(Z > 0) = 1; EP(Z) = 1;

(iii) EQ(W) = EP(ZW) for any random variable W defined on S. Proof:

FINA 3250 | Derivatives for Actuaries I

132

C.Y. Ng

Chapter 5. The Black-Scholes Model Section 1. Change of Probability Measure

Let A be an event. If we put W = I(A) in (iii), we have Q(A) = EP[ZI(A)] . Thus, we can recover the probabilities in measure Q using measure P. Conversely, for a probability measure P on S, if we can define a random variable Z with P(Z > 0) = 1 and EP(Z) = 1, then we can use Z as a Radon-Nikodm derivative and define a new probability measure Q on S by using P with Q(A) = EP[ZI(A)]. Proof:

Example

1. 1

Consider a sample space and let T be the time of death of a newborn. Under Alfreds probability measure, T follows a constant force of mortality of . Andrew wants to create a new probability measure Q using the Radon-Nikodm derivative Z=

( )T , > . e

(a) Check that Z can be used as a Radon-Nikodm derivative to induce a change of measure from P to Q. (b) Under Q, what is the distribution of T?

FINA 3250 | Derivatives for Actuaries I

133

C.Y. Ng

Chapter 5. The Black-Scholes Model Section 1. Change of Probability Measure

Example

1. 2

Consider a sample space and let R be the one-year return of a non-dividend-paying stock. Under measure P, R ~ N(0, 1). We define X = R + . 1 (a) We define the random variable Z = exp{R 2 } . Show that Z can be used as a Radon2 Nikodm derivative to induce a change of measure from P to Q.
(b) What are the distribution of X and R under Q?

In Section 5.3, we need to induce a change of measure based on a standard Brownian motion.
The Girsanov Theorem (Simplified version)

Let {ZP(t): 0 t T} be a standard Brownian motion under probability measure P, and construct a probability measure Q by using the Radon-Nikodm derivative
2T dQ . Z P (T ) = exp dP 2

Then under measure Q, {ZQ(t): 0 t T} defined by ZQ(t) = ZP(t) + t is a standard Brownian motion. Proof: (Out of MFE syllabus) It is obvious that

2T 2T > 0 and E P exp Z P (T ) = 1 . Z P (T ) exp 2 2

FINA 3250 | Derivatives for Actuaries I

134

C.Y. Ng

Chapter 5. The Black-Scholes Model Section 1. Change of Probability Measure

So we can define a probability measure Q by Q(A) = EP[exp{ZP(T) 2 T / 2}I(A)]. We need to show that ZQ possess the 4 properties of standard Brownian motion under Q. It is obvious that ZQ(0) = ZP(0) + 0 = 0. So property 1 holds. We shall tackle properties 2, 3, 4 by using joint moment generating function to show that for 0 s1 s2 t1 t2 T, the increments ZQ(s2) ZQ(s1) and ZQ(t2) ZQ(t1) follow N(0, s2 s1) and N(0, t2 t1), respectively, and that they are independent. The joint MGF of ZQ(s2) ZQ(s1) and ZQ(t2) ZQ(t1) under Q is
M Q (a, b) = E Q exp{a[ Z Q ( s 2 ) Z Q ( s1 )] + b[ Z Q (t 2 ) Z Q (t1 )]} dQ = E P exp{a[ Z Q ( s 2 ) Z Q ( s1 )] + b[ Z Q (t 2 ) Z Q (t1 )]} dP = E P [exp(Y )] where Y is defined as Y = a[ZQ(s2) ZQ(s1)] + b[ZQ(t2) ZQ(t1)] ZP(T) 2 T / 2 = a[ZP(s2) ZP(s1)] + b[ZP(t2) ZP(t1)] ZP(T) + a (s2 s1) + b (t2 t1) 2 T / 2 = [ZP(T) ZP(t2)] + (b )[ZP(t2) ZP(t1)] [ZP(t1) ZP(s2)] + (a )[ZP(s2) ZP(s1)] ZP(s1) + a (s2 s1) + b (t2 t1) 2 T / 2 and is (under P) normal with mean a (s2 s1) + b (t2 t1) 2 T / 2 and variance

2 (T t2) + (b )2(t2 t1) + 2(t1 s2) + (a )2(s2 s1) + 2 s1


= (b2 2b )(t2 t1) + (a2 2a )(s2 s1) + 2 T. This means that MQ(a, b) equals
1 expa ( s 2 s1 ) + b (t 2 t1 ) 2T / 2 + [(b 2 2b )(t 2 t1 ) + (a 2 2a )( s 2 s1 ) + 2T ] 2 a 2 ( s 2 s1 ) b 2 (t 2 t1 ) = exp exp 2 2

which is the product of the MGF of N(0, s2 s1) and N(0, t2 t1). In particular, by letting a = 0, t2 = t and t1 = 0, b 2t E Q exp{bZ Q (t )} = exp( ) 2 which gives property 2 explicitly.

FINA 3250 | Derivatives for Actuaries I

135

C.Y. Ng

Chapter 5. The Black-Scholes Model Section 2. Market Price of Risk

Market Price of Risk

Let {X(t): t 0} follow the SDE dX (t ) = m(X(t), t)dt + s(X(t), t)dZ(t). X (t ) (5.1)

X can be an investment asset, e.g. a bond, or a stock. It can also be something that is not directly tradable, e.g. exchange rate, interest rate or default intensity. It can even be something as far removed from financial markets as the temperature in IFC II in Central. Let V be the price of a derivative dependent only on X and t. Suppose that dV ( X (t ), t ) = mV ( X (t ), t )dt + sV ( X (t ), t )dZ (t ), V ( X (t ), t ) the dividend yield of V is V(X(t), t), and that the risk-free interest rate is r. Notice that here s and sV can be negative. This does not mean that the volatility of the return is negative, but that the random part of the return moves in a direction that is opposite to Z(t). The risk premium for a financial instrument is defined as the total return (capital gains + dividends) subtracted by the risk-free rate. The market price of risk of V is the risk premium per unit volatility (not variance!):
mV + V r . sV

When the denominator is replaced by | sV |, the quantity is called Sharpe ratio.

Example

2. 1

Let S follows (4.4). Let V(S(t), t) be the price of a derivative that pays S(T) at time T. (a) Find V(S(t), t). (b) Hence, find the market price of risk of a derivative that pays S(T) at time T.

Market Price of Risk Equation

The market price of risks of the price of two assets driven by the same Brownian motion must be the same. In particular, any derivative written on a stock following (4.4) have a market price of r risk of = .

FINA 3250 | Derivatives for Actuaries I

136

C.Y. Ng

Chapter 5. The Black-Scholes Model Section 2. Market Price of Risk

Proof: (1) Set-up: Two risky assets U and V, with price processes dU (t ) = mUdt + sUdZ(t), U (t ) dV (t ) = mVdt + sVdZ(t). V (t )

The continuous dividend yield rates of U and V are U are V, respectively. (2) Hedging: Suppose that we have 1 unit of V at time t. Our goal is to purchase / sell appropriate units of U and cash, so that we have an instantaneously risk-free and costless portfolio. Suppose we purchase N units of U and invest B dollars at risk-free rate of interest r. Then the value of the portfolio at time t is (t) = V(t) + NU(t) + B. To make the portfolio costless, B should be chosen so that B = V(t) NU(t). What would happen after an infinitesimally short period dt? The change in the prices of U and V are dU(t) and dV(t). The dollar amount of dividends of 1 unit of U and V are U(t)Udt and V(t)Vdt. The interest earning for 1 dollar is rdt. So,

Hedge Portfolio

The time-t hedge portfolio has N units of U and B dollars, where


N =

volatility coefficient of dV(t) sV V (t ) = , volatility coefficient of dU(t) sU U (t ) B = V(t) NU(t)

(5.2)

FINA 3250 | Derivatives for Actuaries I

137

C.Y. Ng

Chapter 5. The Black-Scholes Model Section 2. Market Price of Risk

(3) Equating the drift of the hedged portfolio to 0: By picking N and B as in (2), is instantaneously risk-free and costless. What should such a portfolio earn? If you have a risk-free investment of 0 dollars, for sure the return is 0 or else we have an arbitrage opportunity here! So, we set the drift of d to 0:

Example

2. 2 [MFE Sample #12]

Consider two non-dividend-paying assets X and Y. There is a single source of uncertainty which is captured by a standard Brownian motion {Z(t)}. The prices of the assets satisfy the stochastic differential equations dX (t ) dY (t ) = 0.07dt + 0.12dZ (t ) and = Adt + BdZ(t ), X (t ) Y (t ) where A and B are constants. You are also given that (i) (ii) d [ln Y(t)] = dt + 0.085dZ(t); The continuously compounded risk-free interest rate is 0.04. (B) 0.0613 (C) 0.0650 (D) 0.0700 (E) 0.0954 (Ans: B)

Determine A. (A) 0.0604

FINA 3250 | Derivatives for Actuaries I

138

C.Y. Ng

Chapter 5. The Black-Scholes Model Section 2. Market Price of Risk

Example

2. 3
dX (t ) dY (t ) = 0.06dt + 0.02dZ (t ) , = 0.09dt + kdZ (t ) . X (t ) Y (t )

The prices of two stocks are governed by:

where Z(t) is a standard Brownian motion and k is a constant. You are given: (i) (ii) The current stock prices are X(0) = 25 and Y(0) = 50. Both stocks pay dividends at a rate proportional to its price. The dividend yields of X and Y are + 0.01 and , respectively.

(iii) The continuously compounded risk-free interest rate is 4%. To construct a zero-investment, risk-free portfolio in which there is exactly 16 shares of X, one needs to trade a certain number of Y and borrow 100 dollars at a risk-free rate. Find .

FINA 3250 | Derivatives for Actuaries I

139

C.Y. Ng

Chapter 5. The Black-Scholes Model Section 3. The Black-Scholes Equation

The Black-Scholes Equation

Now we let V(t) = V(S(t), t) be the time-t price of a non-dividend-paying derivative written on S, and U(t) = S(t). So we have dV (t ) = mV dt + sVdZ(t), V (t ) dS (t ) = ( ) dt + dZ(t). S (t )

Since V is a derivative on S, mV and sV must be functions of S and t. To find mV and sV, we use Its lemma: 1 dV = Vt dt + Vs dS + Vss (dS ) 2 2 1 = Vt dt + Vs [( )S dt + S dZ] + Vss 2 S 2 dt . 2 Thus SV 1 dV 1 = Vt + ( ) SVs + 2 S 2Vss dt + s dZ . 2 V V V
So, we have (1) mV =

1 1 2 2 Vt + ( ) SVs + S Vss , 2 V SV (2) sV = s . V

The sign of sV is the same as Vs and indicates if V increases with Z. | sV | is known as the derivatives volatility. Now we apply the market price of risk equation to get
1 1 2 2 Vt + ( ) SVs + S Vss r r 2 V , = SVs V and hence V must satisfy the following parabolic PDE:

The Black-Scholes Equation

V V 1 2 2 2V = rV . + ( r ) S + S t s 2 s 2 In the above derivations we assume that the option is European. For American options,

(5.3)

The Black-Scholes equation also holds for American option as long as it is not optimal to exercise it.

FINA 3250 | Derivatives for Actuaries I

140

C.Y. Ng

Chapter 5. The Black-Scholes Model Section 3. The Black-Scholes Equation

The Black-Scholes equation has many solutions, corresponding to all European derivatives that can be defined with S as the underlying asset. We shall see shortly that depending on the particular boundary condition, the unique solution of (5.3) is the price of a particular derivative. For a forward contract with time to maturity T, V(s, T ) = For a European call expiring at time T, V(s, T ) = Before we solve the Black-Scholes equation, we illustrate how it can be used as a polygraph for checking if a certain formula is the price of a particular European derivative.

Example

3. 1

Let V(s, t) be the time-t price of zero-coupon bond maturing at T, when the stock price is s. (a) What is V(s, t)? (b) Find that Vt, Vs and Vss. (c) Show that V(s, t) satisfies the Black-Scholes equation. Repeat for the time-t prepaid forward price of a delivery of 1 share at T.

Example

3. 2

Let V(s, t) be the price of a derivative that pays S(T) at T if S(T) > K and nothing otherwise. It is ln(s / K ) + (r + 2 / 2)(T t ) (T t) proposed that V(s, t) = se N(d1) for 0 t T where d1 = . T t (a) Show that the pricing formula above satisfies the boundary condition. (b) Find Vt. (c) Find Vs and Vss and show that V(s, t) satisfies the Black-Scholes equation.

FINA 3250 | Derivatives for Actuaries I

141

C.Y. Ng

Chapter 5. The Black-Scholes Model Section 3. The Black-Scholes Equation

Solution: (a) First, rewrite d1 as d1 =

ln(s / K )

T t

r + 2 / 2

T t . We take t T.

For the case if s > K, d1 , N(d1) 1 and V(s, t) For the case if s < K, d1 The case s = K needs not be considered because it happens with probability 0. (b) From the form of d1 in (a), d1 ln(s / K ) r + 2 /2 (T t ) 3 / 2 (T t ) 1 / 2 = 2 2 t ln(s / K ) (r + 2 / 2)(T t ) = 2 (T t ) 3 / 2 ln(s / K ) + (r + 2 / 2)(T t ) 2(r + 2 / 2)(T t ) = 2 (T t ) 3 / 2 = so that Vt = d1 1 and N ( z ) = zN ( z ). = s s T t d1 T t 2(r + 2 / 2)(T t ) 0.5d1 (r + 2 / 2) T t / = T t 2 (T t ) 3 / 2

(c) Obviously,
V = s

FINA 3250 | Derivatives for Actuaries I

142

C.Y. Ng

Chapter 5. The Black-Scholes Model Section 3. The Black-Scholes Equation

Risk-Neutral Valuation

By substituting sV =

SVs into (5.2), we see that the time-t hedge portfolio is V

Hedging a Contingent Claim using the Underlying Asset

The time-t hedge portfolio has N shares of S and B dollars, where N = Vs(S(t), t), and B = V(S(t), t) NS(t). We shall later spend a whole chapter on hedging. In this section, we are interested in replicating a derivative position as in Section 3.1. Notice that the time-t derivative position can be replicated by N = Vs shares and B dollars invested at risk-free rate. The market price of risk equation says that the instantaneous expected return on V, mV, can be computed from
mV r r = sV

or mV = SN B + r, SN + B SN + B

which says that mV is the weighted average of the expected total return on the stock, , and the risk-free rate r. The discrete analogy of the above is (3.12): eh =
S0 B e h + e rh . S0 + B S0 + B

Since > r, mV r, and this means that the derivative has a risk premium and hence its price cannot be calculated from erT E[terminal payoff]. However, in Chapter 3, we have illustrated that when we move to a world where S(t) earns an expected rate of return of r (by switching from p to p*), then we can use r to discount the expected terminal payoff. Now we try to do the same for the Black-Scholes model. To this end, we first twist the expected rate of appreciation to r by the following treatment:
dS (t ) = ( )dt + dZ (t ) S (t ) = (r )dt + dZ (t ) + ( r )dt

r = (r )dt + dZ (t ) + dt = (r )dt + d[ Z (t ) + t ]

FINA 3250 | Derivatives for Actuaries I

143

C.Y. Ng

Chapter 5. The Black-Scholes Model Section 3. The Black-Scholes Equation

So, if we can create a risk-neutral world (Q measure) where


Relation between P and Q

~ Z (t ) = Z (t ) + t is a standard Brownian motion, then we should have V(S(t), t ) = er(T t ) E*[V(S(T), T ) | F (t)]. By the Girsanov theorem, such a risk-neutral world exists. In particular,

(5.4)

~ under P, {Z(t)} is a standard Brownian motion and {Z (t )} is a Brownian motion with drift and unit volatility; ~ under Q, {Z (t )} is a standard Brownian motion and {Z(t)} is a Brownian motion with drift and unit volatility.

?
(i) (ii)

Example

3. 3 [MFE Sample #61]

Assume the Black-Scholes framework. You are given: S(t) is the price of a stock at time t. The stock pays dividends continuously at a rate proportional to its price. The dividend yield is 1%.
dS (t ) = 0.05dt + 0.25dZ (t ) S (t )

(iii) The stock price process is given by

where {Z(t)} is a standard Brownian motion under the true probability measure. (iv) Under the risk-neutral probability measure, the mean of Z(0.5) is 0.03. Calculate the continuously compounded risk-free interest rate. (A) 0.030 (B) 0.035 (C) 0.040 (D) 0.045 (E) 0.050 (Ans: D)

FINA 3250 | Derivatives for Actuaries I

144

C.Y. Ng

Chapter 5. The Black-Scholes Model Section 3. The Black-Scholes Equation

After we have constructed the risk-neutral measure, we proceed to solve the Black-Scholes equation. The following proof, due to the famous physicist Richman Feynman, show that the unique solution of the Black-Scholes equation is indeed V(S(t), t ) = er(T t ) E*[V(S(T), T ) | F (t)]. The proof has its origin in quantum mechanics.
Risk-Neutral Pricing

The time-t price of a European derivative with terminal payoff V(S(T), T) is given by V(S(t), t ) = er(T t ) E*[V(S(T), T ) | F (t)], where E* signifies that in the calculation of the expectation, S(t) is assumed to follow the riskneutral dynamics ~ dS(t) = (r ) S(t) dt + S(t) dZ (t ), ~ which is equivalent to S(t) = S(0)exp[(r 0.5 2) t + Z (t )] . Proof: (Out of MFE syllabus) We first study the dynamics of V under the Q-measure. By Its lemma, dV(S, t) = Vtdt + VsdS + 0.5Vss(dS)2 ~ = Vtdt + Vs[(r )Sdt + S dZ ] + 0.5Vss( 2S 2dt) ~ = [Vt + (r )SVs + 0.5 2 S 2Vss]dt + SVs dZ ~ = rVdt + S Vs dZ (by (5.3)) Thus, V earns an expected rate of return of r. Then we let W(V, t) = ertV be the discounted price process. By Its lemma, dW(V, t) = Wt dt + WvdV + 0.5Wvv(dV)2 ~ = rertVdt + ert(rVdt + SVs dZ ) + 0 ~ = ert SVs dZ so that the discounted price process is a martingale under Q. Changing the dummy variable in the above from t to u, and then integrating from t to T gives
T ~ W [V ( S (T ), T )] W [V ( S (t ), t )] = e ru S (u )Vs ( S (u ), u )dZ (u ). t

Taking expectation (under Q) conditional on F (t), the history up to time t, and noting that It integral has a mean of 0, we get E*[erT V(S(T), T ) | F (t)] E*[ert V(S(t), t ) | F (t)] = 0. Upon the multiplication of ert on both sides, we have er(T t) E*[V(S(T), T ) | F (t)] V(S(t), t ) = 0. which is the result asserted.

FINA 3250 | Derivatives for Actuaries I

145

C.Y. Ng

Chapter 5. The Black-Scholes Model Section 3. The Black-Scholes Equation

?
(i)

Example

3. 4

For a stock whose time-t price is S(t), you are given that: The stock price process under the true probability measure is d [ln S(t)] = 0.02dt + 0.4dZ(t), (ii) The stock pays no dividends.
S (3) at time 3.

S(0) = 1

where {Z(t)} is a standard Brownian motion under the true probability measure. (iii) The Sharpe ratio of stock price risk is 0.0375. Compute the price of a contingent claim that pays

Example

3. 5

Let W(S(t), t) = er(T t)V(S(t), t ) be the time-t forward price for a delivery of V(S(T), T) at T. Show that W(s, t) satisfies the Kolmogorov backward equation

W W 1 2 2 2W = 0. + ( r ) S + S t s 2 s 2

FINA 3250 | Derivatives for Actuaries I

146

C.Y. Ng

Chapter 5. The Black-Scholes Model Section 4. Valuing a Contingent Claim

Valuing a Contingent Claim

Now we combine risk-neutral methodology with the results in Section 4.6 to price derivatives.
Power Contract

The time-t price of an option whose payoff is Sa(T) at time T is


Ft ,P ( S a ) = S a(t)exp[(r + a(r ) + 0.5a(a 1) 2)(T t)]. T

Proof:

Example

4. 1 [MFE Sample #16]

Assume that the Black-Scholes framework holds. Let S(t) be the price of a non-dividend-paying stock at time t, t 0. The stocks volatility is 20%, and the continuously compounded risk-free interest rate is 4%. You are interested in contingent claims with payoff being the stock price raised to some power. For 0 t < T, consider the equation Ft ,P [ S a (T )] = S a (t ) . T A solution for the equation is a = 1. Determine another a that solves the equation. (A) 4 (B) 2 (C) 1 (D) 2 (E) 4 (Ans: B)

FINA 3250 | Derivatives for Actuaries I

147

C.Y. Ng

Chapter 5. The Black-Scholes Model Section 4. Valuing a Contingent Claim

?
(i) (ii)

Example

4. 2 [MFE Sample #36]

Assume the Black-Scholes framework. Consider a derivative security of a stock. You are given: The continuously compounded risk-free interest rate is 0.04. The volatility of the stock is .

(iii) The stock does not pay dividends. (iv) The derivative security also does not pay dividends. (v) S(t) denotes the time-t price of the stock. 2 (iv) The time-t price of the derivative security is [ S (t )] k / , where k is a positive constant. Find k. (A) 0.04 (B) 0.05 (C) 0.06 (D) 0.07 (E) 0.08 (Ans: E)

Define the time-t risk-neutral d1 and d2 by d1 =


ln[S (t ) / K ] + (r + 2 / 2)(T t )

T t T t

and .

d2 = d1 T t =

ln[ S (t ) / K ] + (r 2 / 2)(T t )

Example

4. 3 (Binary Options)

(a) (Cash-or-nothing call) Find the time-t price of an option that pays $1 at time T if S(T) > K. (b) (Asset-or-nothing call) Find the time-t price of an option that pays S(T) at time T if S(T) > K. [Note: We can use change of numraire in Chapter 7 to transform (a) to (b) directly.]

FINA 3250 | Derivatives for Actuaries I

148

C.Y. Ng

Chapter 5. The Black-Scholes Model Section 4. Valuing a Contingent Claim

You must remember the results in Example 4.3 because they serve as the basic building blocks for the Black-Scholes formula and gap options. Here are the time-t prices for cash-or-nothing and asset-or-nothing calls and puts:
Binary Option Cash-or-nothing call Cash-or-nothing put Asset-or-nothing call Asset-or-nothing put Payoff at Maturity Time-t Price

I(S(T) > K) I(S(T) < K) S(T)I(S(T) > K) S(T)I(S(T) < K)

er(Tt)N(d2) er(Tt)N(d2) S(t)e(Tt)N(d1) S(t)e(Tt)N(d1)

The proofs of the formula for the two puts are analogous to Example 4.3(a) and (b). Now we can price any payoff that is piecewise linear in S(T). Take for an example, the following payoff:
Payoff d c S(T)

K1

K2

The payoff function can be written as d c [ S (T ) K 1 ] I( K 1 < S (T ) < K 2 ) V ( S (T ), T ) = c + K 2 K1

cK 2 dK 1 [I( S (T ) > K 1 ) I( S (T ) > K 2 )] K 2 K1

d c [ S (T )I( S (T ) > K 1 ) S (T )I( S (T ) > K 2 )]. K 2 K1 The time-t price of the derivative in terms of cash/asset-or-nothing calls is

=+

V ( S (t ), t ) =

cK 2 dK 1 r (T t ) e [ N (d 2 ( K 1 )) N (d 2 ( K 2 ))] K 2 K1 =+ d c S (t )e (T t ) [ N (d1 ( K 1 )) N (d1 ( K 2 ))]. K 2 K1

You may also use I(K1 < S(T) < K2) = I(S(T) < K2) I(S(T) < K1) and express the time-t price in terms of cash/asset-or-nothing puts.

FINA 3250 | Derivatives for Actuaries I

149

C.Y. Ng

Chapter 5. The Black-Scholes Model Section 4. Valuing a Contingent Claim

The Black-Scholes Formula

The time-t prices of a European call and put (which has strike K and time to maturity T ) on a stock which follows geometric Brownian motion with volatility and pays continuous proportional dividend at rate are c( S (t ), t ; K , T ) = S (t )e (T t ) N (d1 ) Ke r (T t ) N (d 2 ) , p ( S (t ), t ; K , T ) = Ke r (T t ) N ( d 2 ) S (t )e (T t ) N (d1 ) where d1 = Proof:
ln[ S (t ) / K ] + (r + 2 / 2)(T t )

T t

and d2 = d1 T t .

Example

4. 4 [07 May MFE #8]

Let S(t) denote the price at time t of a stock that pays no dividends. The Black-Scholes framework holds. Consider a European call option with exercise date T, T > 0, and exercise price S(0)erT , where r is the continuously compounded risk-free interest rate. You are given: (i) (ii) S(0) = 100 T = 10

(iii) Var[ln S(t)] = 0.4t, t > 0. Determine the price of the call option. (A) $7.96 (B) $24.82 (C) $68.26 (D) $95.44 (Ans: C) (E) There is not enough information to solve the problem.

Options on Other Underlying Assets

There are two methods that we can change the formula above to price options on stock indices, options on foreign currencies, and futures options.

FINA 3250 | Derivatives for Actuaries I

150

C.Y. Ng

Chapter 5. The Black-Scholes Model Section 4. Valuing a Contingent Claim

Method 1: use the same concept as in Chapter 3. Options on Change to

Stock Indices Foreign Currencies Futures

dividend yield foreign interest rate rf risk-free interest rate r

Proof: (1) Foreign currencies: The proof will be given in Section 7.5. (2) Futures: It follows from #5.11 that the Black-Scholes equation for futures option is the same as that for options on stocks when = r. The trick of lease rate in Section 7.1 then justifies this. The Option Pricing spreadsheet contains a VBA function GBlackScholes to implement the above.
Method 2: use Blacks formula Blacks Formula
c( S (t ), t ; K , T ) = e r (T t ) [ Ft ,T ( S ) N (d 1 ) KN (d 2 )]

and
p ( S (t ), t ; K , T ) = e r (T t ) [ KN (d 2 ) Ft ,T ( S ) N (d 1 )]

where d1 =

ln[ Ft ,T ( S ) / K ] + 0.5 2 (T t )

T t

and d2 =

ln[ Ft ,T ( S ) / K ] 0.5 2 (T t )

T t

= d1 T t .

Proof: Note that Ft,T(S) = S(t) exp[(r )(T t)] for stock with dividend yield .
Options on Ft,T(S)

Stock Indices Foreign Currencies Futures

S(t) exp[(r )(T t)] x(t) exp[(r rf)(T t)] F(t)

Example

4. 5

Consider an at-the-money European put futures option on crude oil. The time to maturity is four months, the exercise price is $20, the risk-free interest rate is 9% per annum, and the volatility of the futures price is 25% per annum. Find the price of the futures option. [Notice that there is a very simple formula for price of ATM futures options.]

FINA 3250 | Derivatives for Actuaries I

151

C.Y. Ng

Chapter 5. The Black-Scholes Model Section 4. Valuing a Contingent Claim

You can also use prepaid forward prices. The good thing is that and r do not appear explicitly.
Prepaid Forward Version of the Black-Scholes Formula
c( S (t ), t ; K , T ) = Ft ,P ( S ) N (d1 ) Ft ,P ( K ) N (d 2 ) T T

and
p ( S (t ), t ; K , T ) = Ft ,P ( K ) N ( d 2 ) Ft ,P ( S ) N (d 1 ) T T

where d1 =

ln[ Ft ,P ( S ) / Ft ,P ( K )] + 0.5 2 (T t ) T T

T t

and d2 = d1 T t .

This form of the Black-Scholes formula is useful in understanding exchange options in Chapter 7 and has been adopted in pricing options on a stock that pays discrete dividends. The underlying assumption of this model is that X (t ) = Ft .P ( S ) is a geometric Brownian motion with volatility . T As a result, is the volatility of the prepaid forward price.

?
(i) (ii)

Example

4. 6 [MFE 07 May #15 modified]

For a six-month European put option on a stock, you are given: The strike price is $50.00. The current stock price is $50.00.

(iii) The only dividend during this time period is $1.50 to be paid in four months. (iv) The volatility of the price of the prepaid forward that delivers 1 share after 6 months is 30%. (v) The continuously compounded risk-free interest rate is 5%. (B) $3.95 (C) $4.19 (D) $4.73 (E) $4.93 (Ans: C) Under the Black-Scholes framework, calculate the price of the put option. (A) $3.50

FINA 3250 | Derivatives for Actuaries I

152

C.Y. Ng

Chapter 5. The Black-Scholes Model Section 5. Estimation of Volatility and Expected Return

Estimation of Volatility and Expected Return

The one parameter in the Black-Scholes pricing formulas that cannot be directly observed is the volatility. We now discuss how can be estimated.
Implied Volatility

Implied volatility () is a term used by many stock analysts (though I doubt if they really understand what it is...). This is the volatility implied by the market price of an option. As an illustration, suppose that the price of a call on a non-dividend-paying stock is 1.875 when S = 21, K = 20, r = 10%, and T = 0.25. The implied volatility is the value of , that when substituted into the Black-Scholes formula, gives C = 1.875. We illustrate how the implied volatility can be found easily using Excel Solver. Step 1: Write a formula to price the call. Set = 30% (or other values) to start with. Use a cell to store as input. The price is 2.10. So the initial guess is too large. Step 2: Press Alt-T to go to Tools (), then select Solver ().

Step 3: Set Target Cell to the cell containing the call price ($E$8 above), click Value of and enter 1.875, and set the variable cell to the cell containing ($B$6 above).

FINA 3250 | Derivatives for Actuaries I

153

C.Y. Ng

Chapter 5. The Black-Scholes Model Section 5. Estimation of Volatility and Expected Return

Step 4: Click Solve. Excel reports the solution 0.234513. Alternatively, we can use the Newton-Raphson method to calculate the implied volatility. For details, see the VBA function ImpliedVol in Option Pricing and Appendix 5.
Estimation of Volatility from Historical Stock Price

Equation (4.5) gives us a way to estimate from historical stock prices. Define: n + 1 : Number of observations Si : Stock price at end of ith interval (i = 0, 1, , n) h : Length of time interval in years (= ti+1 ti, with nh = T = sample period) S0 0 S1 t1 S2 t2

Si1 ti1

Si ti

Si+1 ti+1

Sn1 tn1

Sn time tn = T

Step 1: Let ui = ln(Si / Si 1) for i = 1, 2, , n be the continuously compounded rate of returns (not annualized), then by (4.5), ui ~ N[( 2 /2)h, 2h]. Step 2: Compute the sample mean () of uis by u =
S 1 n 1 n 1 S u i = ln i = ln n . n i=1 S n S n i =1 i 1 0

2 Step 3: Compute the sample variance of uis by s u =

1 n (ui u ) 2 . n 1 i =1 1 n 2 2 In practice, 0, and thus some practitioners use s u ui . n 1 i =1

2 Step 4: Since su is the method of moments estimate of 2h, an estimate of is = s u / h .

You should check how you can use the statistics mode of your calculator to do Step 2 and Step 3.

Example

5. 1 [McDonald Example 18.8]

By using the following hypothetical data of a non-dividend-paying stock over 7 weeks, (simulated from S(0) = 100, = 10% and = 30%), estimate the annualized volatility of the stock.
Week Price ($)

1 100

2 105.04

3 105.76

4 108.93

5 102.5

6 104.8

7 104.13

FINA 3250 | Derivatives for Actuaries I

154

C.Y. Ng

Chapter 5. The Black-Scholes Model Section 5. Estimation of Volatility and Expected Return

From elementary statistics,


2 (n 1) su (n 1) 2 = ~ 2 (n 1) 2 2 h

and we can test H0: 2 = 0.09 versus H1: 2 0.09 at 5% level of significance for the data in Example 5.1: Test statistic: (n 1) 2

2 2 Critical values at 5% significance: 0.025 (5) = 12.83249, 0.975 (5) = 0.831209, thus

The 100(1 )% CI can be obtained from:


2 (n 1) 2 2 1 / 2 (n 1) P / 2 (n 1) = 1 2

Confidence Interval for 2

(n 1) 2 (n 1) 2 2 2 P 2 = 1 (n 1) 1 / 2 (n 1) /2
For Example 5.1, the 95% CI for is 17.198% 67.574%. (Note: The calculation in footnote 5 on p.607 of Derivatives Markets is wrong.)

Example

5. 2 [MFE Sample #17]

You are to estimate a non-dividend-paying stocks annualized volatility using its prices in the past nine months.
Month 1 2 3 4 5 6 7 8 9 Stock Price ($/share) 80 64 80 64 80 100 80 64 80

Calculate the historical volatility for this stock over the period. (A) 83% (B) 77% (C) 24% (D) 22% (E) 20% (Ans: A)

FINA 3250 | Derivatives for Actuaries I

155

C.Y. Ng

Chapter 5. The Black-Scholes Model Section 6. Violation of Black-Scholes Assumption

Estimation of Expected Return

Equation (4.5) states that E[R(t1, t2)] = ( 2 /2) (t2 t1) and Var[R(t1, t2)] = 2 (t2 t1). From the previous Step 2, the sample mean of the daily returns is the method of moments estimate (as well as MLE) of ( 2/2)h. By putting in the estimate of , we have = u ln S (T ) ln S (0) + + 2 /2 = + + 2 / 2 . h T

Notice again from elementary statistics that (n 1) 2 Var (u1 ) 2 h = 2(n 1) Var (u ) = = and Var 2 n n 2 2 and that u and s u (and hence ) are independent. If is known (just assume that),
Var ( ) =

2 4 2 4 1 1 Var (u ) + Var ( 2 ) = + = + . 4 nh 2(n 1) T 2(n 1) h2

It is interesting to see that while the second term dies down when n, the number of observations of returns, increases, the first term does not. This is because in the calculation of , we use only the first and the last stock price in the whole series. Thus, the variance is large. In practice, 2 is unknown and 4 << 2. This gives an estimate of SD( ) / T .

Example

5. 1 (continue)

Give a point estimate of and compute an estimate of its standard error.

Violation of Black-Scholes Assumption

The GBM model for stock price is very elegant and opens the door to many analytic results. But does this model really represent the reality? In this section we investigate this question from two aspects: volatility and distribution of returns.
Evidence from Historical Volatility: Non-constant Volatility

By using data in different part of the stock price series, we can calculate historical volatilities on different periods. The following shows the historical 60-day volatility for S&P 500 and IBM from 1991 to 2002. Each day, the preceding 60 trading days are used to compute daily returns and standard deviations, and the result is multiplied by 252 to produce an annualized volatility:
156

FINA 3250 | Derivatives for Actuaries I

C.Y. Ng

Chapter 5. The Black-Scholes Model Section 6. Violation of Black-Scholes Assumption

Notice that the use of overlapping 60-day intervals induces smoothness in the series, since each days return affects the next 60 days of volatility calculations. But even so, it is evident that historical volatility is not constant over time. This inspires researches of stochastic volatility (SV) model: dS (t ) = ( )dt + [ S (t ), X (t ), t ]dZ (t ) . S (t ) The volatility coefficient in the above can depend on stock price, other variables (e.g. interest rate), and time. Another type of volatility model is ARCH and GARCH time series model.
Evidence from Implied Volatilities: Volatility Surface

We can use all option prices available in the market (with varying strikes and time to maturity) to compute implied volatilities. The following is an example: S&P 500 call index options at 10/28/2004, S = 1127.44, = 1.85%, r = 2%, the table showing the call prices and the corresponding implied volatilities: 11/20/2004 34.80 16.3% 17.10 14.34% 5.80 12.84% Expiration 12/18/2004 41.70 15.59% 24.50 13.96% 13.00 13.36% 1/22/2005 49.10 15.67% 33.00 14.63% 20.00 13.63%

1100 Strike price 1125 1150

The plot of the table above into a 3-dimension graph is called a volatility surface. According to the Black-Scholes framework, is constant, and hence the volatility surface should be flat. But notice that the implied volatilities from the data are not the same. In particular, (1) For a given expiration date, implied volatilities decline with K (this also occurs if you use puts to calculate implied volatilities). This is called volatility skew. The skewness may take a variety of shapes such as smirk, smile and frown for options on different underlying assets. (2) The decline is evident but smaller for later expiration months.

FINA 3250 | Derivatives for Actuaries I

157

C.Y. Ng

Chapter 5. The Black-Scholes Model Section 6. Violation of Black-Scholes Assumption

That the volatility surface is not flat provides strong evidence against the Black-Scholes model. Another result of the Black-Scholes model is that daily stock returns are normal. We can use this as a testing field of the Black-Scholes model.
Violation of Normality of Returns

The following graphs show the histogram of the returns of S&P 500 index and IBM from 1991 /1/2 to 2001/10/24 and the normal distribution with mean and standard deviation equal to the sample mean and sample standard deviation of the returns.

A simple method to see if a data set comes from a normal distribution is to calculate its skewness () and kurtosis (). Skewness and kurtosis are used to measure the lack of symmetry and the thickness of the tail of a distribution. They are defined as Skew(X) =
1

E[( X ) 3 ] =

3 1 , Kurt(X) = 4 E[( X ) 4 ] = 4 . 3 4

FINA 3250 | Derivatives for Actuaries I

158

C.Y. Ng

Chapter 5. The Black-Scholes Model Section 6. Violation of Black-Scholes Assumption

For X ~ N(, 2), Skew(X) = 0 and Kurt(X) = 3. These two results can be proved by noting that X k X Skew(X) and Kurt(X) are of the form E[( ) ] , k = 3, 4, where ~ N(0, 1). Since the MGF of N(0, 1) is exp(0.5t ), Skew(X) and Kurt(X) can be obtained from a Taylor series expansion. We have the following classifications: Skewness
2

>0 =0 <0

positively / right skewed symmetric negatively / left skewed

Kurtosis

>3 =3 <3

leptokurtic mesokurtic platykurtic

It is evident from the histogram that the four return series are left skewed and leptokurtic (i.e., having a sharper peak and fatter tail than a normal distribution) and thus the returns are not normally distributed. Statistical inference can be based on the Jarque-Bera statistic

J=

n (Kurt ( X ) 3) 2 (Skew ( X ) 2 + ) 6 4

which is 2(2) under the null hypothesis of normality. (A better test is the Shapiro-Wilk test.) Another method is to draw a normal probability plot. We shall illustrate its construction with a data set.

Step 1: Arrange the data in ascending order. Step 2: For the ith order statistic, assign the quantile q = i 0.5 to it. (In statistics language, n the q-quantile of a distribution F is the value z such that F(z) = q.)

Step 3: Convert the quantiles q in Step 2 into the z values using z = N1(q). Step 4: Plot the data and the associated z. If the underlying distribution is normal, the data pairs should fall on a straight line.

Example

6. 1

Consider the five data points: {5, 4, 7, 11, 3}. Plot the normal probability plot of the data set. Data 3 4 5 7 11 Quantile assigned 0.5/5 = 10% 1.5/5 = 30% 2.5/5 = 50% 70% 90%

z value 1.282 0.524 0 +0.524 +1.282

FINA 3250 | Derivatives for Actuaries I

159

C.Y. Ng

Chapter 5. The Black-Scholes Model Section 6. Violation of Black-Scholes Assumption

The normal probability plot for the four return series considered are given on p.610 of McDonald. It is again evident that the returns are not normally distributed. Various alternative continuous-time models have been proposed to fit the stock price process observed in reality. They include (1) Mixture of two normal distributions for stock returns (2) Two-state log-normal regime-switching model (RSLN2) for stock price (3) Geometric Brownian motion augmented with jumps arriving according to a Poisson process
Appendix 5: The Newton-Raphson Method

The Newton-Raphson method is one of the best known methods to numerically calculate the roots of a real-valued function. Let the equation be f(x) = 0, where f is differentiable. The Newton-Raphson method starts with a seed x0 for which f(x0) 0 and f (x0) 0. Then the algorithm recursively calculates x1, x2, ... by
x n +1 = x n f ( xn ) , n 0. f ( x n )

Provided that x0 is close to the root and f is continuously differentiable in a neighborhood of the root (and some other minor technical conditions), {xn} converges to the root very quickly. For more information, see http://en.wikipedia.org/wiki/Newton's_method. In the calculation of implied volatility from call option, we want a value so that f( ) = c(S, K, r, , , T) Market price = 0. As a result, if we can compute f , we can numerically compute . Actually, f is the vega of the option, as we shall shortly see in Section 6.1.

FINA 3250 | Derivatives for Actuaries I

160

C.Y. Ng

Chapter 5. The Black-Scholes Model Exercise 5

Exercise 5

Section 5.2 Market Price of Risk


1. A stock index which is currently at 1535 is expected to earn a continuously compounded rate of return of 11% and pays a continuous dividend yield at 1.5%. The continuously compounded risk-free interest rate is 8% and the volatility of the index is 33%. Find the Sharpe ratio of a 1-year 1580-strike call option written on the index. Consider two interest-rate sensitive securities, both of which are positively dependent on the 90-day interest rate. Suppose that the first one has an expected return of 3% per annum and a volatility of 20% per annum, while the second one has a volatility of 30% per annum. Assume that the instantaneous risk-free rate of interest is 6% per annum. (a) Find the market price of risk of the 90-day interest rate. (b) Find the expected return of the second security. 3. The time-t price of an asset S(t) follows S(t) = 10e0.01t + 0.2Z(t) where {Z(t)} is a standard Brownian motion. The asset pays dividends continuously at a rate proportional to its price, and the dividend yield is 2%. If the continuously compounded risk-free interest rate is 4%, find the Sharpe ratio of the stock. 4. Consider two assets X and Y. There is a single source of uncertainty which is captured by a standard Brownian motion {Z(t)}. The prices of the assets satisfy dX (t ) = 0.07dt + 0.1dZ (t ) and X (t ) dY (t ) = 0.07dt 0.12dZ (t ) Y (t )

2.

You are also given that X pays dividends continuously at a rate proportional to its price with a dividend yield is 3%, and that Y pays no dividends. Find r. 5. (Power Contract) Consider the Black-Scholes framework for a stock index S paying a continuous dividend yield and a derivative that pays Y(T) = Sa(T) at time T. In Section 5.3, you will see that the time-t price of this derivative is

V(S(t), t) = Sa(t)exp[(r + a(r ) + 0.5a(a 1) 2)(T t)].


(a) Show, by Its lemma, that the Sharpe ratio of the derivative is ( r) / . (b) Let dY(t) = (S(t), t)Y(t)dt + (S(t), t)Y(t)dZ(t). Does

( S (t ), t ) r r = hold? ( S (t ), t )

(c) Show that V(S(t), t) = e[r + a( r)](T t)E[Sa(T) | S(u): 0 u t]. What does it mean?

FINA 3250 | Derivatives for Actuaries I

161

C.Y. Ng

Chapter 5. The Black-Scholes Model Exercise 5

6.

The prices of two stocks X and Y are governed by: dX (t ) dY (t ) = 0.06dt + 0.25dZ (t ) , = 0.01dt + dZ (t ) . X (t ) Y (t ) where Z(t) is a standard Brownian motion and < 0 is a constant. You are given: (i) (ii) The current stock prices are X(0) = 20 and Y(0) = 50. Both stocks pays dividend at a rate proportional to its price. The dividend yield of X is 3%, while the dividend yield of Y is 2%.

(iii) The continuously compounded risk-free interest rate is 4%. To construct a zero-investment, risk-free portfolio in which there is 100 shares of X, one needs to trade a certain number of Y and borrow c dollars at risk-free rate to finance any net cost. Find c. 7. (DM 20.11) Suppose that the SDE followed by two non-dividend-paying traded assets S1 and S2 are given by dSi(t) = iSi(t)dt + i Si(t)dZi (t), Si(0) = si where {Zi(t): t 0}, i = 1, 2 are two standard Brownian motions. (a) Assume that Z2(t) = Z1(t) so that they are perfectly correlated. A market-maker underwrites k1 units of S1 at t = 0. If he hedges his position by using S2, how many units of S2 should he long or short at t = 0? Express your answer in terms of k1, 1, 2, s1 and s2. Can the market-maker obtain a perfect hedge if Z1(t) and Z2(t) are not perfectly correlated but has a correlation of 1 ? (b) (APT) A non-dividend-paying asset Q follows the stochastic differential equation dQ(t) = QQ(t)dt + Q(t)[1dZ1(t) + 2dZ2(t)]. Show that for the securities market to be arbitrage-free, Q r = 1

1 r r + 2 2 . 2 1

Section 5.3 The Black-Scholes Equation


8. (DM 21.2) Verify that ASa(t)et satisfies the Black-Scholes equation for

2(r ) 1 r r 1 a = 2 2 + . 2 2 2 9. Let S be a stock that provides a continuous dividend yield . (a) Show that V(S(t), t) = S(t)e(T t) Ker(T t) satisfies the Black-Scholes equation. (b) What is this option? (c) What is the price of this option at time 0? So how should the value of K be chosen? 10. (a) Show that V ( S (t ), t ) = S 1 (t )e (
2

+ 2 r )(T t )

satisfies the Black-Scholes equation.

(b) What is the terminal payoff of the corresponding European option?

FINA 3250 | Derivatives for Actuaries I

162

C.Y. Ng

Chapter 5. The Black-Scholes Model Exercise 5

11. Suppose that the futures price {F(t): 0 t T} follows dF(t) = F(t)dt + F(t)dZ(t). Let V(f, t) be the price of a derivative on the futures when the time-t futures price is f. At time t, we form a portfolio that shorts 1 unit of the derivative and enter into Vf (f, t) units of futures. (a) Explain why the value of the portfolio is ( F (t ), t ) = V ( F (t ), t ) . (b) Show that the change in the wealth of the portfolio holder in time [t, t + dt] is V 1 2V 2 2 t + 2 f 2 F dt , which is instantaneously riskless. Hence, show that the corresponding Black-Scholes equation for futures options is V 1 2V 2 2 + F = rV . t 2 f 2 This equation is the same as the Black-Scholes equation for stock index when = r. A futures price can be treated in the same way as a stock providing a dividend yield at rate r for the purpose of valuing derivatives. (This is a point that I have stressed in some of the previous end-of-chapter exercises.) 12. Consider a futures contract on an asset S. The futures contract matures at T and the futures price at time t ( T) is denoted by Ft (so that FT = ST). Show that in the risk-neutral world, F0 = E*(ST). This result shows that the forward price (assuming that forwards and futures are the same) predicts precisely the expected future spot price of an asset in the risk-neutral world. Compare this with the discussion in Section 1.2. 13. For a stock whose time-t price is S(t), you are given: (i) (ii) The stock pays dividends continuously at a rate proportional to its price. The dividend yield is 1%. The stock price process is given by dS (t ) ~ = 0.08dt + dZ (t ) S (t )

~ where {Z (t )} is a standard Brownian motion under the risk-neutral probability measure and > 0 is a constant. dS (t ) (iii) Under the true probability measure, Var S (t ) = 0.04dt. S (t ) ~ (iv) Under the true probability measure, the mean of Z (2) is 0.3. Calculate , the expected total return of the stock.

FINA 3250 | Derivatives for Actuaries I

163

C.Y. Ng

Chapter 5. The Black-Scholes Model Exercise 5

14. Assume the Black-Scholes framework holds for a non-dividend-paying stock. (a) Consider an American call option with strike K. Does the Black-Scholes equation hold for this option? Why? (b) Repeat for an American put option with strike K. 15. (Ratio derivative) For a non-dividend-paying stock S, you are given that the risk-neutral stock price process is ~ d ln S (t ) = 0.065dt 0.2dZ (t ) , ~ where {Z (t )} is a standard Brownian motion under the risk-neutral probability measure. Calculate the time-0 price of contingent that pays S(4) / S(2) at time 4. 16. (DM 21.10) Suppose that a European derivative makes continuous payments at rate V. Show that the Black-Scholes equation becomes V V 1 2 2 2V = (r V )V . + ( r ) S + S t s 2 s 2 Section 5.4 Valuing a Contingent Claim 17. (DM 20.7) Assume the Black-Scholes framework. Suppose S(0) = 100, r = 6%, = 40% and = 0. Compute the price of the following contingent claims: (a) S 2(1), (b) S 0.5(1), (c) S 2(1).

18. For a non-dividend-paying stock whose time-t price is S(t), you are given that: (i) The stock price process is d [ln S(t)] = 0.3dZ(t), where Z(t) is a standard Brownian motion under the true probability measure. (ii) The continuously compounded risk-free interest rate is 0.04. If F0P1 ( S 2 ) = e E[S 2(1)], find . , 19. (Price of straddles and strangles) Assume the Black-Scholes framework. Find the price of an European option that pays |S(T) K| at time T. What if the payoff is [S(T) K2]I(S(T) > K2) [S(T) K1]I(S(T) < K1) ? 20. (Log contract) Consider a stock S that follows the usual Black-Scholes framework. An innovative financial institution has just announced that it will trade a security that pays off a dollar amount equal to ln S(T) at time T. For simplicity, assume S(0) is sufficiently large that the payoff is non-negative. (a) Use risk-neutral valuation to calculate the price of the security at time t in terms of S(t). (b) Confirm that your price satisfies the Black-Scholes equation.

FINA 3250 | Derivatives for Actuaries I

164

C.Y. Ng

Chapter 5. The Black-Scholes Model Exercise 5

21. Assume the Black-Scholes framework for a non-dividend-paying stock S. You are given: (i)
F1,3 ( S ) ln = 0.16 S (1) dS (t ) (ii) E S (t ) = 0.15dt S (t ) (iii) The Sharpe ratio of stock price risk is 0.14.

Calculate ln [S(1)F1,3(S1)]. 22. Assume the Black-Scholes framework. You are given: (i) (ii) The current stock price 10. The stock pays dividends continuously at a rate proportional to its price. The dividend yield is 0.015.

(iii) The volatility of the stock is 0.3. (iv) The continuously compounded risk-free interest rate is 0.06. Compute the price of a contingent claim that pays S(1)S(2) at time 2. 23. (Power option) Assume the Black-Scholes framework. (a) Find the time-0 price of a contingent claim that pays [Sa(T) Ka]+ at time T in terms of S(0), , K, a, r, and T. Assume for simplicity that a > 0. [Hint: use the formula for partial expectation.] (b) Find the corresponding price a contingent claim that pays [Ka Sa(T)]+ at time T by using put-call parity. (c) Program your formulas in Excel and check your result with the Option Pricing spreadsheet. 24. (DM 12.6) Suppose XYZ is a non-dividend-paying stock. Suppose S(0) = 100, = 40%, and r = 6%. (a) What is the price of a 105-strike call option with 1 year to expiration? (b) What is the 1-year forward price for the stock? (c) What is the price of a 1-year 105-strike call option, where the underlying asset is a futures contract maturing at the same time as the option? 25. (DM 12.5) The exchange rate is 95/, the yen-denominated interest rate is 1.5%, the eurodenominated interest rate is 3.5%, and the exchange rate volatility is 10%. (a) What is the price of a 90-strike yen-denominated euro put with 6 months to expiration? (b) What is the price of a 1/90-strike euro-denominated yen call with 6 months to expiration? [This question is also related to Section 7.4.]

FINA 3250 | Derivatives for Actuaries I

165

C.Y. Ng

Chapter 5. The Black-Scholes Model Exercise 5

26. Suppose that the spot exchange rate is $1.2/. The exchange rate has a volatility of 0.15. Assume that the US dollar interest rate is 0.05 and the euro-denominated interest rate is 0.02. Calculate the Black-Scholes price (in US) of a call option to buy 100 Euros with 100 USD six months from now. 27. By using the Blacks formula, explain why the price of an ATM European call futures option always equals the price of a similar ATM put futures option. Note: We have obtained a similar result for stock options using put-call parity in Example 1.2 in Chapter 2. 28. (DM 12.7, 12.8) Suppose S(0) = 100, = 30%, = 3%, and r = 8%. (a) Compute the price of a 95-strike 9-month call on the stock. (b) Compute the price of a 9-month call for which S(0) = 100e0.03 0.75, K = 95e0.08 0.75, = 30%, = 0, and r = 0. (c) What is the 9-month forward price for the stock? (d) Find the price of a 9-month 95-strike call on a futures contract maturing after 9 months. (e) What is the relationship between your answer to (d) and the price you computed in (b)? Why? 29. Consider a non-dividend-paying stock that follows a geometric Brownian motion. You are given: (i) (ii) The current stock price is 100. The stocks volatility is 0.2.

(iii) The continuously compounding risk-free interest rate is 0.04. Consider an option maturing at T = 1. Let S(1) be the time-1 price of the stock. The option pays 10 if 110 < S(1) < 120, and pays 20 if S(1) > 120. It pays nothing if S(1) < 110. Calculate the time-0 value of this option. 30. (DM 12.9) Assume K = 40, = 30%, r = 8%, and T = 0.5, and the stock is to pay a single dividend of 2 tomorrow, with no dividends thereafter. (a) Suppose S(0) = 50. What is the price of the European call? Consider an otherwise identical American call. What is its price? (b) Repeat, only suppose S(0) = 60. (c) Under what circumstance would you not exercise the option today? [Hint: Use Solver.] 31. For a stock whose current price is 50, the volatility of the prepaid forward price for a delivery of one share after 3 months is 30%. If a dividend of 1.5 is expected in two months and the continuously compounded risk-free interest rate is 10%, calculate the price of a three-month 50-strike European put on the stock.

FINA 3250 | Derivatives for Actuaries I

166

C.Y. Ng

Chapter 5. The Black-Scholes Model Exercise 5

32. You have ordered a Rolls Royce car. (i) (ii) The price of the car is 200,000 British pounds, which you will pay when the car is delivered to you in three months. The current exchange rate is two US dollars per British pound, and you will have US $400,000 three months from now.

(iii) Because the US dollar may lose value, you now buy 3-month at-the-money currency options of the European type to exactly cover the shortfall in case it occurs. (iv) The continuously compounded risk-free interest rates in the US and the UK are 5% and 9%, respectively. (v) The natural logarithm of the pound-per-dollar exchange rate is an arithmetic Brownian motion with variance per year being 4%.

Determine total cost of the currency options in US dollars. 33. (Supershare option: related to p.709 of Derivatives Markets) Assume the usual BlackScholes framework for a stock that pays dividends continuously at rate . A supershare option entitles its holder to a time-T payoff of S(T) / KL if KL S(T) KU and 0 otherwise. The time-t price of such option can be expressed as V(S(t), t) = Ft ,P ( S )[ N (d1 ) N (d 2 )] / K L . T Express d1 d2 in terms of KL, KU, , t and T. 34. (09 FIN3250 Final) Consider a firm that has assets worth $500 million currently. There are two 2-year zero-coupon debt issues with different priorities: a senior debt with face value $230 million and a junior debt with face value $120 million. At the end of 2 years, the senior debt holder is paid first, followed by the junior debt holder. The remains (if any) belong to the shareholders. The firm does not pay any dividends, and the volatility of the firms asset is 30%. The continuously compounded risk-free interest rate is 8%. (a) Assuming the Black-Scholes framework for the firms asset, calculate the current market value of the firms equity. (b) Sketch the payoff of the senior debt holder after 2 years as a function of the firms asset after 2 years. Hence, calculate the current market value of the firms senior debt. (c) What is the expected rate of return of the junior debt? (d) How would the answers in (a) and (b) change if the volatility of the firms asset increases, keeping everything else unchanged? You are not required to give reasons. 35. (09 FIN3250 Final) Assume that the Black-Scholes framework holds. The time-t price of a non-dividend-paying stock S follows S(t) = exp[0.06t + 0.3Z(t)], where {Z(t): t 0} is a standard Brownian motion under the actual measure. (a) A European option pays X 1 = S (1) S (2) at time 2, while another European option pays

FINA 3250 | Derivatives for Actuaries I

167

C.Y. Ng

Chapter 5. The Black-Scholes Model Exercise 5

X 2 = S (2) / S (1) at time 2. Find the correlation between X1 and X2 as seen at time 0.

(b) The continuously compounded risk-free rate of interest is 4%. A European call pays 100(ln X2 0.0875)+ at time 2. Find the time-0 price of the call. Section 5.5 Estimation of Volatility and Expected Return 36. (DM 18.7) Suppose you observe the following month-end stock prices for stocks A and B: 0 100 100 1 105 105 Month 2 102 150 3 97 97 4 100 100

Stock A Stock B For each stock:

(a) Compute the average monthly continuously compounded return. What is the annual return? (b) Compute the mean monthly standard deviation. What is the annual standard deviation? 37. You are given the following historical prices of a non-dividend-paying stock: Week 1 2 3 4 5 6 Stock Price 100 98 96 99 102 103

Estimate the stocks expected rate of appreciation and volatility. 38. Consider the following hypothetical data of a non-dividend-paying stock over 7 months (simulated from S(0) = 40, = 8% and = 30%).
Month Price ($)

1 40

2 43.185

3 42.012

4 43.327

5 39.167

6 41.232

7 42.737

(a) Estimate , the annualized volatility, and give a 90% confidence interval of . (b) Test the hypothesis that the annualized volatility is 30%. (c) Estimate , the expected annualized return. Section 5.6 Violation of Black-Scholes Assumptions 39. State two reasons for the volatility skew observed in equity options.

FINA 3250 | Derivatives for Actuaries I

168

C.Y. Ng

Chapter 5. The Black-Scholes Model Exercise 5

40. (Hull 13.19) A stock price is currently $50 and the risk-free interest rate is 5%. By using Option Pricing, translate the following table of European call options on the stock into a table of implied volatilities, assuming no dividends. Strike Price 45 50 55 3 7.0 3.7 1.6 Maturity (months) 6 8.3 5.2 2.9 12 10.5 7.5 5.1

Are the option prices consistent with the Black-Scholes assumption? 41. Two normal probability plots have been drawn. Plot (I) is based on the data set {7, 3, 1, 5, 9}. Plot (II) is based on the data set {1, 9, 7, 11, 6, 13}. Let y1 and y2 be the y-coordinates for data point 7 in the two plots. Calculate y1 y2. Numerical Answers 1. 2. 3. 4. 5 6. 7. 9. 10. 13. 14. 15. 17. 18. 19. 20. 21. 22. 23. 9.0909% (a) 15% (b) 1.5% 0.05 0.0864 (b) (S(t), t) = a( ) + 0.5a(a 1) 2, (S(t), t) = a , No. Y(t) is not the price of an asset. The market price of risk equation only hold for prices. (c) The risk premium of Y is a( r). 12000 k s m 1 1 1 2 s2 (b) Forward (c) 0, K = S(0)exp[(r )T] 1 (b) S (T) 0.12 (a) Yes (b) Not always. Hold only if it is not optimal to exercise. 0.8437 (a) 12461 (b) 9.5123 (c) 1.35104 0.05 The sum of prices of a call and a put: S(0)eT[2N(d1) 1] KerT[2N(d2) 1] (a) V(S(t), t) = e r (T t ) [ln S (t ) + (r 0.5 2 )(T t )] 0.34 111.0711 (a) F0PT ( S a ) N (d1 ) KaerTN(d2) where F0PT ( S a ) = erT Sa(0)exp[a(r )T + 0.5a(a 1) 2 T] . .
d2 = ln[ S (0) / K ] + (r 0.5 2 )T

, d1 = d 2 + a T

FINA 3250 | Derivatives for Actuaries I

169

C.Y. Ng

Chapter 5. The Black-Scholes Model Exercise 5

24. 25. 26. 28.

29. 30. 31. 32. 33. 34. 35. 36. 37. 38.

(b) KaerTN(d2) F0PT ( S a ) N ( d1 ) . (a) 16.33 (b) 106.1837 (c) 16.33 (a) 1.029911 (using Excel, get 1.0483) (b) 0.00012 (using Excel, get 1.22604 104) = (a) / (95 90) 21.3599 (using Excel, get 21.4147) (a) 14.3863 (b) 14.3863 (c) 103.8212 (d) 14.3863 (e) A futures option whose underlying futures matures at the same time as the option matures is equivalent to an option on the underlying asset. This is because the terminal payoff is the same: F(T) = S(T). 5.3900 (a) 10.34731, same as European (b) 19.69032, 20 (c) S(0) < 53.33543 3.03268 by Excel 13800 ln( K H / K L )

T t

(a) 210.1364m (b) 195.4244m (c) 11.977% (d) , (a) 0.4372 (b) 2.33518 (a) all 0 (b) for stock A: 4.717%, 16.34% for stock B: 32.69%, 113.24% 0.32426 and 0.18356 (a) 22.43%, (15.07%, 46.86%) (b) Test statistics = 2.795, not rejected at 1%, 5% and 10 % level of significance (c) 15.75% 39. (1) Leverage: As stock price decreases, the companys leverage increases. This means that the stock becomes more risky and its volatility increases, and vice versa. (2) Crashophobia: Traders are concerned about the possibility of big market crash similar to that experienced in October 1987, and they price options accordingly. 40. The following table shows the implied volatility (in units of %) Stock Price 45 50 55 41. 0.73 3 37.78 34.15 31.98 Maturity (months) 6 34.99 32.78 30.77 12 34.02 32.03 30.45

FINA 3250 | Derivatives for Actuaries I

170

C.Y. Ng

Chapter 2B. Extra Materials for Exam FM Section 1. More on Buying a Stock Option

CHAPTER 2B
Related Readings

Extra Materials for Exam FM

Derivatives Markets (2nd edition)


Section 1.3, 2.6, Appendix 2A, Chapter 4

L earning Objectives Practical issues in buying and exercising options, paylater strategy, uses of derivatives and tax consequences

There are many sections in McDonalds Derivatives Markets that are about market operations and minor applications of derivatives. They are not pertinent to the understanding of the mathematical foundation of derivatives or the general usage of derivatives in mainstream market. I find it quite distracting to put these materials in the main lecture notes because they seriously disrupt the logical flow of the mathematics there. However, questions related to these materials can appear in Exam FM. So, these minor topics are collected and presented in this addendum. 1

More on Buying a Stock Option

The following is a continuation of the discussion of Section 2.1 about options (the top of p.38 of the main notes). There are many practical issues that an option buyer should be aware of. Margin requirement A purchased call or put position does not entail a loss. So there is no need to post any margin. What you would be afraid of is (1) the option would finally be OTM, or (2) the option is finally ITM but the seller is unable to fulfill his obligation to delivery the underlying or cash. That is, there is credit risk of options that are not exchange-traded. As a result, for a written option position, one needs to post collateral because there can be large loss if the stock moves against the writer.
FINA 3250 | Derivatives for Actuaries 171

C.Y. Ng

Chapter 2B. Extra Materials for Exam FM Section 1. More on Buying a Stock Option

Margin rules change over time. Also, different option positions have different margin rules. Costs involved in Exercising Some European options (including all cash-settled ones) are automatically exercised at maturity. For options that would not be automatically exercised (e.g., American options), one must provide exercise instructions prior to brokers deadline. When one exercises, one generally pays commission to the broker. If the option is a call with physical settlement, a stock would be delivered from the option writer. The option writer who is assigned to deliver to the stock may request commission. One needs to pay commission again if he wishes the sell the stock. Taxes Tax rules for derivatives are extremely complicated because different countries adopt different tax laws. Tax treatments in the US (GAAP FAS 133) and EU (IRFS IAS 39) are not the same. In what follows we talk about US GAAP. Another reason why tax is complicated is because interest income, dividend income, and capital gains or losses are taxed differently. For example, capital loss can be used to deduct tax, and dividend incomes are taxes more heavily than interest income in the US. The general tax principles say that gain or loss is taken into account for tax purposes when realized. For dividends and interest that is the time when they are received or paid. For capital asset, a gain or loss is realized at the time the asset is sold, exchanged, or otherwise disposed of. Example 1: If you purchase a call and then sell it without exercising, then the tax treatment is to treat the gain or loss as capital gains or losses on a stock, and accords long-term or short-term capital gains treatment depending on the length of time for which the position has been held. Example 2: If you purchase a call and exercise it, then the law says that it is treated as capital gains or losses, at a cost basis equal to the strike plus option price plus any commission paid. However, the holding period counts only how long you hold the stock. Derivatives can be used to modify the position of a capital or turn one kind of income (say dividends) into another. For example, an investor who owns a stock can hedge the risk by shorting forward or futures (cash-and-carry). The tax treatment for this is not to treat the stock and forward position separately but views that as a bond position. This is called constructive sale: if you own a stock, then entering into certain option or forward positions would mean that even if you continue to own the stock, for tax purposes you are deemed to have sold it. Straddle rules are tax rules intended to control the recognition of losses for tax purposes when there are offsetting risks such as a constructive sale.

FINA 3250 | Derivatives for Actuaries

172

C.Y. Ng

Chapter 2B. Extra Materials for Exam FM Section 2. Paylater Strategy and Equity-linked CD

Paylater Strategy and Equity-linked CD

The following is a continuation of the discussion of the use of collar. It is a continuation of the materials on p.47, immediately before strangles and straddles. Paylater strategy A paylater strategy is an option strategy that has a cost of zero at the beginning. (1) The simplest strategy is a short forward contract. (2) A more complicated strategy is a zero-cost collar where a put with strike K1 is purchased, a call with strike K2 is sold short, the value of K2 is selected such that c(S0, K2, T) = p(S0, K1, T), so that the collar cost is 0. Such a zero-cost collar can be used by a manufacturing company that produces the underlying asset S. To be more specific, by purchasing m units of the collar, the company can guarantee that it can sell m units of the underlying in the price range (K1, K2) at time T.

ST K1 K2 K1 K2

ST

Collar On combining, we get the following payoff diagram:

Stock

ST K1 K2 Collared stock = Bull spread Notice that if K1 is chosen to be the forward price, then by put-call parity, K2 = K1, and this zero-cost collar is reduced to a short forward contract.

FINA 3250 | Derivatives for Actuaries

173

C.Y. Ng

Chapter 2B. Extra Materials for Exam FM Section 2. Paylater Strategy and Equity-linked CD

(3) Still another kind of paylater strategy is a carefully constructed ratio spread. Consider short-selling m units of K2-strike put; buying n units of K1-strike put and where K2 > K1; n is determined by the equation np(S0, K1, T) = mp(S0, K2; T). (Note: since put price is a non-decreasing function of strike, we generally have n > m.) Consider a company that produces (n m) units of the underlying. If he purchases the ratio spread above, the time-T payoff (or profit, because the cost of the spread is 0) is
(n m) S T + n( K 1 S T ) m( K 2 S T ) = nK 1 mK 2 (n m) S T m( K 2 S T ) = nS T mK 2 ( n m) S T ST K1 K1 < ST < K 2 . ST K 2

So, the payoff is for sure not less than nK1 mK2. Note, also that the payoff for the case ST > K2 is the same as an unhedged position. Compare the above with a simple strategy of guaranteeing a minimum sales price by purchasing (n m) units of K2-strike put. The profit in this case would be
(n m)[ K 2 e rT p ( S 0 , K 2 , T )] S T < K 2 rT (n m)[ S T e p ( S 0 , K 2 , T )] S T K 2

Profit

Profit

(n m)K2 ST K1 nK1 mK2 using ratio spread K2

K1

K2

ST

using K2-strike put

The most contrasting difference of the two strategies above is that for the ratio spread, the payoff is not deducted when ST > K2. The cost of the insurance against shortfall in this case is compensated by a more negative (but however still limited) payoff when ST < K1. This cost of insurance against shortfall is paid when a shortfall arises.

FINA 3250 | Derivatives for Actuaries

174

C.Y. Ng

Chapter 2B. Extra Materials for Exam FM Section 2. Paylater Strategy and Equity-linked CD

The following is a discussion of equity-linked certificate of deposit (CD). It is a follow-up of the discussion of straddle and strangles on p.47. Equity-linked CD Recall from FINA3080 that a certificate of deposit is a time deposit. It is similar to savings account in that they are insured (in the US, by FDIC) and thus risk-free. However, they have a fixed term (often 3 months to a few years) and pay a fixed interest rate. It is intended that the CD be held until maturity. There can be a penalty upon early withdrawal. For equity-linked CD (also called variable-rate CD), there is not a fixed interest rate. The return on principle is linked to an equity index instead. Consider a 5.5 year CD with a return linked to the S&P 500. At maturity the CD is guaranteed to repay the principal plus 70% of the S&P 500 return, computed using a simple return. Such a payoff function can be written as S 0.7 L1 + 0.7 5.5 1 = L[1 + ( S 5.5 S 0 ) + ] , S0 S0 + where L is the principle at t = 0. The fair value of this contract at t = 0 is 0.7 L[1 + c( S 0 , S 0 , 5.5)] . S0 The decomposition of an instrument into more fundamental securities is called reverse financial engineering. It is now easy to see that an investor can synthesize such an equity-linked CD by using a 5.5-year bond and a 5.5 year ATM European call (though it is not easy to find such a long-term call). So why there is still a market for these kind of instruments?

The idea is actually simple. Supermarkets sell whole heads of lettuce, and also salad made of lettuce already washed and torn into bite-sized pieces. The transaction cost of the salad preparation leads some customers to prefer the pre-packaged salads. Some complicated instruments can likewise be decomposed into calls and puts. Consider a mandatorily convertible bond, with 6.5% annual coupon rate, and at maturity make payments in shares, with the number of shares dependent upon the firms stock price, as follows: Share price ST < 37.32 37.32 ST 46.28 ST > 46.28 Number of Shares Paid by Bondholders 0.6699 25 / ST 0.5402

Verify that the maturity payment can be expressed as


1 0.6699 S T ( S T 37.32) + + ( S T 46.28) + . 1.24

FINA 3250 | Derivatives for Actuaries

175

C.Y. Ng

Chapter 2B. Extra Materials for Exam FM Section 3. Uses of Derivatives

Uses of Derivatives

The textbook discusses the difference between diversifiable and non-diversifiable risk. According to the textbook, a risk is diversifiable if it is unrelated to other risks. A risk is nondiversifiable if it does not vanish when spread across many investors. An example of diversifiable risk includes the risk that a lightning strike causing a factory to burn down. An example of non-diversifiable risk includes a steep rise in interest rate, causing all bond prices to plumb. The above is a very vague definition. A more precise description used in portfolio theory in finance is given below: Diversifiable / unsystematic risk is risk that is specific to a particular security or sector so its impact on a diversified portfolio is limited. An investor would not be rewarded for higher expected return by taking more diversifiable risk because such a risk can be virtually eliminated from a portfolio through diversification. Diversifiable risk can be due to factors specific to an industry like elasticity of demand of product, or a company like labor unions, research and development, marketing strategy etc. Non-diversifiable / systematic risk is risk that cannot be reduced or eliminated through diversification. It is the relevant portion of an assets risk attributable to market factors that affect all firms and securities such as wars, inflation, and international incidents. In financial markets, while non-diversifiable risks cannot be eliminated, it can be shared: to be held by those most willing to hold it. Such a risk-sharing mechanism benefits everyone, and derivatives can be a very effective way to achieve this goal when used properly. The following is a condense discussion on the use of derivatives. It is a continuation of p.1 of the main lecture notes.
(1) Risk management / hedging

Derivatives are effective in hedging away non-diversifiable risk. The textbook discusses a few reasons why companies hedge. You need to have some elementary finance background to under the following factors:
Taxes:

In the US the tax system permits a loss to be offset against a profit from a different year. So, companies have intentions to use derivatives to shift income. Also, only gains are taxed. Such tax rules entice firms to use derivatives to hedge. Take the following as a simple illustration. Consider a firm that produces one unit per year of a product costing 10. The product can sell for S1 = 11.2 or 9 with 50% probability. The profit after 1 year is thus 1.2 or 1. The expected profit is 0.5(1.2 1) = 0.1. However, if the tax rate is 40%, the after-tax profit is 0.5(1.2 0.6 1) = 0.14. Now suppose that the time-1 forward price is 10.1, and the firm short forward to hedge the risk.

FINA 3250 | Derivatives for Actuaries

176

C.Y. Ng

Chapter 2B. Extra Materials for Exam FM Section 3. Uses of Derivatives

The hedged position is S1 (S1 10.1) = 10.1, and hence the profit is (10.1 10) 0.6 = 0.06. The forward contract transfers net income from a less-valued to a more highly valued state and rises the expected value of cash flows.
Increased debt capacity:

Companies general prefer using debt financing over equity financing because debt financing is cheaper (this is again to tax treatment: interest payments to debt holder are tax-deductible). However debt financing substantially increases the risk of bankruptcy because interest payments cannot be defaulted. A firm that has more stable earnings has a greater debt capacity and derivatives can be used to reduce the volatility of earnings.
Bankruptcy and distress costs:

A large loss can threaten the survival of a firm. If a firm is in financial distress, then it would find it hard to retain customers, suppliers, and external financing (debt) would become more costly. To reduce the financial and non-financial finance distress cost, it would be better to hedge.
Managerial risk aversion:

Managers are usually risk-adverse because unlike normal investors, their salaries, bonus, and compensation options are all tied to the firm. They benefit from reducing uncertainty of the firm, even though that may not to the best interest of the firm.
Non-financial risk management:

However, there are many risks that cannot be hedged away using derivatives. For example, a US company that receives income by selling keyboard and mice in Japan may use foreign currency options to control currency risks. However, the company cannot use derivatives to hedge away risk result from the possibility of lawsuits for repetitive stress injuries resulted from bad product design. To hedge such risks, insurance is the most effective way. There are many reasons why some companies decide not to hedge using derivatives. Transacting in derivatives entails extra transaction costs, such as commissions and bidask spread. The firm needs to assess costs and benefits of different strategies, and this needs costly expertise. The firm must monitor transactions and have managerial controls in pace to prevent unauthorized trading. The firm must be prepared for tax and accounting consequences.

FINA 3250 | Derivatives for Actuaries

177

C.Y. Ng

Chapter 2B. Extra Materials for Exam FM Section 3. Uses of Derivatives

(2) Speculation

Investors can make bets that are highly leveraged by paying less or no upfront costs. It is much cheaper to bet on the stock price to increase by purchasing call options then by purchasing the underlying stock. Actually short-selling is also treated as a form of speculation because it means borrowing upfront, hoping to cover the position at a lower price.
(3) Arbitrage

One may take profit by observing mispricing in the market. There are many sophisticated investors who are constantly waiting for mispricing on stocks and derivatives to make a living.
(4) Regulatory arbitrage

One may use derivatives to circumvent regulatory restrictions, taxes and accounting rules (e.g. if you wish to keep the voting right of a stock, but eliminate the risk, do cash-and-carry).

FINA 3250 | Derivatives for Actuaries

178

You might also like